You are on page 1of 62

1

Some Basic Concepts


of Chemistry
Mole Concept Gram Atomic, Gram Molecular Weight (M) It is
the weight of 1.0 mole (Avogadros numbers) of atoms,
l
One Mole Avogadros Number ( N A ) = 6.023 1023 . It
molecules or ions in gram unit.
is the number of atoms present in exactly 12 g of ( C12 )
isotope. M = A amu Avogadro number = A gram
l
Atomic Weight (A) Atomic weight is the relative weight Hence, gram molecular weight (M) is numerically equal to
of one atom of an element with respect to a standard weight. the atomic weight or (molecular weight) in gram unit because

Weight of one atom of an element 1.0 mole of amu is 1.0 g.


A =
1
th part by weight of an atom of (C12 ) isotope Empirical and Molecular Formula Empirical
12 formula is the simplest formula of a compound with the
l
amu (atomic mass unit) Weight elements in the simple whole number ratio, and a molecular
1 formula is same or a multiple of the empirical formula.
1 amu = th part by weight of an atom of ( C12 ) isotope e.g.
12
Molecular formula Empirical formula
1
= g = 1.66 10-24 g C6 H6 (benzene) CH
NA
C6 H12O6 (glucose) CH2O
Atomic weight (A) amu = Absolute atomic weight. H2O2 HO

NOTE H2S 2 O8 (persulphuric acid) HSO4


l
Atomic weight is a relative weight that indicates the relative
heaviness of one atom of an element with respect to amu weight. l
Laws of Chemical Combination Elements combine
l
Atomic weight has no unit because it is the ratio of weights. in a fixed mass ratio, irrespective of their supplied mass ratio,
l
One mole of an amu = 1.00 g. e.g.
1
l
Change of Scale for Atomic Weight If an amu is H2 + O H2 O
2 2
defined differently as (1/x)th part by weight of an atom of 2g 16 g 18 g
(C12 ) isotope rather (1/12)th part then the atomic weight ( A ) Here, H2 and O2 combines in a fixed mass ratio of 1 : 8.
can be derived as : No matter in what ratio we mixed hydrogen and oxygen, they
x
A = A will always combine in 1:8 mass ratio (stoichiometric mass
12 ratio).
Where, A = conventional atomic weight l
Limiting Reactant It is the reactant that is consumed
Molecular Weight (MW) Like atomic weight, it is the completely during a chemical reaction. If the supplied mass
relative weight of a molecule or a compound with respect to ratio of reactants are not stoichiometric ratio, one of the
amu weight. reactant is consumed completely leaving parts of others
unreacted. One that is consumed completely is known as
Molecular weight
limiting reactant.
Weight of one molecule of a compound
= Limiting reactant determine the amount of
1
th part by weight of an atom of C12 isotope product in a given chemical reaction
12
2 Some Basic Concepts of Chemistry

Concentration Units Equivalent Concept, Neutralisation


Normality (N) It is the number of gram equivalent of
l
and Redox Titration
solute present in one litre of solution : l
Equivalent Weight Equivalent weight of an element is
Eq
N = that part by weight which combines with 1.0 g of hydrogen or
V ( in litres ) 8.0 g of oxygen or 35.5 g of chlorine.
(i) Molarity (M) It is the moles of solute dissolve in (i) Equivalent weight of a salt (EW)
one litre of solution. Molar mass
=
n Net positive (or negative) valency
M = : n = Number of moles of solute
V e.g. Equivalent weight
(\V = Volume of solution in litre) M M M
CaCl 2 = , AlCl 3 = , Al 2 (SO4 )3 =
Molarity (M ) Volume (V ) = n (moles of solute) 2 3 6
If volume is in mL; MV = millimoles Molar mass
(ii) Equivalent weight of acids =
If d (g/cc) is density of a solution and it contains x % of solute Basicity
of molar mass M, its molarity can be worked out as e.g. Equivalent weight
M
1000 dx 10dx HCl = M (basicity = 1); H2SO4 = (basicity = 2)
Molarity = = 2
100 M M
M
H3PO4 = (basicity = 3)
(ii) Molality (m) It is the number of moles of solute 3
present in 1.0 kg of solvent. Molar mass
(iii) Equivalent weight of bases =
Acidity
Moles of solute ( n )
m= 1000 e.g. Equivalent weight
Weight of solvent in gram M M
NOTE NaOH = M, Ca(OH)2 = , Al(OH)3 =
2 3
Molality is a true concentration unit, independent of temperature
while molarity depends on temperature. l
The number of gram-equivalents (Eq)
(iii) Normality (N ) It is the number of gram equivalents of Weight of compound w
Equivalent = =
solute in one litre of solution. Equivalent weight Equivalent weight
Gram equivalents of solute (Eq)
N = l
Mole Equivalent Relationship In a given weight
Volume of solution in litre (w) of sample, number of moles (n) and number of
(iv) Mole Fraction (c i ) It is the fraction of moles of a equivalents (eq) are related as
w w
particular component in a mixture as n= and Eq =
ni M Equivalent weight
ci = n
ni Q
Eq
=
M
= n-factor
i=1 n Equivalent weight

(v) ppm (parts per million) Strength It is defined as l


n-factor For salt, it is valency, for acid it is basicity, for
parts of solute present in (106 part) of solution. base it is acidity.
l
Dilution Formula If a concentrated solution is diluted,
l
Normality/Molarity Relationship
following formula work Eq n N Eq MW
N = and M = = = = n-factor
V V M n EW
M 1 V1 = M 2V2
l
Acid-Base Titration In acid-base titration, at the
(M 1 andV1 are the molarity and volumes before dilution and
End Point.
M 2 andV2 are molarity and volumes after dilution)
Gram equivalent of acid = Gram equivalent of base
l
Mixing of two or more solutions of different
molarities If two or more solutions of molarities l
Titration of a Mixture of NaOH/Na2CO3
( M 1 , M 2 , M 3 , ... ) are mixed together, molarity of the The mixture is analysed by titrating against a standard acid in
resulting solution can be worked out as : presence of phenolphthalein and methyl orange indicators.
M 1 V1 + M 2 V2 + M 3 V3 K Phenolphthalein end point occur when the following
M =
V1 + V2 + V3 K neutralisation is complete :
Some Basic Concepts of Chemistry 3

NaOH + HCl NaCl + H2 O Quick Balancing of a Redox Reaction


Na 2 CO3 + 2HCl NaHCO3 + NaCl Cross-multiplication by net change in oxidation number per
unit formula of oxidising agent and reducing agent will
1 millimol of (HCl) = 1 millimol of ( NaOH + Na 2 CO3 )
balance the redox reaction in term of OA and RA as:
Methyl orange end point occur when the following
neutralisation is complete : DON = 1
NaOH + HCl NaCl + H2 O K 2 Cr2 O7 + Fe2+ 2Cr 3+ + Fe3+
Na 2 CO3 + 2HCl 2NaCl + H2 O + CO2
methylorange end point millimol (HCl) DON = 12 - 6 = 6
= millimol (NaOH) + 2 millimol of (Na 2 CO3 ) Hence, multiplying Fe2+ by 6 and K 2 Cr2 O7 by 1 will balance
l
Titration of a mixture of NaHCO 3 / Na 2CO 3 the reaction in terms of OA and RA.
The mixture is analysed by titrating against a standard acid in Disproportionation Reaction It is a special type of
presence of phenolphthalein and methyl orange indicators. redox reaction in which similar species is oxidised as well
Phenolphthalein end point occur when the following reduced, e.g. Br2 + NaOH NaBr + NaBrO3 . In this
neutralisation is complete : reaction, bromine is reduced to bromide ion and the same is
oxidised to bromate ion, hence bromine is undergoing
Na 2 CO3 + HCl NaHCO3 + NaCl
disproportionation reaction.
millimol (HCl) = millimol (Na 2 CO3 )
Equivalent Weight of OA / RA
Methyl orange end point occur when the following
neutralisation is complete : Equivalent weight of OA/RA
Na 2 CO3 + 2HCl 2NaCl + H2 O + CO2 =
Molar mass
NaHCO3 + HCl NaCl + H2 O + CO2 Change in ON per formula unit
Methyl orange end point millimol (HCl)
M
= millimol (NaHCO3 ) + 2 millimol of (Na 2 CO3 ) e.g. KMnO4 + H+ Mn 2+ : E =
l
Percentage Strength of Oleum It is the mass of 5
H2 SO4 obtained on hydrolysis of 100 g of oleum as : M
K 2 Cr2 O7 + H+ 2Cr 3+ : E =
H2 S2 O7 + H2 O 2H2 SO4 6
The net reaction is :
2KI I2 + 2K + :
SO3 + H2 O H2 SO4
80 18 98 [ E = M ( DON per I- = 1)]
80 2Na 2 S2 O3 Na 2 S4 O6 + 2Na + :
% of free SO3 in oleum = (% Strength-100)
18 [E = M ( DON per Na 2 S2 O3 = 1)]
l
Redox Reaction and Redox Titration l
n-Factor and Normality/Molarity Relationship
(i) Oxidation Loss of electrons or increase in oxidation Eq
N =
number is called oxidation. V
(ii) Reduction Gain of electron or decrease in oxidation n
and M =
number is called reduction. V
+6 +2 +3 N Eq
=
K 2 Cr2 O7 + FeSO4 + H2 SO4 Fe2 (SO4 )3 M n
+3 MW
+ Cr2 (SO4 )3 = = n-factor
EW
In the above redox reaction, chromium is reduced from (n-factor = Change in oxidation number per formula unit).
(+ 6 to +3) and iron is oxidised from (+ 2 to + 3). Hence, l
Redox Titration At the end point:
K 2 Cr2 O7 is known as oxidising agent (itself reduced) Gram equivalents of OA = Gram equivalent of RA.
and FeSO4 reducing agent (itself oxidised).
4 Some Basic Concepts of Chemistry

Topic 1 Mole Concept


Objective Questions I (Only one correct option) 10. The normality of 0.3 M phosphorus acid (H3PO3) is
1. The molecular formula of a commercial resin used for (a) 0.1 (b) 0.9 (1999, 2M)
exchanging ions in water softening is C8 H7 SO3 Na (c) 0.3 (d) 0.6
(molecular weight = 206). What would be the maximum 11. In which mode of expression, the concentration of a solution
uptake of Ca 2+ ions by the resin when expressed in mole per remains independent of temperature? (1988, 1M)
gram resin? (2015 JEE Main) (a) Molarity (b) Normality (c) Formality (d) Molality
1 1 2 1
(a) (b) (c) (d) 12. A molal solution is one that contains one mole of solute in
103 206 309 412 (a) 1000 g of solvent (1986, 1M)
2. 3 g of activated charcoal was added to 50 mL of acetic acid (b) 1.0 L of solvent
solution (0.06 N) in a flask. After an hour it was filtered and (c) 1.0 L of solution
the strength of the filtrate was found to be 0.042 N. The (d) 22.4 L of solution
amount of acetic acid adsorbed (per gram of charcoal) is
13. If 0.50 mole of BaCl 2 is mixed with 0.20 mole of Na 3 PO4 ,
(a) 18 mg (b) 36 mg (2015 JEE Main)
the maximum number of moles of Ba 3 (PO4 )2 that can be
(c) 42 mg (d) 54 mg formed is (1981, 1M)
3. The ratio mass of oxygen and nitrogen of a particular gaseous (a) 0.70 (b) 0.50 (c) 0.20 (d) 0.10
mixture is 1 : 4. The ratio of number of their molecule is
14. 2.76 g of silver carbonate on being strongly heated yields a
(a) 1 : 4 (b) 7 : 32 (2014 Main)
residue weighing (1979, 1M)
(c) 1 : 8 (d) 3 : 16 (a) 2.16 g (b) 2.48 g (c) 2.32 g (d) 2.64 g
4. The molarity of a solution obtained by mixing 750 mL of 15. When the same amount of zinc is treated separately with
0.5 M HCl with 250 mL of 2 M HCl will be (2013 Main)
excess of sulphuric acid and excess of sodium hydroxide, the
(a) 0.875 M (b) 1.00 M ratio of volumes of hydrogen evolved is (1979, 1M)
(c) 1.75 M (d) 0.0975M (a) 1 : 1 (b) 1 : 2 (c) 2 : 1 (d) 9 : 4
5. Dissolving 120 g of urea (mol. wt. 60) in 1000 g of water 16. The largest number of molecules is in (1979, 1M)
gave a solution of density 1.15 g/mL. The molarity of the (a) 36 g of water
solution is (2011)
(b) 28 g of CO
(a) 1.78 M (b) 2.00 M
(c) 46 g of ethyl alcohol
(c) 2.05 M (d) 2.22 M
(d) 54 g of nitrogen pentaoxide (N2 O5 )
6. Given that the abundances of isotopes 54 Fe, 56 Fe and 57 Fe
17. The total number of electrons in one molecule of carbon
are 5%, 90% and 5%, respectively, the atomic mass of Fe is dioxide is (1979, 1M)
(a) 55.85 (b) 55.95 (c) 55.75 (d) 56.05 (2009) (a) 22 (b) 44 (c) 66 (d) 88
7. Mixture X = 0.02 mole of [Co(NH3 )5 SO4 ]Br and 0.02 mole 18. A gaseous mixture contains oxygen and nitrogen in the ratio
of [Co(NH3 )5 Br]SO4 was prepared in 2 L solution. of 1:4 by weight. Therefore, the ratio of their number of
1 L of mixture X + excess of AgNO3 solution Y molecules is (1979, 1M)

1 L of mixture X + excess of BaCl 2 solution Z (a) 1 : 4 (b) 1 : 8 (c) 7 : 32 (d) 3 : 16


Number of moles of Y and Z are (2003, 1M)
Fill in the Blanks
(a) 0.01, 0.01 (b) 0.02, 0.01
(c) 0.01, 0.02 (d) 0.02, 0.02 19. The weight of 1 1022 molecules of CuSO4 5H2 O is
8. Which has maximum number of atoms? (2003, 1M)
. . (1991, 1M)

(a) 24 g of C (12) (b) 56 g of Fe (56) 20. 3.0 g of a salt of molecular weight 30 is dissolved in 250 g
(c) 27 g of Al (27) (d) 108 g of Ag (108) water. The molarity of the solution is . (1983, 1M)

9. How many moles of electron weighs 1 kg? 21. The total number of electrons present in 18 mL of water is
1 . . (1980, 1M)
(a) 6.023 1023 (b) 1031 (2002, 3M)
9.108 22. The modern atomic mass unit is based on the mass of
6.023 1 . . (1980, 1M)
(c) 1054 (d) 108
9.108 9.108 6.023
Some Basic Concepts of Chemistry 5

Integer Answer Type Questions 34. n-butane is produced by monobromination of ethane


followed by Wurtzs reaction.Calculate volume of ethane at
23. A compound H2 X with molar weight of 80 g is dissolved in NTP required to produce 55 g n-butane, if the bromination
a solvent having density of 0.4 g mL-1 . Assuming no change takes place with 90% yield and the Wurtzs reaction with
in volume upon dissolution, the molality of a 3.2 molar 85% yield. (1989, 3M)
solution is (2014 Adv.)
35. A sugar syrup of weight 214.2 g contains 34.2 g of sugar
24. 29.2% (w/W ) HCl stock solution has density of 1.25g mL - 1 . (C12 H22 O11 ). Calculate (i) molal concentration and (ii) mole
The molecular weight of HCl is 36.5 g mol - 1 . The volume fraction of sugar in syrup. (1988, 2M)
(mL) of stock solution required to prepare a 200 mL solution
36. An unknown compound of carbon, hydrogen and oxygen
0.4 M HCl is (2012)
contains 69.77% C and 11.63% H and has a molecular
weight of 86. It does not reduces Fehlings solution but
Subjective Questions forms a bisulphate addition compound and gives a positive
25. 20% surface sites have adsorbed N2 . On heating N2 gas iodoform test. What is the possible structure(s) of unknown
evolved from sites and were collected at 0.001 atm and 298 K compound? (1987, 3M)
in a container of volume is 2.46 cm 3 . Density of surface sites 37. The density of a 3 M sodium thiosulphate solution
is 6.023 1014 /cm 2 and surface area is 1000 cm 2 , find out the ( Na 2 S2 O3 ) is 1.25 g per mL. Calculate (i) the percentage by
number of surface sites occupied per molecule of N2 . weight of sodium thiosulphate (ii) the mole fraction of
(2005, 3M) sodium thiosulphate and (iii) the molalities of Na + and
26. In a solution of 100 mL 0.5 M acetic acid, one gram of active S2 O2-
3 ions. (1983, 5M)

charcoal is added, which adsorbs acetic acid. It is found that 38. (a) 1.0 L of a mixture of CO and CO2 is taken. This mixture
the concentration of acetic acid becomes 0.49 M. If surface is passed through a tube containing red hot charcoal. The
area of charcoal is 3.01 102 m2 , calculate the area occupied volume now becomes 1.6 L. The volumes are measured
by single acetic acid molecule on surface of charcoal. (2003) under the same conditions. Find the composition of
mixture by volume.
27. Find the molarity of water. Given: r = 1000 kg/m3 (2003)
(b) A compound contains 28 per cent of nitrogen and
28. A plant virus is found to consist of uniform cylindrical 72 per cent of a metal by weight. 3 atoms of metal
particles of 150 in diameter and 5000 long. The specific combine with 2 atoms of nitrogen. Find the atomic
volume of the virus is 0.75 cm 3 /g. If the virus is considered to weight of metal. (1980, 5M)
be a single particle, find its molar mass. (1999, 3M) 39. 5.00 mL of a gas containing only carbon and hydrogen were
29. 8.0575 10-2 kg of Glaubers salt is dissolved in water to mixed with an excess of oxygen (30 mL) and the mixture
3 -3
obtain 1 dm of solution of density 1077.2 kg m . Calculate exploded by means of electric spark. After explosion, the
the molality, molarity and mole fraction of Na 2 SO4 in volume of the mixed gases remaining was 25 mL. On adding
solution. (1994, 3M) a concentrated solution of KOH, the volume further
diminished to 15 mL, the residual gas being pure oxygen. All
30. A is a binary compound of a univalent metal. 1.422 g of A volumes have been reduced to NTP. Calculate the molecular
reacts completely with 0.321 g of sulphur in an evacuated formula of the hydrocarbon gas. (1979, 3M)
and sealed tube to give 1.743 g of a white crystalline solid B,
that forms a hydrated double salt, C with Al 2 (SO4 )3 . Identify 40. In the analysis of 0.5 g sample of feldspar, a mixture of
A, B and C. (1994, 2M)
chlorides of sodium and potassium is obtained, which weighs
0.1180 g. Subsequent treatment of the mixed chlorides with
31. Upon mixing 45.0 mL 0.25 M lead nitrate solution with silver nitrate gives 0.2451 g of silver chloride. What is the
25.0 mL of a 0.10 M chromic sulphate solution, precipitation percentage of sodium oxide and potassium oxide in the
of lead sulphate takes place. How many moles of lead sample? (1979, 5M)
sulphate are formed? Also calculate the molar concentrations
of species left behind in the final solution. Assume that lead 41. The vapour density (hydrogen = 1) of a mixture consisting of
sulphate is completely insoluble. (1993, 3M) NO2 and N2 O4 is 38.3 at 26.7C. Calculate the number of
moles of NO2 in 100 g of the mixture. (1979, 5M)
32. Calculate the molality of 1.0 L solution of 93% H2 SO4 ,
(weight/volume). The density of the solution is 1.84 g/mL. 42. Accounts for the following. Limit your answer to two
(1990, 1M) sentences, Atomic weights of most of the elements are
33. A solid mixture (5.0 g) consisting of lead nitrate and sodium fractional. (1979, 1M)
nitrate was heated below 600C until the weight of the 43. Naturally occurring boron consists of two isotopes whose
residue was constant. If the loss in weight is 28.0 per cent, atomic weights are 10.01 and 11.01. The atomic weight of
find the amount of lead nitrate and sodium nitrate in the natural boron is 10.81. Calculate the percentage of each
mixture. (1990, 4M) isotope in natural boron. (1978, 2M)
6 Some Basic Concepts of Chemistry

Topic 2 Equivalent Concept, Neutralisation and Redox Titration


Objective Questions I (Only one correct option) 10. For the redox reaction
1. From the following statements regarding H2 O2 choose the MnO4- + C2 O24 - + H+ Mn 2+ + CO2 + H2 O
incorrect statement. (2015 Main) The correct coefficients of the reactants for the balanced
(a) It can act only as an oxidising agent reaction are
(b) It decomposed on exposure to light MnO-4 C2 O42 - H+ (1992)
(c) It has to be stored in plastic or wax lined glass bottles in (a) 2 5 16
dark (b) 16 5 2
(d) It has to be kept away from dust (c) 5 16 2
2. Consider a titration of potassium dichromate solution with (d) 2 16 5
acidified Mohrs salt solution using diphenylamine as 11. The volume strength of 1.5 N H2 O2 is (1990, 1M)
indicator. The number of moles of Mohr's salt required per
(a) 4.8 (b) 8.4 (c) 3.0 (d) 8.0
mole of dichromate is (2007, 3M)
(a) 3 (b) 4 (c) 5 (d) 6 12. The oxidation number of phosphorus in Ba(H2 PO2 )2 is
3. In the standardisation of Na 2 S2 O3 using K 2 Cr2 O7 by (a) +3 (b) +2 (1988)
iodometry, the equivalent weight of K 2 Cr2 O7 is (2001, 1M) (c) +1 (d) 1
(a) (molecular weight)/2 13. The equivalent weight of MnSO 4 is half of its molecular
(b) (molecular weight)/6 weight, when it converts to (1988, 1M)
(c) (molecular weight)/3 (a) Mn 2 O3 (b) MnO2 (c) MnO-4 (d) MnO2-
4
(d) same as molecular weight
4. The reaction, 3ClO - (aq) ClO3 (aq) + 2Cl - (aq) is an Objective Question II (More than one correct option)
example of (2001) 14. For the reaction, I- + ClO-3 + H2 SO4 Cl - + HSO-4 + I2
(a) oxidation reaction the correct statement(s) in the balanced equation is/are
(b) reduction reaction (a) stoichiometric coefficient of HSO-4 is 6 (2014 Adv)
(c) disproportionation reaction (b) iodide is oxidised
(d) decomposition reaction (c) sulphur is reduced
5. An aqueous solution of 6.3 g oxalic acid dihydrate is made up (d) H2 O is one of the products
to 250 mL. The volume of 0.1 N NaOH required to
completely neutralise 10 mL of this solution is (2001, 1M) Assertion and Reason
(a) 40 mL (b) 20 mL (c) 10 mL (d) 4 mL Read the following questions and answer as per the direction
6. Among the following, the species in which the oxidation given below :
number of an element is + 6 (2000) (a) Statement I is true; Statement II is true; Statement II is the
(a) MnO-4 (b) Cr(CN)3- correct explanation of Statement I.
6
(b) Statement I is true; Statement II is true; Statement II is not
(c) NiF62- (d) CrO2 Cl 2 the correct explanation of Statement I.
7. The oxidation number of sulphur in S 8 , S 2 F 2 , H 2 S (c) Statement I is true; Statement II is false.
respectively, are (1999) (d) Statement I is false; Statement II is true.
(a) 0, +1 and 2 (b) +2, +1 and 2 15. Statement I In the titration of Na 2 CO3 with HCl using
(c) 0, +1 and +2 (d) 2, +1 and 2 methyl orange indicator, the volume required at the
8. The number of moles of KMnO 4 that will be needed to react equivalence point is twice that of the acid required using
completely with one mole of ferrous oxalate in acidic phenolphthalein indicator.
medium is (1997) Statement II Two moles of HCl are required for the
2 3 4 complete neutralisation of one mole of Na 2 CO3 . (1991, 2M)
(a) (b) (c) (d) 1
5 5 5
9. The number of moles of KMnO 4 that will be needed to react Fill in the Blanks
with one mole of sulphite ion in acidic solution is (1997) 16. The compound YBa 2 Cu 3 O7 , which shows super
2 3 4 conductivity, has copper in oxidation state . Assume
(a) (b) (c) (d) 1
5 5 5 that the rare earth element yttrium is in its usual + 3 oxidation
state. (1994, 1M)
Some Basic Concepts of Chemistry 7

Integer Answer Type Questions 27. A 5.0 cm 3 solution of H2 O2 liberates 0.508 g of iodine from
an acidified KI solution. Calculate the strength of H2 O2
17. The difference in the oxidation numbers of the two types of solution in terms of volume strength at STP. (1995, 3M)
sulphur atoms in Na 2 S4 O6 is (2011)
28. One gram of commercial AgNO3 is dissolved in 50 mL of
18. Among the following, the number of elements showing only water. It is treated with 50 mL of a KI solution. The silver
one non-zero oxidation state is O, Cl, F, N, P, Sn, Tl, Na, Ti iodide thus precipitated is filtered off. Excess of KI in the
(2010)
filtrate is titrated with (M/10) KIO 3 solution in presence of
19. A student performs a titration with different burettes and 6 M HCl till all I- ions are converted into ICl. It requires
finds titrate values of 25.2 mL, 25.25 mL, and 25.0 mL. The 50 mL of (M/10) KIO 3 solution, 20 mL of the same stock
number of significant figures in the average titrate value is solution of KI requires 30 mL of (M/10) KIO3 under similar
(2010) conditions. Calculate the percentage of AgNO3 in the
sample.
Subjective Questions Reaction KIO3 + 2KI + 6HCl 3ICl + 3KCl + 3H2 O
20. Calculate the amount of calcium oxide required when it (1992, 4M)
reacts with 852 g of P4 O10 . (2005, 2M) 29. A 2.0 g sample of a mixture containing sodium carbonate,
21. Hydrogen peroxide solution (20 mL) reacts quantitatively sodium bicarbonate and sodium sulphate is gently heated till
with a solution of KMnO4 (20 mL) acidified with dilute the evolution of CO2 ceases. The volume of CO2 at 750 mm
H2 SO4 . The same volume of the KMnO4 solution is just Hg pressure and at 298 K is measured to be 123.9 mL. A 1.5 g
decolourised by 10 mL of MnSO4 in neutral medium of the same sample requires 150 mL of (M/10) HCl for
simultaneously forming a dark brown precipitate of hydrated complete neutralisation. Calculate the percentage
MnO2 . The brown precipitate is dissolved in 10 mL of 0.2 M composition of the components of the mixture. (1992, 5M)
sodium oxalate under boiling condition in the presence of 30. A 1.0 g sample of Fe2 O3 solid of 55.2% purity is dissolved in
dilute H2 SO4 . Write the balanced equations involved in the acid and reduced by heating the solution with zinc dust. The
reactions and calculate the molarity of H2 O2 . (2001) resultant solution is cooled and made up to 100.0 mL. An
22. How many millilitres of 0.5 M H2 SO4 are needed to dissolve aliquot of 25.0 mL of this solution requires for titration.
Calculate the number of electrons taken up by the oxidant in
0.5 g of copper (II) carbonate? (1999, 3M)
the reaction of the above titration. (1991, 4M)
23. An aqueous solution containing 0.10 g KIO3 (formula weight = 31. A solution of 0.2 g of a compound containing Cu 2+ and
214.0) was treated with an excess of KI solution. The solution
C2 O2-
4 ions on titration with 0.02 M KMnO4 in presence of
was acidified with HCl. The liberated I2 consumed 45.0 mL
H2 SO4 consumes 22.6 mL of the oxidant. The resultant
of thiosulphate solution decolourise the blue starch-iodine
solution is neutralised with Na 2 CO3 , acidified with dilute
complex. Calculate the molarity of the sodium thiosulphate
acetic acid and treated with excess KI. The liberated iodine
solution. (1998, 5M)
requires 11.3 mL of 0.05 M Na 2 S2 O3 solution for complete
24. To a 25 mL H2 O2 solution, excess of acidified solution of reduction. Find out the mole ratio of Cu 2+ to C2 O2- 4 in the
potassium iodide was added. The iodine liberated required compound. Write down the balanced redox reactions
20 mL of 0.3 N sodium thiosulphate solution. Calculate the involved in the above titrations. (1991, 5M)
volume strength of H2 O2 solution. (1997, 5M) 32. A mixture of H2 C2 O4 (oxalic acid) and NaHC2 O4 weighing
25. A 3.00 g sample containing Fe3 O4 , Fe2 O3 and an inert 2.02 g was dissolved in water and the solution made up to one
impure substance, is treated with excess of KI solution in litre. Ten millilitres of the solution required 3.0 mL of 0.1 N
presence of dilute H2 SO4 . The entire iron is converted into sodium hydroxide solution for complete neutralisation. In
Fe2+ along with the liberation of iodine. The resulting another experiment, 10.0 mL of the same solution, in hot
solution is diluted to 100 mL . A 20 mL of the diluted dilute sulphuric acid medium, required 4.0 mL of 0.1 N
solution requires 11.0 mL of 0.5 M Na 2 S2 O3 solution to potassium permanganate solution for complete reaction.
reduce the iodine present. A 50 mL of the dilute solution, Calculate the amount of H2 C2 O4 and NaHC2 O4 in the
after complete extraction of the iodine required 12.80 mL of mixture. (1990, 5M)
0.25 M KMnO4 solution in dilute H2 SO4 medium for the 33. An organic compound X on analysis gives 24.24 per cent
oxidation of Fe2+ . Calculate the percentage of Fe2 O3 and carbon and 4.04 per cent hydrogen. Further, sodium extract
Fe3 O4 in the original sample. (1996, 5M) of 1.0 g of X gives 2.90 g of silver chloride with acidified
26. A 20.0 cm 3 mixture of CO, CH4 and He gases is exploded by silver nitrate solution. The compound X may be represented
an electric discharge at room temperature with excess of by two isomeric structures Y and Z. Y on treatment with
oxygen. The volume contraction is found to be 13.0 cm 3 . A aqueous potassium hydroxide solution gives a dihydroxy
further contraction of 14.0 cm 3 occurs when the residual gas compound while Z on similar treatment gives ethanal. Find
is treated with KOH solution. Find out the composition of the out the molecular formula of X and gives the structure of Y
gaseous mixture in terms of volume percentage. (1995, 4M) and Z. (1989, 5M)
8 Some Basic Concepts of Chemistry

34. An equal volume of a reducing agent is titrated separately with 36. 5 mL of 8 N nitric acid, 4.8 mL of 5 N hydrochloric acid
1 M KMnO4 in acid, neutral and alkaline medium. The volumes and a certain volume of 17 M sulphuric acid are mixed
of KMnO4 required are 20 mL in acid, 33.3 mL in neutral and 100 together and made up to 2 L. 30 mL of this acid mixture
mL in alkaline media. Find out the oxidation state of manganese exactly neutralise 42.9 mL of sodium carbonate solution
in each reduction product. Give the balanced equations for all the containing one gram of Na 2 CO3 10H2 O in 100 mL of
three half reaction. Find out the volume of 1M K 2 Cr2 O7 water. Calculate the amount in gram of the sulphate ions
consumed, if the same volume of the reducing agent is titrated in in solution. (1985, 4M)
acid medium. (1989, 5M) 37. 2.68 10-3 moles of a solution containing an ion A n+
35. A sample of hydrazine sulphate ( N2 H 6 SO4 ) was dissolved in 100 require 1.61 10-3 moles of MnO-4 for the oxidation of
mL of water, 10 mL of this solution was reacted with excess of
A n+ to A O-3 in acidic medium. What is the value of n ?
ferric chloride solution and warmed to complete the reaction.
(1984, 2M)
Ferrous ion formed was estimated and it, required 20 mL of M/50
potassium permanganate solution. Estimate the amount of 38. 4.08 g of a mixture of BaO and unknown carbonate
hydrazine sulphate in one litre of the solution. MCO3 was heated strongly. The residue weighed 3.64 g.
Reaction 4Fe3+ + N2 H4 N2 + 4Fe2+ + 4H+ This was dissolved in 100 mL of 1 N HCl. The excess
acid required 16 mL of 2.5 N NaOH solution for
MnO-4 + 5Fe2+ + 8H+ Mn 2+ + 5Fe3+ + 4H2 O complete neutralisation. Identify the metal M.
(1988, 3M) (1983, 4M)

Topic 1 Topic 2
1. (d) 2. (d) 3. (b) 4. (a) 1. (a) 2. (d) 3. (b) 4. (c)
5. (c) 6. (b) 7. (a) 8. (a) 5. (a) 6. (d) 7. (a) 8. (b)
9. (a) 10. (a) 11. (b) 12. (c)
9. (d) 10. (d) 11. (d) 12. (a)
13. (b) 14. (a, b, d) 15. (b) 16. 7/3
13. (d) 14. (a) 15. (a) 16. (a)
17. (5) 18. (2) 19. (3)
17. (a) 18. (c) 19. (4.14 g) 20. (0.4)
21. (6.02310 24 ) 22. C-12 isotope 23. (8)
24. (8)

Topic 1 Mole Concept 2. Given, initial strength of acetic acid = 0.06 N


1. We know the molecular weight of C8 H7 SO3Na Final strength = 0.042 N; Volume given = 50 mL
= 12 8 + 1 7 + 32 + 16 3 + 23 = 206 \Initial millimoles of CH3COOH = 0.06 50 = 3
we have to find, mole per gram of resin. Final millimoles of CH3COOH = 0.042 50 = 2.1
\ 1g of C8 H7 SO3Na has number of mole \ Millimoles of CH3COOH adsorbed = 3 - 2.1= 0.9 mmol
weight of given resin 1 = 0.9 60 mg = 54 mg
= = mol (mO 2 )
Molecular, weight of resin 206
nO 2 (M O 2 )
Now, reaction looks like 3. =
nN 2 (mN 2 )
2C8 H7 SO3Na + Ca 2+ (C8 H7 SO3 )2 Ca + 2Na (M N 2 )
Q 2 moles of C8 H7 SO3Na combines with 1 mol Ca 2+ where, mO 2 = given mass of O2 , mN 2 = given mass of N2 ,
1 M O 2 = molecular mass of O2 , M N 2 = molecular mass of N2 ,
\1 mole of C8 H7 SO3Na will combine with mol Ca 2+ nO 2 = number of moles of O2 , nN 2 = number of moles of
2
N2
1
\ mole of C8 H7 SO3 Na will combine with mO 28 1 28 7
206 = 2 = =
1 1 1 mN 2 32 4 32 32
mol Ca 2+ = mol Ca 2+
2 206 412
Some Basic Concepts of Chemistry 9

M 1V1 + M 2V2 10. Phosphorus acid is a dibasic acid as :


4. From the formula, M f =
V1 + V2 O
Given, V1 = 750 mL, M 1 = 0.5 M
HP OH only two replaceable hydrogens
V2 = 250 mL, M 2 = 2 M
750 0.5 + 250 2 875 OH
= = = 0.875 M
750 + 250 1000 Therefore, normality = molarity basicity = 0.3 2 = 0.60
Moles of solute
5. Molarity = 11. Molality is defined in terms of weight, hence independent of
Volume of solution (L) temperature. Remaining three concentration units are defined in
120 terms of volume of solution, they depends on temperature.
Moles of urea = =2
60 12. Molality of a solution is defined as number of moles of solute
Weight of solution = Weight of solvent + Weight of solute present in 1.0 kg (1000 g) of solvent.
= 1000 + 120 = 1120 g 13. The balanced chemical reaction is
1120 g 1 3BaCl 2 + 2Na 3PO4 Ba 3 (PO4 )2 + 6NaCl
Volume = = 0.973 L
1.15 g / mL 1000 mL / L In this reaction, 3 moles of BaCl 2 combines with 2 moles of Na 3PO4.
2.000 Hence, 0.5 mole of of BaCl 2 require
Molarity = = 2.05M
0.973 2
0.5 = 0.33 mole of Na 3PO4.
6. From the given relative abundance, the average weight of Fe 3
can be calculated as Since, available Na 3PO4 (0.2 mole) is less than required mole (0.33),
54 5 + 56 90 + 57 5 it is the limiting reactant and would determine the amount of product
A= = 55.95 Ba 3 (PO4 )2.
100
Q 2 moles of Na 3PO4 gives 1 mole Ba 3 (PO4 )2
7. 1.0 L of mixture X contain 0.01 mole of each 1
[(Co(NH3 )5 SO4 )]Br and [Co(NH3 )5 Br)]SO4. Also, with \ 0.2 mole of Na 3PO4 would give 0.2
2
AgNO3, only [(Co(NH3 )5 SO4 )]Br reacts to give AgBr
precipitate as = 0.1 mole Ba 3 (PO4 )2
[Co(NH3 )5 SO4 ]Br + AgNO3 [Co(NH3 )5 SO4 ]NO3 14. Unlike other metal carbonates that usually decomposes into metal
1.0 mol Excess oxides liberating carbon dioxide, silver carbonate on heating
+ AgBr
1.0 mol decomposes into elemental silver liberating mixture of carbon
With BaCl 2, only [Co(NH3 )5 Br)]SO4 reacts giving BaSO4 dioxide and oxygen gas as :
precipitate as Heat 1
Ag2CO3 (s) 2Ag (s) + CO2 (g ) + O (g )
2 2
[Co(NH3 )5 Br]SO4 + BaCl 2 [Co(NH3 )5 Br]Cl 2 + BaSO4
1.0 mol Excess 1 mol MW = 276 g 2 108 = 216 g
Hence, moles of Y and Z are 0.01 each. Hence, 2.76 g of Ag2CO3 on heating will give
216
8. Number of atoms = Number of moles 2.76 = 2.16g Ag as residue.
276

Avogadros number (N A) 15. The balanced chemical reaction of zinc with sulphuric acid and
24 NaOH are
Number of atoms in 24 g C = NA = 2NA Zn + H2SO4 ZnSO4 + H2 (g )
12
56 Zn + 2NaOH + 2H2O Na 2[ Zn(OH)4 ] + H2 (g )
Number of atoms in 56 g of Fe = NA = NA
56 Since, one mole of H2 (g ) is produced per mole of zinc with both
27 sulphuric acid and NaOH respectively, hydrogen gas is produced in
Number of atoms in 27 g of Al = NA = NA the molar ratio of 1:1 in the above reactions.
27
108 36
Number of atoms in 108 g of Ag = NA = NA 16. Number of molecules present in 36 g of water = N A = 2N A
108 18
Hence, 24 g of carbon has the maximum number of atoms. 28
Number of molecules present in 28 g of CO = NA = NA
28
9. Mass of an electron = 9.108 10-31 kg 46
Number of molecules present in 46 g of C2H5OH = NA = NA
Q 9.108 10-31 kg = 1.0 electron 46
1 1031 1 54
\ 1 kg = electrons = Number of molecules present in 54 g of N2O5 = N A = 0.5 N A
9.108 10 -31
9.108 6.023 1023 108
Here, NA is Avogadros number. Hence, 36 g of water contain the
1
= 108 mole of electrons largest (2NA ) number of molecules.
9.108 6.023
10 Some Basic Concepts of Chemistry

17. In a neutral atom, atomic number represents the number of 24. Mass of HCl in 1.0 mL stock solution
protons inside the nucleus and equal number of electrons around 29.2
it. Therefore, the number of total electrons in molecule of CO2 = 1.25 = 0.365 g
100
= electrons present in one carbon atom
Mass of HCl required for 200 mL 0.4 M HCl
+ 2 electrons present in one oxygen atom = 6 + 2 8 = 22. 200
Weight of a compound in gram (w) = 0.4 36.5 = 0.08 36.5 g
18. = Number of moles (n) 1000
Molar mass (M ) \ 0.365 g of HCl is present in 1.0 mL stock solution.
Number of molecules (N ) 0.08 36.5
= 0.08 36.5 g HCl will be present in = 8.0 mL
Avogadro number (NA ) 0.365
w (O2 ) N (O2 ) 25. Partial pressure of N2 = 0.001 atm,
= (i)
32 NA
T = 298 K, V = 2.46 dm 3.
w (N2 ) N (N2 )
And = (ii) From ideal gas law : pV = nRT
28 NA pV 0.001 2.46
n(N2 ) = = = 10-7
Dividing Eq. (i) by Eq. (ii) gives RT 0.082 298
N (O2 ) w (O2 ) 28 1 28 7
= = = Number of molecules of N2 = 6.023 1023 10-7
N (N2 ) w (N2 ) 32 4 32 32
= 6.023 1016
19. Molar mass of CuSO4 5H2O Now, total surface sites available
= 63.5 + 32 + 4 16 + 5 18 = 249.5 g = 6.023 1014 1000 = 6.023 1017
Also, molar mass represents mass of Avogadro number of 20
molecules in gram unit, therefore Surface sites used in adsorption = 6.023 1017
100
Q 6.023 1023 molecules of CuSO4 5H2O weigh 249.5 g
= 2 6.023 1016
249.5
\ 1022 molecules will weigh 1022 = 4.14 g Sites occupied per molecules
6.023 1023
Number of sites 2 6.023 1016
Number of moles of solute = = = 2
20. Molarity = Number of molecules 6.023 1016
Volume of solution in litre
Weight of solute 1000 26. Initial millimol of CH3COOH = 100 0.5 = 50
=
Molar mass Volume in mL millimol of CH3COOH remaining after adsorption
3 1000 = 100 0.49 = 49
= = 0.4 M
30 250
millimol of CH3COOH adsorbed = 50 49 = 1
21. Considering density of water to be 1.0 g/mL, 18 mL of water is number of molecules of CH3COOH adsorbed
18 g (1.0 mol) of water and it contain Avogadro number of 1
molecules. Also one molecule of water contain = 6.023 1023 = 6.023 1020
1000
2 (one from each H-atom) + 8 (from oxygen atom)
3.01 102
= 10 electrons. Area covered up by one molecule =
6.02 1020
1.0 mole of H2O contain = 10 6.023 1023
= 5 10-19 m 2
= 6.023 1024 electrons.
27. Mass of 1.0 L water = 1000 g
22. Carbon-12 isotope. According to modern atomic mass unit, one 1000
atomic mass unit (amu) is defined as one-twelfth of mass of an Molarity = = 55.56 mol L-1
18
atom of C-12 isotope, i.e.
1 28. Volume of one cylinderical plant virus = pr2l
1 amu (u) = weight of an atom of C-12 isotope.
12 = 3.14 (75 10-8 )2 5000 10-8 cm 3 = 8.83 10-17 cm 3
23. PLAN This problem can be solved by using concept of conversion of Volume of a virus
molarity into molality. Mass of one virus =
Specific volume
Molarity = 3.2 M
Let volume of solution = 1000 mL = Volume of solvent 8.83 10-17 cm 3
= = 1.1773 10-16 g
Mass of solvent = 1000 0.4 = 400 g 0.75 cm 3 g-1
Since, molarity of solution is 3.2 molar Molar mass of virus
\ n solute = 3.2 mol = Mass of one virus Avogadros number
3.2 = 1.1773 10-16 6.023 1023 g
Molality (m) = =8
400 / 1000
= 70.91 106 g
Hence, correct integer is (8).
Some Basic Concepts of Chemistry 11

29. Molar mass of Glaubers salt (Na 2SO4 10H2O) Also, millimol of Cr(NO3 )2 formed
= 23 2 + 32 + 64 + 10 18 = 322g = 2 millimol of Cr2 (SO4 )3 reacted
5
Mole of Na 2SO4 10H2O in 1.0 L solution =
80.575
= 0.25 Molarity of Cr(NO3 )2 = = 0.071 M
322 70
Molarity of solution = 0.25 M 32. 93% H2SO4 solution weight by volume indicates that there is
Also, weight of 1.0 L solution = 1077.2 g 93 g H2SO4 in 100 mL of solution.
weight of Na 2SO4 in 1.0 L solution = 0.25 142 = 35.5 g If we consider 100 mL solution, weight of solution = 184 g
Weight of water in 1.0 L solution = 1077.2 35.5 = 1041.7 g Weight of H2O in 100 mL solution = 184 93 = 91 g
0.25 Moles of solute
Molality = 1000 = 0.24 m Molality = 1000
1041.7 Weight of solvent (g)
Mole of Na 2SO4
Mole fraction of Na 2SO4 = =
93 1000
= 10.42
Mole of Na 2SO4 + Mole of water 98 91
0.25
= = 4.3 10-3. 33. Heating below 600C converts Pb(NO3 )2 into PbO but to
1041.7
0.25 + NaNO3 into NaNO2 as
18
D 1
30. Compound B forms hydrated crystals with Al 2 (SO4 )3. Also, B is Pb(NO3 )2 PbO(s) + 2NO2 + O
2 2
formed with univalent metal on heating with sulphur. Hence, MW : 330 222
D 1
compound B must has the molecular formula M 2SO4 and NaNO3 NaNO2 (s) + O
compound A must be an oxide of M which reacts with sulphur to 2 2
MW : 85 69
give metal sulphate as 28
A + S M 2SO4 Weight loss = 5 = 1.4 g
100
B
Q 0.321 g sulphur gives 1.743 g of M 2SO4 Weight of residue left = 5 1.4 = 3.6 g
\ 32.1 g S (one mole) will give 174.3 g M 2SO4 Now, let the original mixture contain x g of Pb(NO3 )2.
Therefore, molar mass of M 2SO4 = 174.3 g Q 330 g Pb(NO3 )2 gives 222 g PbO
174.3 = 2 Atomic weight of M + 32.1 + 64 222 x
\ x g Pb(NO3 )2 will give g PbO
Atomic weight of M = 39, metal is potassium (K) 330
K2SO4 on treatment with aqueous Al 2 (SO4 )3 gives potash-alum. Similarly, 85 g NaNO3 gives 69 g NaNO2
K2SO4 + Al 2 (SO4)3 + 24H2O K2SO4Al 2 (SO4)3 24H2O 69 (5 - x )
(5 x) g NaNO3 will give g NaNO2
B C 85
If the metal oxide A has molecular formula MOx, two moles of it 222 x 69 (5 - x )
combine with one mole of sulphur to give one mole of metal Residue : + = 3.6 g
330 85
sulphate as
Solving for x gives, x = 3.3 g Pb(NO3 )2
2KOx + S K2SO4
NaNO3 =1.7 g.
x = 2, i.e. A is KO2.
34. Reactions involved are
31. The reaction involved is
C2H6 + Br2 C2H5Br + HBr
3Pb(NO3 )2 + Cr2 (SO4 )3 3PbSO4 (s) + 2Cr(NO3 )3
2C2H5Br + 2Na C4H10 + 2NaBr
millimol of Pb(NO3 )2 taken = 45 0.25 = 11.25 Actual yield of C4H10 = 55 g which is 85% of theoretical yield.
millimol of Cr2 (SO4 )3 taken = 2.5 55 100
Here, chromic sulphate is the limiting reagent, it will determine Theoretical yield of C4H10 = = 64.70 g
85
the amount of product.
Also, 2 moles (218 g) C2H5Br gives 58 g of butane.
Q 1 mole Cr2 (SO4 )3 produces 3 moles PbSO4.
64.70 g of butane would be obtained from
\ 2.5 millimol Cr2 (SO4 )3 will produce 7.5 millimol PbSO4. 2
Hence, mole of PbSO4 precipitate formed = 7.5 10-3 64.70 = 2.23 moles C2H5Br
58
Also, millimol of Pb(NO3 )2 remaining unreacted Also yield of bromination reaction is only 90%, in order to have
11.25 7.50 = 3.75 2.23 moles of C2H5Br, theoretically
Molarity of Pb(NO3 )2 in final solution 2.23 100
= 2.48 moles of C2H5Br required.
millimol of Pb(NO3 )2 3.75 90
= = = 0.054 M
Total volume 70 Therefore, moles of C2H6 required = 2.48
Volume of C2H6 (NTP) required = 2.48 22.4 = 55.55 L.
12 Some Basic Concepts of Chemistry

34.2 38. (a) After passing through red-hot charcoal, following reaction
35. Moles of sugar = = 0.1
342 occurs
Moles of water in syrup = 214.2 34.2 C(s) + CO2 (g ) 2CO(g )
= 180 g If the 1.0 L original mixture contain x litre of CO2, after
Moles of solute passing from tube containing red-hot charcoal, the new
Therefore, (i) Molality = 1000 volumes would be :
Weight of Solvent (g)
0.1 2x (volume of CO obtained from CO2) + 1
= 1000 = 0.55 x(original CO) = 1 + x =1.6 (given)
180
Mole of sugar x = 0.6
(ii) Mole fraction of sugar = Hence, original 1.0 L mixture has 0.4 L CO and 0.6 L of CO2 ,
Mole of sugar + Mole of water
0.1 i.e. 40% CO and 60% CO2 by volume.
= = 9.9 10-3 (b) According to the given information, molecular formula of
0.1 + 10
the compound is M 3N2. Also, 1.0 mole of compound has 28 g
36. From the given elemental composition, empirical formula can of nitrogen. If X is the molar mass of compound, then :
be derived as : 28
X = 28
O 100
Element C H
X = 100 = 3 Atomic weight of M + 28
Weight % 69.77 11.63 18.60
72
Mole % 5.81 11.63 1.1625 (obtained by
Atomic weight of M = = 24
3
dividing from M )
39. In the present case, V n (Qall the volumes are measured under
Simple ratio 5 10 1
identical conditions of temperature and pressure) Hence, the
Hence, empirical formula is C5H10O and empirical formula reaction stoichiometry can be solved using volumes as :
weight is 86.
y y
CxH y (g ) + x + O2 (g ) x CO2 (g ) + H2O (l )
Since, empirical formula weight and molecular weight both are 4 2
(86), empirical formula is the molecular formula also.
volume of CO2 (g ) + O2 (g ) (remaining unreacted) = 25
Also, the compound does not reduce Fehlings solution, Volume of CO2 (g ) produced
therefore it is not an aldehyde, but it forms bisulphite, it must be = 10 mL (15 mL O2 remaining)
a ketone.
Q 1 mL CxH y produces x mL of CO2
Also, it gives positive iodoform test, it must be a methyl ketone. \ 5 mL CxH y will produce 5 xmL of CO2= 10 mL
O x=2
y
C3H7 C CH3 Also, 1 mL CxH y combines with x + mL of O2
4
Based on the above information, the compound may be one of
y
the following : 5 mL CxH y will combine with 5 x + mL of O2
O CH3 O 4
y
CH3CH2CH2 C CH3 or CH3 CH C CH3 5 x + = 15 (15 mL of O2 out of 30 mL)
2-pentanone 3-methyl -2-butanone
4
(remaining unreacted)
37. (a) Let us consider 1.0 L solution for all the calculation. y = 4, hence hydrocarbon is C2H4.
(i) Weight of 1 L solution = 1250 g
40. Oxides of sodium and potassium are converted into chlorides
Weight of Na 2S2O3 = 3 158 = 474 g according to following reactions :
474 Na 2O + 2HCl 2NaCl + H2O
Weight percentage of Na 2S2O3= 100 = 37.92
1250 K2O + 2HCl 2KCl + H2O
(ii) Weight of H2O in 1 L solution = 1250 - 474 = 776 g
Finally all the chlorides of NaCl and KCl are converted into
3
Mole fraction of Na 2S2O3 = = 0.065 AgCl, hence
776
3+ moles of (NaCl + KCl) = moles of AgCl
18
3 2 (one mole of either NaCl or KCl gives one mole of AgCl)
+
(iii) Molality of Na = 100 = 7.73 m
776
Some Basic Concepts of Chemistry 13

Now, let the chloride mixture contain x g NaCl. 2. n-factor of dichromate is 6.


x 0.118 - x 0.2451
+ = Also, n-factor of Mohrs salt is 1 as :
58.5 74.5 143.5 O. A
FeSO4 (NH4 )2 SO4 6H2O Fe3+
Solving for x gives x = 0.0338 g (mass of NaCl)
Mohrs salt
Mass of KCl = 0.118 0.0338 = 0.0842 g
1 Q 1 mole of dichromate = 6 equivalent of dichromate.
Also, moles of Na 2O = moles of NaCl
2 \ 6 equivalent of Mohrs salt would be required.
1 0.0338 Since, n-factor of Mohrs salt is 1, 6 equivalent of it would also
Mass of Na 2O = 62 = 0.0179 g be equal to 6 moles.
2 58.5
1 0.0842 Hence, 1 mole of dichromate will oxidise 6 moles of Mohrs salt.
Similarly, mass of K2O = 94 = 0.053 g
2 74.5 3. The following reaction occur between S2O2- 2-
3 and Cr2O 7 :
0.0179
Mass % of Na 2O = 100 = 3.58 % 26H+ + 3S2O32- + 4Cr2O27- 6SO24- + 8Cr 3+ + 13H2O
0.5
0.053 Change in oxidation number of Cr2O72- per formula unit is 6 (it is
Mass % of K2O = 100 = 10.6 %
0.5 always fixed for Cr2O72- ).
41. From the vapour density information Molecular weight
Hence, equivalent weight of K2Cr2O7 =
Molar mass = Vapour density 2 6
(Q Molar mass of H2 = 2) 4. It is an example of disproportionation reaction because the same
= 38.3 2 = 76.6 species (ClO- ) is being oxidised to ClO-3 as well as reduced to Cl - .
Now, let us consider 1.0 mole of mixture and it contains 5. Oxalic acid dihydrate H2C2O4 2H2O : mw = 126
x mole of NO2.
It is a dibasic acid, hence equivalent weight = 63
46 x + 92 (1 - x ) = 76.6 6.3 1000
Normality = = 0.4 N
x = 0.3348 63 250
100 N 1V1 = N 2V2
Also, in 100 g mixture, number of moles =
76.6 0.1 V1 = 0.4 10
100
Moles of NO2 in mixture = 0.3348 = 0.437 Hence, V1 = 40 mL
76.6
6. In MnO-4 , oxidation state of Mn is +7
42. Most of the elements found in nature exist as a mixture of
isotopes whose atomic weights are different. The atomic weight In Cr(CN)63- , oxidation state of Cr is +3
of an element is the average of atomic weights of all its naturally In NiF62- , Ni is in + 4 oxidation state.
occurring isotopes.
In CrO2Cl 2, oxidation state of Cr is +6.
43. Average atomic weight
S Percentage of an isotope Atomic weight 7. In S8 , oxidation number of S is 0, elemental state.
=
100 In S2F2, F is in 1 oxidation state, hence S is in + 1 oxidation
10.01x + 11.01 (100 - x) state.
10.81 = In H2S, H is in +1 oxidation state, hence S is in 2 oxidation
100
state.
x = 20%
Therefore, natural boron contains 20% (10.01) isotope and 80% 8. The balanced redox reaction is :
other isotope. 3MnO-4 + 5FeC2O4 + 24H+ 3Mn 2+ + 5Fe3+
+ 10CO2 + 12H2O
Topic 2 Equivalent Concept, Neutralisation Q 5 moles of FeC2O4 require 3 moles of KMnO4
and Redox Titration 3
\ 1 mole of FeC2O4 will require mole of KMnO4.
1. H2O2 acts as an oxidising as well as reducing agent, because 5
oxidation number of oxygen in H2O2 is -1. So, it can be oxidised 9. The balanced chemical reaction is :
to oxidation state 0 or reduced to oxidation state 2.
2MnO-4 + 5SO23- + 6H+ 2Mn 2+ + 5SO24- + 3H2O
H2O2 decomposes on exposure to light. So, it has to be stored in
plastic or wax lined glass bottles in dark for the prevention of Q 5 moles SO2-
3 reacts with 2 moles of KMnO 4

exposure. It also has to be kept away from dust. 2


\ 1 mole of SO2-
3 will react with mole KMnO4.
5
14 Some Basic Concepts of Chemistry

10. The balanced redox reaction is : 17. Na2S4O6 is a salt of H2S4O6 which has the following structure
2MnO4- + 5C2O24- + 16H + 2Mn 2+ + 10CO2 + 16H2O O O
Hence, the coefficients of reactants in balanced reaction are 2, 5 (0) (v)
and 16 respectively. HO S S S S OH

11. Volume strength of H2O2 = Normality 5.6 = 1.5 5.6 = 8.4 V O O

12. In Ba(H2PO2 )2, oxidation number of Ba is +2. Therefore, Difference in oxidation number of two types of sulphur = 5
H2PO2- : 2 (+1) + x + 2 (-2) = - 1 18. Only F and Na show only one non-zero oxidation state.
x=+1 O = O- , O2 - , O2 + ;
Cl = - 1to + 7
13. Equivalent weight in redox system is defined as :
N = - 3 to + 5
Molar mass
E= P = - 3 to + 5
n-factor
Sn = + 2, + 4
Here n-factor is the net change in oxidation number per formula Tl = + 1, + 3 (rare but does exist)
unit of oxidising or reducing agent. In the present case, n-factor
Ti = + 2, + 3, + 4
is 2 because equivalent weight is half of molecular weight. Also,
1 19. Average titrate value is 25.15, but the number of significant
n-factor MnSO4 Mn 2O3 1 (+ 2 + 3)
2 figure cannot be greater than the same in either of them being
MnSO4 MnO2 2 (+ 2 + 4) manipulated.
MnSO4 MnO4- 5 (+ 2 + 7) 20. The balanced reaction is
MnSO4 MnO42- 4 (+ 2 + 6)
6CaO + P4O10 2Ca 3 (PO4 )2
Therefore, MnSO4 converts to MnO2. 852
Moles of P4O10 = =3
14. PLAN This problem includes concept of redox reaction. A redox 284
reaction consists of oxidation half-cell reaction and reduction Moles of CaO required = 3 6 = 18
half-cell reaction. Write both half-cell reactions, i.e. oxidation
half-cell reaction and reduction half-cell reaction.Then balance
Mass of CaO required = 18 56 = 1008 g
both the equations. 21. Meq of oxalate = 10 0.2 2 = 4
Now determine the correct value of stoichiometry of H2SO4.
Meq of MnO2 formed = Meq of oxalate = 4
Oxidation half-reaction, 2 I - I 2 + 2 e - (i)
- Meq of KMnO4 in 20 mL = 4
Here, I is converted into I 2. Oxidation number of I is increasing
Normality of H2O2 20 = 4
from 1 to 0 hence, this is a type of oxidation reaction.
Normality of H2O2 = 0.20 N
Reduction half-reaction 0.20
Molarity of H2O2 = = 0.10 M
2
6H+ + ClO-3 + 6e- Cl - + 3H2O (ii)
The balanced reactions are
l
Here, H2 O releases as a product. Hence, option (d) is
2KMnO4 + 5H2O2 + 3H2SO4 2MnSO4 + 5O2
correct.
+ K2SO4 + 8H2O
Multiplying equation (i) by 3 and adding in equation (ii)
MnO2 + Na 2C2O4 + 2H2SO4 MnSO4 + Na 2SO4
6I- + ClO-3 + 6H+ Cl - + 3I2 + 3H2O + 2CO2 + 2H2O
6I- + ClO-3 + 6H2SO4 Cl - + 3I2 + 3H2O + 6HSO4- 22. The balanced chemical reaction is
CuCO3 + H2SO4 CuSO4 + H2O + CO2
l
Stoichiometric coefficient of HSO-4 is 6.
0.5 1000
Hence, option (a), (b) and (d) are correct. millimol of CuCO3 = = 4.048
123.5
15. Both assertion and reason are factually true but the reason does Millimol of H2SO4 required = 4.048
not exactly explain the assertion. The correct explanation is, Q Millimol = Molarity Volume (in mL)
methyl orange and phenolphthalein changes their colour at 4.048
different pH. Volume = = 8.096 mL
0.50
16. If x is the oxidation state of Cu then :
7
3 + 2 2 + 3x + 7 (- 2) = 0 x =
3
Some Basic Concepts of Chemistry 15

23. The redox reaction involved are : Now, solving equations (i) and (ii),
IO-3 -
+ 5I + 6H +
3I2 + 3H2O x = 10 mL, y = 4 mL and volume of He = 20 14 = 6 mL
10
I2 + 2S2O32-
2I- + S4O62- Vol % of CO = 100 = 50%
20
0.1 4
millimol of KIO3 used = 1000 = 0.467 Vol % of CH4 = 100= 20%
214 20
millimol of I2 formed = 3 0.467 = 1.4 Vol % of He = 30%
millimol of Na 2S2O3 consumed = 2 1.4 = 2.8
2.8 27. The redox reaction involved is :
Molarity of Na 2S2O3 = = 0.062 M
45 H2O2 + 2I- + 2H+ 2H2O + I2

24. Meq of H2O2 = Meq of I2 = Meq of Na 2S2O3 If M is molarity of H2O2 solution, then
0.508 1000
If N is normality of H2O2, then 5M = (Q 1 mole H2O2 1 mole I2)
254
N 25 = 0.3 20 N = 0.24 M = 0.4
Volume strength = N 5.6 = 1.334 V Also, n-factor of H2O2 is 2, therefore normality of H2O2 solution
25. Let the original sample contains x millimol of Fe3O4 and is 0.8 N.
y millimol of Fe2O3. In the first phase of reaction, Volume strength = Normality 5.6 = 0.8 5.6 = 4.48 V
Fe3O4 + I- 3Fe2+ + I2 (n-factor of Fe3O4 = 2) 28. The reaction is
-
Fe2O3 + I 2Fe 2+
+ I2 (n-factor of Fe2O3 = 2) KIO3 + 2KI + 6HCl 3ICl + 3KCl + 3H2O
KIO3 required for 20 mL original KI solution = 3 millimol.
Meq of I2 formed = Meq (Fe3O4 + Fe2O3 )
= Meq of hypo required 7.5 millimol KIO3 would be required for original 50 mL KI.
2x + 2 y = 11 0.5 5 = 27.5 (i) Original 50 mL KI solution contain 15 millimol of KI.
Now, total millimol of Fe2+ formed = 3x + 2 y. In the reaction
After AgNO3 treatment.
Fe2+ + MnO-4 + H+ Fe3+ + Mn 2+ 5 millimol of KIO3 is required, i.e. 10 millimol KI is remaining.
n-factor of Fe2+ = 1 5 millimol KI reacted with 5 millimol of AgNO3.
Meq of MnO-4 = Meq of Fe 2+ 5
Mass of AgNO3 = 170 = 0.85 g
1000
3x + 2 y = 12.8 0.25 5 2 = 32 (ii)
Solving Eqs. (i) and (ii), we get Mass percentage of AgNO3 = 85%
x = 4.5 29. CO2 is evolved due to following reaction :
and y = 9.25 2NaHCO3 Na 2CO3 + H2O + CO2
4.5 pV
Mass of Fe3O4 = 232 = 1.044 g Moles of CO2 produced =
1000 RT
1.044
% mass of Fe3O4 = 100 = 34.80% =
750 123.9

1
3 760 1000 0.082 298
9.25
Mass of Fe2O3 = 160 = 1.48 g = 5 10-3
1000
1.48 Moles of NaHCO3 in 2 g sample = 2 5 10-3 = 0.01
% mass of Fe2O3 = 100 = 49.33%
3 millimol of NaHCO3 in 1.5 g sample
0.01
26. The reaction involved in the explosion process is = 1.5 1000= 7.5
1 2
CO(g ) + O (g ) CO2 (g )
2 x2 Let the 1.5 g sample contain x millimol Na 2CO3, then
x mL mL x mL
2 2x + 7.5 = millimol of HCl = 15
CH4 (g ) + 2 O2 (g ) CO2 (g ) + 2H2O(l ) x = 3.75
y mL 2 y mL y mL 7.5 84
Mass of NaHCO3 = = 0.63 g
The first step volume contraction can be calculated as : 1000
x 3.75 106
x + + y + 2 y - (x + y) = 13 Mass of Na 2CO3 = = 0.3975 g
2 1000
x + 4 y = 26 (i) 0.63
% mass of NaHCO3 = 100 = 42 %
The second volume contraction is due to absorption of CO2. 1.50
Hence, x + y = 14 (ii) 0.3975
% mass of Na 2CO3 = 100 = 26.5%
1.5
16 Some Basic Concepts of Chemistry

30. Mass of Fe2O3 = 0.552 g 35.5


33. Mass of chlorine in 1.0 g X = 2.9 = 0.717 g
0.552 143.5
millimol of Fe2O3 = 1000 = 3.45
160 Now, the empirical formula can be derived as :
During treatment with Zn-dust, all Fe3+ is reduced to Fe2 + , C H Cl
hence % wt : 24.24 4.04 71.72
millimol of Fe2 + (in 100 mL) = 3.45 2 = 6.90 Mole : 2 4 2
6.90
In 25 mL aliquot, = 1.725 millimol Fe2+ ion. Simple ratio : 1 2 1
4
Empirical formula = CH2Cl.
Finally Fe2+ is oxidised to Fe3+ , liberating one electron per Fe2+
ion. Therefore, total electrons taken up by oxidant. Because X can be represented by two formula of which one
gives a dihydroxy compound with KOH indicates that X has two
= 1.725 10-3 6.023 1023
chlorine atoms per molecule.
= 1.04 1021 X = C2H4Cl 2 with two of its structural isomers.
31. With KMnO4, oxalate ion is oxidised only as : Cl CH2 CH2 Cl and CH3 CHCl 2
5C2O42- + 2MnO-4 +
+ 16H 2Mn 2+
+ 10CO2 + 8H2O I II
On treatment with KOH, I will give ethane-1, 2-diol, hence it is
Let, in the given mass of compound, x millimol of C2O2-
4 ion is Y. Z on treatment with KOH will give ethanal as
present, then
ClCH2CH2Cl + OH- CH2 CH2
Meq of C2O42- = Meq of MnO-4

2x = 0.02 5 22.6 OH OH
x = 1.13 (Y )
- 2+ H2 O
At the later stage, with I , Cu is reduced as : CH3CHCl 2 + KOH CH3CH(OH)2 CH3CHO
2Cu 2+ + 4I - 2CuI + I2 Unstable (Z )

and I2 + 2S2O32- 2I + -
S4O62- 34. Let the n-factor of KMnO4 in acid, neutral and alkaline media
Let there be x millimol of Cu 2+ . are N 1, N 2 and N 3 respectively. Also, same volumes of reducing
agent is used everytime, same number of equivalents of KMnO4
Meq of Cu 2+ = Meq of I2 = meq of hypo
would be required every time.
x = 11.3 0.05 = 0.565 100 5
20N 1 = N 2 = 100N 3 N 1 = N 2 = 5N 3
Moles of Cu 2+ : moles of C2O42- = 0.565 : 1.13 = 1 : 2 3 3
Also, n-factors are all integer and greater than or equal to one but
32. Let us consider 10 mL of the stock solution contain x millimol less than six, N 3 must be 1.
oxalic acid H2C2O4 and y millimol of NaHC2O4. N 1 = 5, N 2 = 3
When titrated against NaOH, basicity of oxalic acid is 2 while \ In acid medium MnO-4 Mn 2+
that of NaHC2O4 is 1. In neutral medium MnO-4 Mn 4+
2x + y = 3 0.1 = 0.3 (i) In alkaline medium MnO-4 Mn6+
When titrated against acidic KMnO4, n-factors of both oxalic
meq of K2Cr2O7 required = 100
acid and NaHC2O4 would be 2.
100 = 1 6 V (n-factor = 6)
2x + 2 y = 4 0.1 = 0.4 (ii)
V = 100/ 6 = 16.67 mL
Solving equations (i) and (ii) gives
1
y = 0.1, x = 0.1 35. Meq of MnO-4 required = 20 5 = 2
0.1 50
In 1.0 L solution, mole of H2C2O4 = 100 = 0.01 Meq of Fe2+ present in solution = 2
1000
0.1 millimol of Fe2+ present in solution = 2 (n-factor = 1)
Mole of NaHC2O4 = 100 = 0.01 Also,
1000
Q 4 millimol of Fe2+ are formed from 1 millimol N2H4
Mass of H2C2O4 = 90 0.01 = 0.9 g 1 1
Mass of NaHC2O4 = 112 0.01= 1.12 g \ 2 millimol Fe2+ from 2 = millimol N2H4
4 2
Some Basic Concepts of Chemistry 17

Therefore, molarity of hydrazine sulphate solution 38. During heating MCO3 is converted into MO liberating CO2
1 1 1 while BaO is remaining unreacted :
= =
2 10 20 Heat
1 MCO3 (s) MO(s) + CO2 (g ) 0.44 g = 0.01 mol
In 1 L solution mol N2H6SO4 is present.
20 BaO(s) BaO(s)
1 4.08 g 3.64 g
Amount of N2H6SO4 = 130 = 6.5 gL-1
20 From the decomposition information, it can be deduced that the
36. Molecular weight of Na 2CO3 10H2O = 286 original mixture contained 0.01 mole of MCO3 and the solid
residue, obtained after heating, contain 0.01 mole (10 millimol)
1 1000
Molarity of carbonate solution = = 0.035 of MO.
286 100
Also, millimol of HCl taken initially = 100
Normality of carbonate solution = 2 0.035 = 0.07 N
85 millimol of NaOH used in back-titration = 16 2.5 = 40
In acid solution : Normality of HNO3 = = 0.02
2000 millimol of HCl reacted with oxide residue = 60
5 4.8 HCl reacts with oxides as :
Normality of HCl = = 0.012
2000 MO + 2HCl MCl 2 + H2O
Let normality of H2SO4 in final solution be N. 10 millimol 20 millimol

(N + 0.02 + 0.012) 30 = 0.07 42.9 BaO + 2HCl BaCl 2 + H2O


N = 0.0681 60 20 = 40 millimol
Gram equivalent of SO2- 4 in 2 L solution = 2 0.0681 Therefore, the residue contain 20 millimol of BaO.
= 0.1362 Also, molar mass of BaO = 138 + 16
96 = 154
Mass of SO2-
4 in solution = 0.1362 = 6.5376 g 154 20
2 Mass of BaO = = 3.08 g
1000
37. For the oxidation of A n+ as :
Mass of MCO3 = 4.08 3.08 = 1.0 g
A n+ AO-3 n-factor = 5 n
Q 0.01 mole of MCO3 weight 1.0 g
Gram equivalent of A n+ = 2.68 10-3 (5 - n)
\ 1 mole of MCO3 = 100 g
Now equating the above gram equivalent with gram equivalent
of KMnO4 : 100 = (Atomic weight of metal) + (12 + 3 16)
2.68 10-3 (5 - n) = 1.61 10-3 5 Atomic weight of metal = 40, i.e. Ca
n=+2
CHAPTER TEST
1. In the titration of a certain H2SO4 solution, 60 mL of Assertion-Reason Type
5.0 M NaOH solution was used to completely
Following two questions have assertion followed by
neutralise 75 mL of the acid. The molarity of the acid the reason. Answer them according to the following
solution may be expressed as options.
5 M 60 mL 5 M 75 mL 2 (a) Both assertion and reason are correct and reason is
(a) (b)
2 75 mL 60 mL the correct explanation of the assertion,
75 mL 2 60 mL 75 mL 2 (b) Both assertion and reason are correct but reason is
(c) (d)
5.0 M 60 mL 5.0 M not the correct explanation of assertion.
(c) Assertion is correct but reason is incorrect.
2. 20 mL of a solution containing equal moles of Na2CO3 (d) Assertion is incorrect but reason is correct.
and NaHCO3 required 16 mL of a 0.16 M HCl solution
7. Assertion If certain volume of a basic solution require
to reach the phenolphthalein end point. What volume
x mL of HCl, 2x mL of H2SO4 of same molarity would
of a 0.10 M H2SO4 solution would have been required be required.
had methyl orange been used as indicator?
Reason HCl is a monobasic acid while H2SO4 is a
(a) 38.40 mL (b) 24.60 mL dibasic acid.
(c) 19.20 mL (d) 0.30 mL
8. Assertion If 10 mL of a H2O2 solution required 8.00 mL
3. When aqueous solution of Na2S is titrated with dilute of 0.02 M acidified KMnO4 solution for complete
and acidified KMnO4 solution, Na2SO3 is formed. In oxidation, 12.50 mL of same H2O2 will oxidise completely
this reaction, moles of KMnO4 reduced per mole of to 5.00 mL of 0.10 M Na2C2O4 solution.
Na2S is
Reason H2O2 act as both oxidising as well as
(a) 0.833 (b) 1.2 (c) 1.50 (d) 1.8
reducing agent.
4. For the following metals A, B, C, D react with each
other : (NR = No reaction) 9. The alkenes are compounds of carbon and hydrogen
with the general formula CnH2n. If 0.561 g of any alkene
A + B + NR B + C + NR
is burned in excess oxygen, what number of moles of
B + D + B + + D C + D + C + + D H2O is formed?
A+ + C C + + A D + A+ D + + A (a) 0.0400 mol (b) 0.0600 mol
Which is the order of the metals in increasing reducing (c) 0.0800 mol (d) 0.400 mol
strength? 10. What is the density (in g mL -1) of a 3.60 M aqueous
(a) B < D < A< C (b) C < B < D < A sulphuric acid solution that is 29.0% H2SO4 by mass?
(c) A < D < B < C (d) < C < A< B
(a) 1.22 (b) 1.45
5. 40 mL 0.05 M solution of sodium sesquicarbonate (c) 1.64 (d) 1.88
dehydrate (Na2CO3NaHCO32H2O) is titrated against 11. Magnetite, Fe3O4, can be converted into metallic iron
0.05 M HCl solution, x mL of acid is required to reach by heating with carbon monoxide as represented by
the phenolphthalein end point while y mL of same acid this equation:
were required when methyl orange indicator was used Fe3O4 (s) + CO (g) Fe (s) + CO2 (g)
in a separate titration. Which of the following is(are) The kilograms of Fe3O4 which must be processed in
correct statements? this way to obtain 5.00 kg of iron, if the process is 85%
(a) y - x = 80 mL efficient is closest to? [M : Fe = 56]
(b) y + x = 160 mL
(a) 6.92 kg (b) 8.15 kg
(c) If the titration is started with phenolphthalein indicator
(c) 20.8 kg (d) 24.4 kg
and methyl orange is added at the end point, 2 x mL of
HCl would be required further to reach the end point 12. How many H2O molecules are there in a snowflake
(d) If the same volume of same solution is titrated against that weighs 4.0 10 - 4 g?
0.10 M NaOH, x/2 mL of base would be required
(a) 1.3 1019 (b) 2.4 10 20
6. Which of the following regarding oxalate compound -5
(c) 2.2 10 (d) 6.02 10 23
is/are true?
(a) Oxalic acid (H 2 C 2 O 4 ) can be estimated by titrating 13. Atomic weight of an element X is 120 when one amu is
against either KOH or KMnO 4 solution. defined as 1/18th part by weight of an element of C12.
(b) KHC 2 O 4 can be estimated by titrating against either On the same scale, atomic weight of another element
KOH or K 2 CrO 4 and in both analyses equivalent
Y is 72. Which of the following statement regarding X
weight of KHC 2 O 4 is 64. (MW of KHC 2 O 4 = 128)
(c) K 2 C 2 O 4 can be estimated by titrating against either and Y is(are) correct?
HCl or KClO 3 and in both analyses equivalent weight (a) On conventional scale, atomic weight of X is 80
of K 2C 2O 4 is 83. (MW of K 2 C 2 O 4 = 166) (b) On conventional scale, atomic weight of Y is 108
(d) If 10 mL of a K 2 C 2 O 4 solution required 8.0 mL of a (c) On a scale when an amu is defined to be 1/30th of the
0.12 M HCl solution, 10 mL of the same K 2 C 2 O 4 weight of an atom of C12, atomic weight of X is 200
solution would require 9.60 mL of a 0.02 M acidified (d) On a scale when an amu is defined to be 1/15th of the
KMnO 4 solution. weight of an atom of C12, atomic weight of Y is 90
14. Which of the following statement regarding Avogadro 17. What is the approximate molar mass of unknown metal M?
number is(are) correct? (a) 20 (b) 40 (c) 56 (d) 60
(a) It is 6.023 10 23
Assertion-Reason Type
(b) It is the number of atoms present in exactly 12 g of C-12
isotope Following two questions have assertion followed by the reason.
(c) It is the number of atoms present in 1.0 mole of any Answer them according to the following options.
substance (a) Both assertion and reason are correct and reason is the
(d) It is the number of atoms of deuterium present in its 2.0 g correct explanation of the assertion.
(b) Both assertion and reason are correct but reason is not the
Comprehension for Q. Nos. 15 to 17 correct explanation of assertion.
4.0 g of a mixture of NaCl and an unknown metal iodide MI 2 was (c) Assertion is correct but reason is incorrect.
dissolved in water to form its aqueous solution. To this aqueous (d) Assertion is incorrect but reason is correct.
solution, aqueous solution of AgNO 3 was added gradually so 18. Assertion The average mass of an atom of Mg is 24.305 u,
that silver halides are precipitated. The precipitates were which is not the actual mass of an atom of Mg.
weighed at regular interval and following curve for the mass of
precipitate versus volume of AgNO 3 added was obtained. With Reason This is the average mass determined considering
the knowledge of the fact that halides are precipitated different isotopes of Mg present in natural Mg.
successively, i.e. when less soluble halide is precipitating, the
19. Assertion A 8.0 g N2H4 (M = 32) has more atoms than 6.0
other halide remain in the solution, answer the following
questions: (Molar mass of Ag = 108, I = 127, Na = 23). g H2O.
Reason N2H4 has more atoms per molecule than water.
9
20. Match the statements of Column I with values of Column II.
8
7 Column I Column II
Mass of precipitate (in g)

6 A. Different number of p. 4.25 g NH3 and


atoms 4.5 g of H2O
5
B. Same number of q. 2.20 g CO2 and
4 molecules 0.90 g H2O
3 C. Same numbers of atoms r. 4.0 g CH3Cl and
as well as molecules 5.0 g NH3
2
D. Different numbers of s. 4.80 g O2 and
1 atoms as well as 2.80 g CO
0 molecules
5 10 15 20 25 30 35 40 45 50
21. On a conventional scale, atomic weight of sulphur is 32. If on
Volume of AgNO3
a new scale, an amu is defined as one-third by weight of an
atom of C-12 isotope, what would be the atomic weight of
15. What is the approximate mass percentage of MI2?
sulphur on this new scale?
(a) 25 (b) 40
(c) 60 (d) 75 22. If equal volumes of 3.5 M CaCl2 and 3.0 M NaCl are mixed,
16. What is the approximate molarity of AgNO3 solution? what would be the molarity of chloride ion in the final
(a) 0.1 (b) 0.5 solution?
(c) 1.0 (d) 1.5

Answers
1. (a) 2. (c) 3. (b) 4. (c) 5. (a, b, c, d) 6. (a, c, d) 7. (d) 8. (b) 9. (a) 10. (a)
11. (b) 12. (a) 13. (b, c) 14. (a, b, d ) 15. (d) 16. (c) 17. (b) 18. (a) 19. (b)
20. A s; B p; C q; D r 21. (8) 22. (5)
2
Atomic Structure

Preliminary Developments and Bohrs Model


l
Terms Related to Atom

Atomic number (Z ) Equals to the number of protons (or electrons) in a neutral atom.
Mass number (A) Number of protons (Z) + number of neutrons (N)

Isotopes Atoms of same element having different mass number, e.g. 6C12, 6C14.

Isobars Atoms of different elements having same mass number, e.g. 6C14, 7 N14.

Atoms of different elements having same number of neutrons,


Isotones
e.g. 19K39, 20Ca 40, 16S36.

Isoelectronic Species having same number of electrons, e.g. K + , Ca 2+ , Ar, Cl- .

Different atoms having same neutron to proton difference (neutron excess),


Isodiaphers
e.g. 92U 238 and 90Th234 both have ( N - Z ) equals to 54.

Isosters Species having same number of atoms and electrons are isosters, e.g. N 2, CO.

l
The Details About Some of the Important Subatomic Particles are given in the Table Below:
Particles and Absolute Relative Mass Mass Discoverer
symbol charge (C) charge (gram) (MeV/C 2)

Electron (e) -1.602 10-19 -1 9.1 10-28 0.511 JJ Thomson


Proton (p) +1.602 10-19 +1 1.6726 10-24 938.5 Rutherford
-24
Neutron (n) 0 0 1.6749 10 940 Chadwick
Positron (B + / e+ )
+1.602 10-19 +1 9.1 10-28 0.511 D Anderson
(anti-electron)
Emilio Segra and
Anti-proton ( p) -1.602 10-19 -1 1.6726 10-24 938.5
Owen chamberlain

l
Rutherfords Nuclear Model of Atom Rutherford performed an alpha particle ( He2+ ) scattering experiment on a thin gold foil and
presented that :
(i) most part of atom is empty.
(ii) every atom possesses a highly dense, positively charged centre called nucleus.
(iii) entire mass of atom is concentrated inside the nucleus.
(iv) later Rutherford model was abandoned due to its failure to comply with classical theory of electromagnetic radiation. This theory
also failed to explain the line spectrum of H-atom.
Atomic Structure 21

l
Plancks Equation l
Potential Energy
E = hn =
hc - Ze2 2kZ 2
Ep = =- 2
l 4 pe 0 r n
(where, h = 6.625 10-34 Js) l
Speed of Electron in th Bohr Orbit
l
Black Body Radiation Radiation by a black body (an 2.18 106 Z
ideal body that emits and absorbs radiations of all ms -1 .
n
frequencies) can be explained by Plancks quantum theory.
Number of revolutions made by an electron in nth Bohrs orbit is
l
Photoelectric Effect When a light of frequency more v
than threshold frequency (n 0 ) strikes a metal surfaces it Revolution/second = n ; (vn = Speed in nth orbit)
2p rn
ejects electron. This phenomenon is called photoelectric
effect. Advanced Concept
1
Hence, men 2 = hn - hn 0 (Quantum Mechanical Theory)
2
= h (n - n 0 ) Electronic Configuration and
...(i) Quantum Number
where, n = Frequency of light, l
Emission Spectrum of H-atom The frequency, wavelengths
n 0 = Threshold frequency or wave-number of a spectral line in any of the series in the emission
h = Plancks constant = 6.625 10 -34
Js spectrum of hydrogen atom can be calculated using the following
Rydbergs equation :
1 1 1
men 2 = hc - 1 1 1
2 l l0 = n = RH Z 2 2 - 2 ; RH = 1.09678 107 m - 1
l n1 n2
...(ii)
(i) For Lyman series : n1 = 1, n2 = 2, 3
l
Bohrs Model Bohr proposed an idea of stationary orbits
in which electron revolves. According to Bohrs model, (occur in UV region)
electron does not lose energy as long as it stays in an (ii) For Balmer series : n1 = 2, n2 = 3, 4
stationary orbit. When an electron jumps to lower stationary (occur in visible region)
orbit, energy is lost in the form of electromagnetic radiation. (iii) For Paschen series : n1 = 3, n2 = 4 , 5
Conversely when energy is supplied, electron jumps to
(occur in IR region)
higher stationary orbit. According to Bohrs model,
(iv) For Brackett series : n1 = 4, n2 = 5, 6
(i) Angular momentum of an electron is quantised :
nh (occur in IR region)
Angular momentum ( mvr ) = (v) For Pfund series : n1 = 5, n2 = 6, 7
2p
(i) (occur in IR region)
where, n = 1, 2, 3, , l
Wave-Particle Duality (de-Broglie Relationship)
(orbit number) h
l= ; ( p = momentum ( mv ))
(ii) Centrifugal force of orbiting electron is exactly balanced p
by the electrostatic attraction between nucleus and
electron.
l
Bohrs de-Broglie Relationship 2p r = n l, i.e. in a given
mv 2
Ze 2 stationary orbit, the number of de-Broglie wavelengths is equal to
= (ii) orbit number. If an electron at rest is accelerated by a potential
r 4 pe 0 r2
difference of V volt, then de-Broglie wavelength is :
l
Bohrs Radius Using the above relationships (i and ii), h
radius of a stationary orbit is l=
2meV
n2
rn = a0 , l
Heisenbergs Uncertainty Principle According to this
Z
principle, simultaneous and accurate measurement of both position
where, a0 = 0.529 (First Bohr and momentum of an electron in an atom is impossible.
radius)
h
l
Energy of Stationary Orbit (E n ) Dx Dp
4p
- kZ 2
En = , (Q k = 13.6 eV = 2.18 10-18 J) Here, (Dx is uncertainty in position and Dp is uncertainty in
n2 momentum.)
22 Atomic Structure

l
Quantum Numbers To describe an electron completely l
Electronic Configuration Electrons are filled in atomic
inside the atom, four sets of quantum numbers are required. They orbitals in increasing order of their energy according to Aufbau
are as : principle :
(i) Principal Quantum Number ( n ) This specify position 1s
and energy of an electron in the atom. Possible values of n 2s 2p
are 0, 1, 2, 3, , . 3s 3p 3d
(ii) Angular Momentum (Azimuthal or Subsidiary) 4s 4p 4d 4f
Quantum Number ( l) This is used to specify subshell 5s 5p 5d 5f
(orbital). Possible values of l are 0, 1, 2, ( n - 1). Orbitals
6s 6p 6d
with different values of l are denoted as :
l = 0, for s-orbital spherical 7s 7p
l = 1, for p-orbital dumb-bell shape l
During filling up of electrons in atomic orbitals. Paulis
l = 2, for d-orbital double dump-bell shape
exclusion principle must be obeyed, i.e. no two electrons in an
l = 3, for f-orbital etc.
atom can have all four sets of quantum numbers same. This limits
The value of l also determine shape of orbital as mentioned the number of electrons in an orbital to two.
above.
The value of l determines orbital angular momentum (L)
l
Hunds rule of maximum multiplicity is obeyed, i.e. orbitals of
same energies (degenerate atomic orbitals) are first singly
as :
occupied, then pairing starts. Exception occurs in case of Cr (24)
h
L = l ( l + 1) and Cu (29).
2p
Cr =1s2 , 2s2 2 p 6 , 3s2 3 p 6 3d 5 , 4 s1 (not 3d 4 4 s2 )
The value of l also determine the magnitude of magnetic
moment as : Cu =1s2 , 2s2 2 p 6 , 3s2 3 p 6 3d 10 , 4 s1 (not 3d 9 4 s2 ).
eh eh Quantum Mechanical Model
= 9.27 10-14 J
l
mL = l ( l + 1) where,
4 p mC 4p mC (The Schrodingers Equation)
(iii) Magnetic Quantum Number ( m ) It determine the 2y 2y 2y 8 p2 m
2
+ 2
+ 2
+ (E - V ) y = 0
preferred orientation of orbitals in three dimensional space. x y z h2
Its possible values are : y = Wave function, represents an orbital
m = - l , ...., 0, ...., + l E = Total energy of an electron
e.g. for l = 2, m = - 2, 1, 0, +1, +2. V = Potential energy associated with electron
Total values of m for a given value of l = ( 2l + 1) = total
l
Solution of the second order differential equation
number of orbitals in a given orbital. (Schrodinger equation) gives the normalised wave functions
Splitting of spectral lines occur when placed in a magnetic
l
( y ). Normalised wave-function for some of the orbitals are :
field (Zeeman effect) or in an electric field (Stark effect). 3/ 2
Total lines from a single line in the normal spectrum 1 1
for s-orbital y1 s = e- r / a 0
= ( 2l + 1). p a0
l
Total number of orbitals in nth orbit = n 2 1
1
3/ 2
r - r / 2a 0
y 2s = 2 - a e
l
Total number of electrons in nth orbit = 2n 2 4 2p a0 0
(iv) Spin Quantum Number ( s ) Electrons spin on its own 4
1 1
axis like a top, in clockwise and anti-clockwise directions. for px -orbital y 2 px = r sin q cos f e- r / 2 a 0
4 2p a0
The two directions of spinning is denoted by spin quantum
number as : Radial Wave Functions and Number of Nodes As y
1 1 is plotted against distance from nucleus ( r ), ( n - l - 1) radial
s = + and -
2 2 nodes are obtained. At nodes, y changes its sign.
The spin quantum numbers are also denoted by up-half Probability density ( y 2 ) and radial distribution function
arrow ( ) and down-half arrow ( ) but neither the +1/2 and | Rn , l |2 r2 . For an orbital, probability density, y 2 is
1/2 or the or are specific for any direction, they just
represent the two opposite directions of spinning of maximum at nucleus but, | Rn , l |2 r2 is minimum at nucleus.
electrons. Also, when any of these two functions are plotted against r,
( n - l - 1) nodes are obtained.
Atomic Structure 23

Some plots of y and | R n, l |2 r 2

1s
1. y y 2s

|Rnl| 2r 2

|Rnl| 2r 2
3.

r r r r

3p
y 3s y 4.

|Rnl| 2r 2
2.

r r
r

Topic 1 Preliminary Developments and Bohrs Model


Objective Questions I (Only one correct option) 7. Rutherfords experiment on scattering of a-particles showed
1. Rutherfords experiment, which established the nuclear for the first time that the atom has (1981, 1M)

model of the atom, used a beam of (2002, 3M) (a) electrons (b) protons
(a) b -particles, which impinged on a metal foil and got (c) nucleus (d) neutrons
absorbed
(b) g-rays, which impinged on a metal foil and got scattered Objective Questions II
(c) helium atoms, which impinged on a metal foil and got (One or more than one correct option)
scattered 8. The energy of an electron in the first Bohr orbit of H-atom is
(d) helium nuclei, which impinged on a metal foil and got 13.6 eV. The possible energy value(s) of the excited state(s)
scattered
for electrons in Bohr orbits of hydrogen is (are) (1988)
2. Rutherfords alpha particle scattering experiment (a) - 3.4 eV (b) - 4.2 eV
eventually led to the conclusion that (1986, 1M) (c) - 6.8 eV (d) + 6.8 eV
(a) mass and energy are related
9. The atomic nucleus contains (1988, 1M)
(b) electrons occupy space around the nucleus
(a) protons (b) neutrons
(c) neutrons are burried deep in the nucleus
(c) electrons (d) photons
(d) the point of impact with matter can be precisely determined
10. The sum of the number of neutrons and proton in the isotope of
3. The radius of an atomic nucleus is of the order of
hydrogen is (1986, 1M)
(a) 10-10 cm (b) 10-13 cm (1985, 1M)
(a) 6 (b) 5
(c) 10-15 cm (d) 10-8 cm
(c) 4 (d) 3
4. Bohrs model can explain (1985, 1M)
11. When alpha particles are sent through a thin metal foil, most of
(a) the spectrum of hydrogen atom only
them go straight through the foil, because (1984, 1M)
(b) spectrum of an atom or ion containing one electron only
(a) alpha particles are much heavier than electrons
(c) the spectrum of hydrogen molecule
(b) alpha particles are positively charged
(d) the solar spectrum
(c) most part of the atom is empty space
5. The increasing order (lowest first) for the values of e/m (d) alpha particles move with high velocity
(charge/mass) for electron ( e ), proton (p), neutron (n) and
alpha particle (a) is (1984, 1M)
12. Many elements have non-integral atomic masses, because
(a) they have isotopes (1984, 1M)
(a) e, p, n, a (b) n, p, e, a (c) n, p, a , e
(d) n, a, p, e (b) their isotopes have non-integral masses
(c) their isotopes have different masses
6. Rutherfords scattering experiment is related to the size of (d) the constituents, neutrons, protons and electrons, combine to
the (1983, 1M) give fractional masses
(a) nucleus (b) atom (c) electron (d) neutron
24 Atomic Structure

Match the Columns Fill in the Blanks


13. According to Bohrs theory, 14. The light radiations with discrete quantities of energy are
En = Total energy K n = Kinetic energy called ................ . (1993, 1M)
Vn = Potential energy rn = Radius of nth orbit 15. The mass of a hydrogen is kg. (1982, 1M)
Match the following : (2006, 6M)
16. Isotopes of an element differ in the number of in their
Column I Column II nuclei.
A. Vn / K n = ? p. 0 (1982, 1M)

If radius of nth orbit 17. Elements of the same mass number but of different atomic
B. q. 1 numbers are known as .
E nx , x = ? (1983, 1M)

Angular momentum in Subjective Questions


C. r. 2
lowest orbital
1 18. With what velocity should an a-particle travel towards the
y
D. n Z , y = ? s. 1 nucleus of a copper atom so as to arrive at a distance 10-13 m
r
from the nucleus of the copper atom ? (1997 (C), 3M)

Topic 2 Advanced Concept (Quantum Mechanical Theory)


Electronic Configuration and Quantum Number
Objective Questions I (Only one correct option) 6. Which hydrogen like species will have same radius as that
1. Which of the following is the energy of a possible excited of Bohr orbit of hydrogen atom? (2004, 1M)
2+ 3+
state of hydrogen? (2015 Main) (a) n = 2, Li (b) n = 2, Be
(a) + 13.6 eV (b) 6.8 eV (c) n = 2, He+ (d) n = 3, Li 2+
(c) 3.4 eV (d) + 6.8 eV 7. If the nitrogen atom had electronic configuration 1s7 , it
2. The correct set of four quantum numbers for the valence would have energy lower than that of the normal ground
electrons of rubidium atom ( Z = 37 ) is (2013 Main) state configuration 1s2 2s2 2 p 3 , because the electrons
1 1
(a) 5, 0, 0, + (b) 5, 1, 0, + would be closer to the nucleus, yet 1s7 is not observed,
2 2 because it violates
1 1
(c) 5, 1, ,1, + (d) 5, 0, 1, + (a) Heisenberg uncertainty principle (2002, 3M)
2 2
(b) Hunds rule
3. Energy of an electron is given by (c) Pauli exclusion principle
Z2 (d) Bohr postulate of stationary orbits
E = - 2.178 10-18 J 2
n 1 1
(2013 Main) 8. The quantum numbers + and - for the electron spin
Wavelength of light required to excite an electron in an 2 2
hydrogen atom from level n = 1to n = 2 will be represent (2001, 1M)
(a) rotation of the electron in clockwise and anti-clockwise
(h = 6.62 10-34 Js and c = 3.0 108 ms -1 )
direction respectively
(a) 1.214 10-7 m (b) 2.816 10-7 m (b) rotation of the electron in anti-clockwise and clockwise
-7
(c) 6.500 10 m (d) 8.500 10-7 m direction respectively
(c) magnetic moment of the electron pointing up and down
4. The kinetic energy of an electron in the second Bohr orbit of a respectively
hydrogen atom is [a0 is Bohr radius] (2012) (d) two quantum mechanical spin states which have no
h2 h2 classical analogue
(a) (b)
4p 2ma02 16p 2ma02 9. The wavelength associated with a golf ball weighing 200 g
h2 h2 and moving at a speed of 5 m/h is of the order (2001, 1M)
(c) (d)
32p 2 ma02 64p 2ma02 (a) 10-10 m (b) 10-20 m (c) 10-30 m (d) 10-40 m

5. The number of radial nodes in 3s and 2p respectively are 10. The number of nodal planes in a px orbital is (2001, 1M)
(a) 2 and 0 (b) 0 and 2 (2005, 1M) (a) one (b) two
(c) 1 and 2 (d) 2 and 1 (c) three (d) zero
Atomic Structure 25

11. The electronic configuration of an element is 23. The orbital diagram in which the Aufbau principle is violated
1s2 , 2s2 2 p 6 , 3s2 3 p 6 3d 5 , 4 s1 . This represents its (2000, 1M) (1988, 1M)
(a) excited state (b) ground state (a)
(c) cationic form (d) anionic form
(b)
12. The electrons, identified by quantum numbers n and l,
(i) n = 4, l = 1, (ii) n = 4, l = 0, (iii) n = 3, l = 2, (iv) n = 3, l = 1 (c)
can be placed in order of increasing energy, from the lowest
to highest, as (1999, 2M)
(d)
(a) (iv) < (ii) < (iii) < (i) (b) (ii) < (iv) < (i) < (iii)
(c) (i) < (iii) < (ii) < (iv) (d) (iii) < (i) < (iv) < (ii) 24. The wavelength of a spectral line for an electronic transition
13. The energy of an electron in the first Bohr orbit of H-atom is is inversely related to (1988, 1M)
13.6 eV. The possible energy value(s) of the excited state(s) (a) the number of electrons undergoing the transition
for electrons in Bohr orbits of hydrogen is (are) (1998, 2M) (b) the nuclear charge of the atom
(a) 3.4 eV (b) 4.2 eV (c) 6.8 eV (d) + 6.8 eV (c) the difference in the energy of the energy levels involved in
the transition
14. For a d-electron, the orbital angular momentum is(1997, 1M) (d) the velocity of the electron undergoing the transition
h h h h
(a) 6 (b) 2 (c) (d) 2 25. The ratio of the energy of a photon of 200 wavelength
2p 2p 2p 2p
radiation to that of 4000 radiation is (1986, 1M)
15. The first use of quantum theory to explain the structure of 1 1
atom was made by (1997, 1M)
(a) (b) 4 (c) (d) 2
4 2
(a) Heisenberg (b) Bohr
26. Which one of the following sets of quantum numbers
(c) Planck (d) Einstein
represents an impossible arrangement? (1986, 1M)
16. Which of the following has the maximum number of n l m s
unpaired electrons ? (1996, 1M) (a) 3 2 2 1/2
(a) Mg2+ (b) Ti 3+ (c) V3+ (d) Fe2+ (b) 4 0 0 1/2
1
17. The orbital angular momentum of an electron in 2s-orbital is (c) 3 2 3
(1996, 1M) 2
1 h h h 1
(a) + (b) zero (c) (d) 2 (d) 5 3 0 -
2 2p 2p 2p 2
18. Which of the following relates to photons both as wave 27. Electromagnetic radiation with maximum wavelength is
motion and as a stream of particles ? (1992, 1M) (a) ultraviolet (b) radio wave (1985, 1M)
(a) Interference (b) E = mc2 (c) X-ray (d) infrared
(c) Diffraction (d) E = hn 28. Which electronic level would allow the hydrogen atom to
19. Which of the following does not characterise X-rays ? absorb a photon but not to emit a photon? (1984, 1M)
(a) The radiation can ionise gases (1992, 1M) (a) 3s (b) 2p (c) 2s (d) 1s
(b) It causes ZnS to fluoresce 29. Correct set of four quantum numbers for the valence
(c) Deflected by electric and magnetic fields (outermost) electron of rubidium ( Z = 37 ) is (1984, 1M)
(d) Have wavelengths shorter than ultraviolet rays 1 1
(a) 5, 0, 0, + (b) 5, 1, 0 , +
20. The correct set of quantum numbers for the unpaired electron 2 2
of chlorine atom is (1989, 1M) 1 1
(c) 5, 1, 1, + (d) 6, 0, 0, +
n l m n l m 2 2
(a) 2 1 0 (b) 2 1 1 30. The principal quantum number of an atom is related to the
(c) 3 1 1 (d) 3 0 0 (a) size of the orbital (1983, 1M)
21. The correct ground state electronic configuration of (b) spin angular momentum
chromium atom is (1989, 1M) (c) orientation of the orbital in space
(a) [ Ar ] 3d 5 4 s1 (b) [ Ar ] 3d 4 4 s2 (d) orbital angular momentum
(c) [ Ar ] 3d 6 4 s0 (d) [ Ar ] 4 d 5 4 s1 31. Any p-orbital can accommodate upto (1983, 1M)
(a) four electrons
22. The outermost electronic configuration of the most
(b) six electrons
electronegative element is (1988, 90, 1M)
(c) two electrons with parallel spins
(a) ns2 np3 (b) ns2 np4 (c) ns2 np5 (d) ns2 np6
(d) two electrons with opposite spins
26 Atomic Structure

Objective Questions II 38. The orbital angular momentum quantum number of the
state S 2 is (2010)
(One or more than one correct option) (a) 0 (b) 1 (c) 2 (d) 3
32. The ground state electronic configuration of nitrogen atom can
be represented by (1999, 3M) Match the Columns
(a) 39. Match the entries in Column I with the correctly related
quantum number(s) in Column II. (2008, 6M)
(b)
Column I Column II
(c) A. Orbital angular momentum of p. Principal
the electron in a quantum
(d) hydrogen-like atomic orbital. number
B. A hydrogen-like one-electron q. Azimuthal
33. Which of the following statement (s) is (are) correct ? wave function obeying Paulis quantum
(1998, 2M) principle. number
(a) The electronic configuration of Cr is [Ar] 3d 5 4 s1 (atomic
C. Shape, size and orientation of r. Magnetic
number of Cr = 24)
hydrogen-like atomic orbitals. quantum
(b) The magnetic quantum number may have a negative value number
(c) In silver atom, 23 electrons have a spin of one type and 24 of
D. Probability density of electron s. Electron spin
the opposite type. (atomic number of Ag = 47)
at the nucleus in quantum
(d) The oxidation state of nitrogen in HN3 is 3
hydrogen-like atom. number
76
34. An isotone of 32 Ge is (1984, 1M)
77 77
(a) 32 Ge (b) 33 As Fill in the Blanks
77 78
(c) 34 Se (d) 34 Se 40. The outermost electronic configuration of Cr is
.......................... . (1994, 1M)
Assertion and Reason 41. 8 g each of oxygen and hydrogen at 27C will have the total
Read the following questions and answer as per the direction kinetic energy in the ratio of .......... . (1989, 1M)
given below :
42. The uncertainty principle and the concept of wave nature of
(a) Both Statement I and Statement II are correct; Statement II matter were proposed by ............ and .............respectively.
is the correct explanation of Statement I (1988, 1M)
(b) Both Statement I and Statement II are correct; Statement II 43. Wave functions of electrons in atoms and molecules are
is not the correct explanation of Statement I called .............. . (1993, 1M)
(c) Statement I is correct; Statement II is incorrect
44. The 2 px, 2 p y and 2 p z orbitals of atom have identical
(d) Statement I is incorrect; Statement II is correct
shapes but differ in their ........... . (1993, 1M)
35. Statement I The first ionisation energy of Be is greater than
that of B. 45. When there are two electrons in the same orbital, they have
spins. (1983, 1M)
Statement II 2p-orbital is lower in energy than 2s. (2000)

True/False
Passage Based Problems
46. In a given electric field, b-particles are deflected more
The hydrogen-like species Li2+ is in a spherically symmetric
than a-particles in spite of a-particles having larger
state S 1 with one radial node. Upon absorbing light the ion
charge. (1993, 1M)
undergoes transition to a state S 2 . The state S 2 has one radial
node and its energy is equal to the ground state energy of the 47. The electron density in the XY-plane in 3d x 2 - y2
orbital is
hydrogen atom. zero. (1986, 1M)
36. The state S 1 is (2010) 48. The energy of the electron in the 3d-orbital is less than that
(a) 1s (b) 2s in the 4s-orbital in the hydrogen atom. (1983, 1M)
(c) 2p (d) 3s
49. Gamma rays are electromagnetic radiations of wavelengths
37. Energy of the state S 1 in units of the hydrogen atom ground of 10-6 to 10-5 cm. (1983, 1M)
state energy is (2010)
(a) 0.75 (b) 1.50 50. The outer electronic configuration of the ground state
(c) 2.25 (d) 4.50 chromium atom is 3d 4 4 s2 . (1982, 1M)
Atomic Structure 27

Integer Answer Type Questions 61. Calculate the wave number for the shortest wavelength
transition in the Balmer series of atomic hydrogen.
51. In an atom, the total number of electrons having quantum (1996, 1M)
numbers (2014 Adv.)
1 62. Iodine molecule dissociates into atoms after absorbing
n = 4, | ml | = 1and ms = - is light to 4500. If one quantum of radiation is absorbed by
2
each molecule, calculate the kinetic energy of iodine
52. The atomic masses of He and Ne are 4 and 20 amu, atoms.
respectively. The value of the de-Broglie wavelength of He gas
(Bond energy of I2 = 240 kJ mol 1 ) (1995, 2M)
at -73C is M times that of the de-Broglie wavelength of Ne
at 727C. M is (2013 Adv.) 63. Find out the number of waves made by a Bohrs electron in
53. The work function (f) of some metals is listed below. The one complete revolution in its 3rd orbit. (1994, 3M)
number of metals which will show photoelectric effect when 64. What transition in the hydrogen spectrum would have the
light of 300 nm wavelength falls on the metal is (2011) same wavelength as the Balmer transition n = 4 to n = 2 of
Metal Li Na K Mg Cu Ag Fe Pt W He+ spectrum? (1993, 3M)
F (eV) 2.4 2.3 2.2 3.7 4.8 4.3 4.7 6.3 4.75 65. Estimate the difference in energy between 1st and 2nd
Bohrs orbit for a hydrogen atom. At what minimum
54. The maximum number of electrons that can have principal
atomic number, a transition from n = 2 to n = 1energy level
quantum number, n = 3and spin quantum number, ms = -1 / 2 ,
would result in the emission of X-rays with
is (2011)
l = 3.0 108 m ? Which hydrogen atom-like species does
Subjective Questions this atomic number correspond to? (1993, 5M)

55. (a) Calculate velocity of electron in first Bohr orbit of hydrogen 66. According to Bohrs theory, the electronic energy of
atom (Given, r = a0 ). hydrogen atom in the nth Bohrs orbit is given by :
(b) Find de-Broglie wavelength of the electron in first Bohr orbit. -21.7 10-19
En = J
(c) Find the orbital angular momentum of 2p-orbital in terms of n2
h / 2p units. (2005, 2M) Calculate the longest wavelength of electron from the third
56. (a) The Schrodinger wave equation for hydrogen atom is Bohrs orbit of the He+ ion. (1990, 3M)
3/ 2
1 1 r 67. What is the maximum number of electrons that may be
y 2s = 2 - e-r / 2a 0
4 (2p )1/ 2 a0 a0 present in all the atomic orbitals with principal quantum
number 3 and azimuthal quantum number 2 ? (1985, 2M)
where, a0 is Bohrs radius. Let the radial node in 2s be at r0.
Then, find r in terms of a0. 68. Give reason why the ground state outermost electronic
(b) A base ball having mass 100 g moves with velocity configuration of silicon is (1985, 2M)
100 m/s. Find out the value of wavelength of base ball. 3s 3p 3s 3p
(2004, 2M) and not
57. The wavelength corresponding to maximum energy for
hydrogen is 91.2 nm. Find the corresponding wavelength for 69. The electron energy in hydrogen atom is given by
He+ ion. (2003, 2M) 21.7 10-12
En = - erg. Calculate the energy required to
58. Calculate the energy required to excite 1 L of hydrogen gas at n2
1 atm and 298 K to the first excited state of atomic hydrogen. remove an electron completely from the n = 2 orbit. What
The energy for the dissociation of HH bond is 436 kJ mol -1 . is the longest wavelength (in cm) of light that can be used to
(2000) cause this transition? (1984, 3M)

59. An electron beam can undergo diffraction by crystals. Through 70. Calculate the wavelength in Angstroms of the photon that
what potential should a beam of electrons be accelerated so is emitted when an electron in the Bohrs orbit, n = 2
that its wavelength becomes equal to 1.54 . (1997 (C), 2M) returns to the orbit, n = 1 in the hydrogen atom. The
ionisation potential of the ground state hydrogen atom is
60. Consider the hydrogen atom to be proton embedded in a cavity
2.17 10-11 erg per atom. (1982, 4M)
of radius a0 (Bohrs radius) whose charge is neutralised by the
addition of an electron to the cavity in vacuum, infinitely 71. The energy of the electron in the second and third Bohrs
slowly. Estimate the average total energy of an electron in its orbits of the hydrogen atom is - 5.42 10-12 erg and
ground state in a hydrogen atom as the work done in the above - 2.41 10-12 erg respectively. Calculate the wavelength
neutralisation process. Also, if the magnitude of the average
of the emitted light when the electron drops from the third
kinetic energy is half the magnitude of the average potential
to the second orbit. (1981, 3M)
energy, find the average potential energy. (1996, 2M)
28 Atomic Structure

Topic 1 21. (a) 22. (c) 23. (b) 24. (c)


1. (d) 2. (b) 3. (b) 4. (b) 25. (d) 26. (c) 27. (b) 28. (d)
5. (d) 6. (a) 7. (c) 8. (a) 29. (a) 30. (a) 31. (d) 32. (a,d)
9. (a,b) 10. (d) 11. (a,c) 12. (a,c) 33. (a,b,c) 34. (b,d) 35. (c) 36. (b)
13. A r; B q; C p, D s 14. (photons) 37. (c) 38. (b)
15. (1 . 66 10 27 kg) 16. (neutrons) 17. (isobars) 39. A q; B ,p q, r, s C p, q, r D p, q, r
40. Cr = [Ar] 3d 5 , 4s 1 41. 1 : 16
Topic 2
42. Heisenberg, de-Broglie. 43. orbital
1. (c) 2. (a) 3. (a) 4. (c)
44. Orientation in space 45. opposite
5. (a) 6. (b) 7. (c) 8. (d)
46. True 47. False
9. (c) 10. (a) 11. (b) 12. (a)
48. True 49. False 50. False 51. (6)
13. (a) 14. (a) 15. (b) 16. (d)
52. (5) 53. (4) 54. (9)
17. (b) 18. (d) 19. (c) 20. (c)

Topic 1 Preliminary Developments 10. The isotopes of hydrogen are 1 H2 and 1 H3 .


and Bohrs Model
11. Alpha particles passes mostly undeflected when sent through
1. Rutherford used a-particle (He2+ nuclei) in his experiment. thin metal foil mainly, because
2. According to Rutherfords model, there is a heavily positively (i) it is much heavier than electrons.
charged nucleus and negatively charged electrons occupies (ii) most part of atom is empty space.
space around it in order to maintain electro-neutrality. 12. Many elements have several isotopes. For such elements,
3. Radius of a nucleus is in the order of 10-13 cm, a fact. atomic mass is average of the atomic masses of different
isotopes, which is usually non-integral.
4. Bohrs model is applicable to one-electron system only.
5. Neutron has no charge, hence e / m is zero for neutron. Next, 1 Ze2
13. A. Vn = -
2+
a-particle (He ) has very high mass compared to proton 4 pe 0 r

and electron, therefore very small e / m ratio. Proton and Ze2
1 Vn
Kn =
electron have same charge (magnitude) but former is heavier,
8pe 0 r Kn
= - 2 (r )
hence has smaller value of e / m.
e Ze2
: n<a < p<e B. En = - r- 1 x =-1 (q )
m 8pe 0 r
6. The negligibly small size of nucleus compared to the size of h
C. Angular momentum = l ( l + 1) = 0 in 1s-orbital
atom was first established in Rutherfords experiment. 2p ( p ).
7. The most important findings of Rutherfords experiment is a0 n 2 1
discovery of nucleus. D. rn = Z (s )
Z rn
8. Energy of electron in H-atom is determined by the
14. Photons have quantised energy.
expression:
13.6 10-3
En = - eV where, n =1, 2, 3, .... 15. Mass of one H-atom = kg = 1.66 10-27 kg
n2 6.023 1023
13.6 16. Isotopes have different number of neutrons.
In excited states, E2 = - = - 3.4 eV
4
17. Isobars have same mass number but different atomic numbers.
13.6
E3 = - = - 1.51eVetc. 18. When a-particle stop at 10-13 m from nucleus, kinetic energy
9
is zero, i.e. whole of its kinetic energy at the starting point is
9. Nucleus is composed of neutrons and protons. now converted into potential energy.
Atomic Structure 29

Potential energy of this a-particle can be determined as Also, Bohrs radius for H-atom is, r = n 2 a0
Z1 Z2 e 2 Substituting r in Eq. (i) gives
PE = -
( 4 pe 0 ) r h2 h2
KE = 2 2 2 when n = 2 , KE =
8p n a0 m 32p2 a02 m
(Z1 = + 2, Z2 = + 29,
e 0 = 8.85 10-12 J-1 C2 m-1 , r =10-13 m) 5. The number of radial nodes is given by expression ( n - l -1).
2 29 (1.6 10-19 )2 For 3s, number of nodes = 3 - 0 - 1 = 2
| PE | = J
4 3.14 8.85 10 -12
10-13 For 2 p, number of nodes = 2 - 1 -1 = 0

= 1.33 10 -13
J a0 n 2
6. Expression for Bohrs orbit is, rn = = a0
Z
= kinetic energy of a-particle at t =0 when n = 2, Z = 4.
1
KE = mv2 = 1.33 10-13 7. 1s7 violate Pauli exclusion principle, according to which an
2 orbital cannot have more than two electrons.
2 1.33 10-13 1 1
v= = 6.3 106 ms -1 8. + and - just represents two quantum mechanical spin
4 1.66 10-27 2 2
states which have no classical analogue.
Topic 2 Advanced Concept 9. Using the de-Broglies relationship :
(Quantum Mechanical Theory) h 6.625 10-34
l= = = 2.3 10-30 m
Electronic Configuration and mv 0.2 5
Quantum Number 60 60
13.6
1. \ En = - eV where, n = 1, 2, 3 ... 10. Nodal plane is an imaginary plane on which probability of
n2 finding an electron is minimum. Every p-orbital has one
-13.6 nodal plane :
In excited states, E2 = = -3.4 eV
4
2. Given, atomic number of Rb, Z = 37 px
Thus, its electronic configuration is [ Kr ]5s1 . Since, the last
electron or valence electron enter in 5s subshell. YZ-plane, a nodal plane
So, the quantum numbers are n = 5, l = 0,(for s-orbital) m = 0
(Q m = + l to -l ), s = + 1 / 2 or - 1 / 2. 11. 1s2 2 s2 2 p 6 3s2 3 p 6 3d 5 4 s1 is ground state electronic
Z2 configuration of Cr.
3. Given, in the question E = - 2.178 10-18 J 2
n 12. (i) n = 4 , l = 1 4 p-orbital
(ii) n = 4 , l = 0 4s-orbital
For hydrogen Z = 1, (iii) n = 3, l = 2 3d-orbital
1 (iv) n = 3, l = 1 3d-orbital
So, E1 = - 2.178 10-18 J 2
1 According to Aufbau principle, energies of above mentioned
1 orbitals are in the order of
E2 = - 2.178 10-18 J 2
2 (iv) 3 p < (ii) 4s < (iii) 3d < (i) 4 p
Now, E1 - E2 13. The energy of an electron in a Bohr atom is expressed as
-18 1 1 hc
kZ 2 where, k = Constant,
i.e. DE = 2.178 10 2 - 2=
1 2 l En = - 2 Z = Atomic number,
n
1 6.62 10-34 3.0 108 n = Orbit number
1
2.178 10-18 2 - 2 = = - 13.6 eV for H ( n = 1)
1 2 l
- 13.6
\ l 1.21 10-7 m when n = 2 , E2 = eV = - 3.40 eV
22
4. According to Bohrs model, (n can have only integral value 1, 2, 3, )
nh n2h2 h
mvr = ( mv )2 = 14. The orbital angular momentum ( L ) = l ( l + 1)
2p 4 p2 r 2 2p
2 2
1 2 n h h
KE = mv = 2 2 (i) = 6 ( l = 2 for d -orbital )
2 8p r m 2p
30 Atomic Structure

15. Bohr first made use of quantum theory to explain the 29. The valence shell configuration of Rubidium (Rb) is
structure of atom and proposed that energy of electron in an 1 1
[ Kr ] 5s1 n = 5, l = 0, m = 0, s= + or -
atom is quantised. 2 2
16. Mg 2 + = 1s2 2s2 2 p 6 no unpaired electron 30. The principal quantum number n represents orbit number
Ti 3+ 2 2 6 2 6 1
= 1s 2 s 2 p 3s 3 p 3d one unpaired electron hence, determine the size of orbitals.
31. According to Pauli exclusion principle, an atomic orbital can
V3+ = 1s2 2 s2 2 p 6 3s2 3 p 6 3d 2 two unpaired electrons
accommodate at the most, two electrons, with opposite spins.
Fe2+ =1s2 2 s2 2 p 6 3s2 3 p 6 3d 6 four unpaired electrons 32. Both (a) and (d) are correct. The three electrons in the
17. Expression for orbital angular momentum (L) is 2 p-orbitals must have same spin, no matter up spin or down
h spin.
L = l ( l + 1) = 0 for 2s-electrons
2p 33. (a) Cr = [Ar] 3d 5 4 s1 , an exception to aufbau principle.
Q For s-orbital, l =0. (b) For a given value of l, m can have any value from
2 (-l to +l ), so can have negative value.
18. Diffraction is property of wave, E = mc determine energy of
(c) Ag is in copper group with d 10 s1 configuration,
particle and E = hn determine energy of photon. Interference
i.e. 46 electrons are spin paired.
phenomena is exhibited by both matter and waves.
34. Isotones have same number of neutrons.
19. X-rays is electrically neutral, not deflected in electric or 76
magnetic fields. 32 Ge , 33 As 77 and 34 Se78 have same number (44) of
20. Cl (17) =1s2 2 s2 2 p 6 3s2 3 p 5 neutrons, hence they are isotones.
35. Assertion is correct Be (1s2 , 2s2 ) has stable electronic
configuration, removing an electron require more energy
The last, unpaired electron has, n = 3, l = 1( p ) and m can than the same for B( 2 p1 ). Reason is incorrect
have any of the three value (-1, 0, + 1). (Aufbau principle).
21. Cr (24) =1s2 2 s2 2 p 6 3s2 3 p 6 3d 5 4 s1 36. S 1 is spherically symmetrical state, i.e. it correspond to a
144
424443
Ar s-orbital. Also, it has one radial node.
The above configuration is exception to Aufbaus principle. Number of radial nodes = n - l - l
n - 0 -1=1
22. Fluorine, a halogen, is the most electronegative atom, has the
electronic configuration 2s2 2 p 5 (valence shell). n = 2 i.e. S 1 = 2s-orbital.

23. Option (b) is wrong representation according to aufbau 37. Ground state energy of electron in H-atom ( EH )
principle. A high energy atomic orbital ( 2 p ) cannot be filled kZ 2
unless the low energy orbital ( 2s ) is completely occupied. EH = = k ( Z = 1, n = 1)
n2
1 1 hc
24. Transition energy ( DE ) = kZ 2 - = For S 1 state of Li 2+ ,
n12 n22 l
k ( 3 )2 9
1 E= = k = 2.25 k
i.e. DE 22 4
l
38. In S 2 state, E ( Li 2+ ) = K (given)
hc E l
25. E= 1 = 2 =2 qk
l E2 l1 K =
n2
26. n l m s n=3
1 Since, S 2 has one radial node.
3 2 -3
2 3 - l -1 = 1
This is the wrong set of quantum number because | m | cannot l =1
be greater than l.
39. A. Orbital angular momentum
27. The wavelength order is h
( L ) = l ( l + 1)
X-ray < ultraviolet < infrared < radio wave 2p
28. When electron jumps to lower orbit photons are emitted i.e. L depends on azimuthal quantum number only.
while photons are absorbed when electron jumps to higher B. To describe a one electron wave function, three quantum
orbit. 1s-orbital is the lower most, electron in this orbital can numbers n, l and m are needed. Further to abide by Pauli
absorb photons but cannot emit. exclusion principle, spin quantum number(s) is also needed.
Atomic Structure 31

C. For shape, size and orientation, only n, l and m are needed. 53. Energy of photon
D. Probability density (y 2 ) can be determined if n, l and m are hc hc 6.625 1034 3 108
known. = J = eV = = 4 . 14 eV
l el 300 109 1.602 1019
40. Cr = [Ar] 3d 5 4 s1 For photoelectric effect to occur, energy of incident photons
41. 1 : 16 must be greater than work function of metal. Hence, only Li,
Na, K and Mg have work functions less than 4.14 V.
42. Heisenberg proposed uncertainty principle and de-Broglie
proposed wave nature of electron. 54. When n = 3, l = 0, 1, 2 i.e. there are 3s, 3p and 3d-orbitals. If
all these orbitals are completely occupied as
43. orbital
44. 2 px ,2 p y and 2 p z have different orientation in space.
45. Two electrons in same orbital must have opposite spin. 1
Total 18 electrons, 9 electrons with s = + and 9 with
21
46. Very large mass of alpha particles than beta particles is s= .
responsible for less deflection in former case. 2
Alternatively In any nth orbit, there can be a maximum of
47. 3d x 2 - y 2 orbital lies in XY -plane. 2n2 electrons. Hence, when n = 3, number of maximum
48. Aufbau principle. 1
electrons = 18. Out of these 18 electrons, 9 can have spin
49. This is the wavelength of infrared radiation. 1 2
and remaining nine with spin + .
50. Cr = 3d 5 4 s1 . 2
nh
55. (a) mvr =
51. PLAN This problem is based on concept of quantum number. Follow 2p
the following steps to solve this problem.
Write all possible orbitals having combination of same principal, nh 6.625 10-34
v= =
azimuthal, magnetic and spin quantum number. 2pmr 2 3.14 9.1 10-31 0.529 10-10
Then count the all possible electrons having given set of
quantum numbers. = 2.18 106 ms - 1
For n = 4, the total number of possible orbitals are h 6.625 10-34
(b) l = = = 0.33 10-9 m
4s 4p 4d 4f
mv 9.1 10-31 2.18 106
0 1 0 +1 2 1 0 +1 +2 3 2 1 0 +1 +2 +3 (c) Orbital angular momentum
h h
According to question | m l | = 1, i.e. there are two possible ( L ) = l ( l + 1) = 2
values of m l , i.e. +1 and 1 and one orbital can contain
2p 2p
1 [Q For p-orbital, l = 1]
maximum two electrons one having s = + and other having
2 56. (a) At radial node, y 2 must vanishes, i.e.
s = -1 / 2 . 2 2 r0
So, total number of orbitals having {| m l | = 1} = 6 1 r0 - a 0
y 22 s = 0 = 2 - e
Total number of electrons having 4 2p a0
1 r0
{| m l | = 1and ms = - } = 6 2- = 0 r0 = 2a0
2 a0
52. PLAN KE = 1 mv 2 = 3 RT h 6.625 10-34
2 2 (b) l= = = 6.625 10-35 m
mv 100 10-3 100
\ m2 v2 = 2mKE \ mv = 2mKE
h h h = 6.625 10-25 (negligibly small)
l (wavelength) = =
mv 2mKE 2m(T ) 57. The general Rydbergs equation is
where, T = Temperature in Kelvin 1 1 1
n= = R ( Z )2 2 - 2
h l
l(He at -73 C = 200 K ) = n1 n2
2 4 200 1
h Z2
l (Ne at 727C = 1000 K ) = l
2 20 1000 l ( He+ ) Z ( H )2 1
= + 2
=
l( He ) 2 20 1000 l (H) Z ( He ) 4
\ =M = =5
l( Ne ) 2 4 200 l ( H ) 91.2
l ( He+ ) = = nm = 22.8 nm
Thus, M =5 4 4
32 Atomic Structure

pV 1 1 61. The Rydbergs equation for H-atom is


58. Moles of H2 = = = 0.0409
RT 0.082 298 1 1 1
= n (wave number) = RH 2 - 2
Bond energy = 0.0409 436 = 17.84 kJ l n1 n2
Number of H-atoms produced after dissociation
For Balmer series, n1 = 2 and n2 = 3, 4 , 5, ...,
= 2 0.0409 6.023 1023 = 4.93 1022
For shortest l, n2 has to be maximum, i.e. infinity. Then
-18 1 1 1
Transition energy/atom = 2.18 10 1 - J n = RH -
4 4
3
2.18 10-18 J
=
4 RH 1.09 107
= = = 2.725 106 m-1
Total transition energy 4 4
3
= 2.18 10-18 4.93 1022 J 62. After breaking of the bond of I2 molecule, the remaining
4 energy would be distributed uniformly to iodine atoms as
= 80.60 103 J = 80.60 kJ their kinetic energy, i.e.
Therefore, total energy required E (energy of photon) = Bond energy + 2 kinetic energy
= dissociation energy + transition energy 6.625 10-34 3 108 240 103
= + 2 Ek
= (17.84 + 80.60 ) kJ = 98.44 kJ 4500 10-10 6.023 1023
59. If accelerated by potential difference of V volt, then Ek = 2.16 1020 J/atom
1 2 63. The Bohr de-Broglie relationship is
mv = eV
2 2pr = nl = circumference of Bohrs orbit.
p2 i.e. number of complete waves formed in one complete
= eV , here p = momentum ( mv )
2m revolution of electron in any Bohr orbit is equal to orbit
h h number, hence three.
Using de-Broglie equation, l = =
p 2meV 64. The expression for transition wavelength is given by
6.625 10-34 Rydbergs equation :
1.54 10-10 = -31 -19
( 2 9.1 10 1.6 10 1/ 2
V) 1 1 1
= RH Z 2 2 - 2
Solving for V gives : V = 63.56 V. l n1 n2
60. The work done in the given neutralisation process is Equating the transition wavelengths of H-atom and He+
ion,
W =- F dr
a0
1 1 4 4
e2 RH 2 - 2 = RH 2 - 2
and F = n
1 n2 2 4
4pe 0 r2
Equating termwise on left to right of the above equation

e 2 1 e2 gives
W = = - = Total energy ( E )
4 pe 0 r a 0 4 pe 0 r n1 = 1 and n2 = 2
Now, if V is magnitude of potential energy, then according to 65. For H-atom, the energy of a stationary orbit is determined
given information, kinetic energy ( Ek ) isV / 2. Therefore, as
V k
E = -V + (PE is always negative) En = - where, k = constant ( 2.18 10-18 J )
2 n2
V 1 3
=- DE ( n = 2 to n = 1) = k 1 - = k
2 4 4
-e 2 = 1.635 10-18 J
V = - 2E =
2pe 0 r
Atomic Structure 33

For a H-like species, energy of stationary orbit is determined 68. The 2nd configuration is against Hunds rule of maximum
as multiplicity which states that the singly occupied degenerate
kZ 2 atomic orbitals must have electrons of like spins.
En = -
n2 69. The required transition is n1 = 2 to n2 = and corresponding
where, Z = atomic number transition energy is
1 1 1 1 21.7
DE = kZ 2 2 - 2 DE = 21.7 10-12 2 - 2 erg = 10-12 erg
n1 n2 n
1 n 2 4
1 DE k 2 1 1 3 = 5.425 10-12 erg
= = Z - = RH Z 2
l hc hc 1 4 4 The longest wavelength that can cause above transition can
2 4 4 be determined as :
Z = = = 4.05
3RH l 3 1.097 107 3 10-8
hc 6.625 10-34 3 108
+ l= =
Z = 2 (He ) DE 5.425 10-12 10-7
= 3.66 10-7 m = 3.66 10-5 cm
66. For H-like species, the energy of stationary orbit is expressed
as 70. Ionisation potential of H-like species
= E1 = 2.17 10-11 erg
E ( X ) = Z2 E (H )
1
For He+ ( Z = 2 ) DE = 2.17 10-11 1 - 2 10-7 J
2
4 21.7 10-19
E =- J -18 hc
n2 = 1.6275 10 J l =
DE
For longest wavelength transition from 3rd orbit, electron
6.625 10-34 3 108
must jump to 4th orbit and the transition energy can be = m
determined as 1.6275 10-18
1 1 = 122 10-9 m = 1220
DE = + 4 21.7 10-19 - J = 4.22 10-19 J
9 16 71. Transition energy = [ - 2.41 - ( - 5.42 )] 10-12 erg
hc
Also, Q DE = = 3.01 10-12 erg
l
hc 6.625 10-34 3 108 = 3.01 10-19 J [Q 1erg = 10-7 J ]
\ l= = m hc
DE 4.22 10-19 Also, DE =
l
= 471 10-9 m = 471 nm 6.625 10-34 3 108
l= m
67. Ten, the given value of n and l correspond to 3d-orbital 3.01 10-19
which has five fold degeneracy level.
= 660 10-9 m = 660 nm
CHAPTER TEST
1. Which of the following statements is incorrect? spectrum containing 15 bright lines. Out of these 15 lines,
(a) When hydrogen atoms were excited, a line spectrum 9 lines were found to have wavelengths smaller than
rather than a continuous spectrum was observed 121 nm while 5 lines have wavelengths greater than
(b) It was assumed that electrons in the hydrogen atoms 121 nm. Based on the above information, answer the
were allowed to have only discrete amounts of energy following three questions.
(c) In Balmer series, the emitted light falls in the visible 6. What is the value of ni ?
region of spectrum in which the electron's final energy
level is 2 (a) 2 (b) 3 (c) 4 (d) 5
(d) Einstein postulated that light behaved as a particle
7. What is the ionisation energy of the given species in
called a photon, and the photoelectric effect was used
to support this postulation eV unit ?
(a) 13.6 (b) 54.4 (c)122.4 (d) 217.6
2. Which of the following is not a legal set of quantum
numbers for an electron in an atom? 8. Which electronic transition correspond to the
(a) n = 5, l = 3, ml = 2, m s = 1/2 emission of 2nd longest wavelength?
(b) n = 3, l = 2 , ml = 3, m s = - 1/2 (a) 6 5 (b) 6 4 (c) 5 4 (d) 4 3
(c) n = 1, l = 1, ml = 0, m s = - 1/2
(d) n = 6, l = 3, ml = - 3, m s = 1/2 Assertion-Reason Type
Following two questions have assertion followed by the
3. The first emission line in visible region in the emission reason. Answer them according to the following
spectrum of He+ ion would correspond to which of the options.
following electronic transition? (a) Both assertion and reason are correct and reason is
(a) 6 3 (b) 7 4 (c) 5 4 (d) 6 4 the correct explanation of the assertion
(b) Both assertion and reason are correct but reason is
4. Consider the following radial distribution function not the correct explanation of assertion
diagrams. Which of the following has the correct (c) Assertion is correct but reason is incorrect
matching of curve and orbital? (d) Assertion is incorrect but reason is correct

II III 9. Assertion In H-atom, photons of higher intensity is


emitted when electron falls from 2nd to 1st orbit than
when it falls from 4th to 2nd orbit.
I Reason Transition energy for 2nd to 1st orbit is
4pr2R(Y)2

greater than for 4th to 2nd orbit.


10. Assertion Emission spectrum of H-atom is a line
B
spectrum rather a continuous spectrum.
Reason Emission spectrum of every pure atom is
always a line spectrum.
r (distance from nucleus)
(a) I (3s), II (3p), III (3d) (b) I (3d), II (3p), III (3s) 11. Match the statements of Column I with values of
(c) I (3p), II (3d), III (3s) (d) I (3s), II (3d), III (3p) Column II.
5. A one electron system has its electron revolving in the Column I Column II
3rd orbit. The light of maximum wavelength which can A. n=6 p. Ionisation energy is greater
eject the electron from the third orbit has energy of than 0.75 eV. [ionisation
6.04 eV. Which of the following statement regarding energy of hydrogen atom
the above mentioned species is (are) true? when in ground state is 13.6
(a) The ionisation energy of the species is 54.36 eV eV]
(b) If the electron falls to second Bohr orbit, visible light B. n=5 q. Has radius greater than the
would be emitted 5th Bohrs radius of He+.
(c) The atomic number is 2 C. n=4 r. Has more than six lines.
(d) A visible light may bring about transition from 4th to
D. n=3 s. Has two or more lines in the
higher orbit
visible region.
Passage for Q.Nos. 6 to 8 12. In how many different ways, the electrons of the
A one electron species initially in the some excited state outermost orbital of carbon can be represented in box
( n i ) is irradiated with a light of wavelength 121 nm when
the electron is promoted to a further higher orbit ( n f ) . In diagram format such that they all corresponds to the
returning back to the ground state, it gives an emission ground state electronic configuration?

Answers
1. (c) 2. (b) 3. (d) 4. (c) 5. (a, c, d) 6. (b) 7. (c) 8. (c) 9. (d) 10. (b)
11. A q,r,s; B q,r,s; C p,q,s; D p 12. (6)
3
Periodic Classification and
Periodic Properties
History and Periodic Classification Periodic Properties
l
Mendeleef s Periodic Law According to Mendeleef, l
Metallic and Non-metallic Character
The physical and chemical properties of the elements are a (i) The tendency of an element to loose electrons and form
periodic function of their atomic weights. He arranged the positive ions is called electropositive character or
elements in increasing order of atomic weights into groups metallic character, e.g. alkali metals are the most
and periods. electropositive elements.
(ii) The tendency of an element to accept electrons to form an
l
Drawbacks of Mendeleef s Periodic Table anion is called non-metallic character or electronegative
Anomaly is found in three pair of elements. Tellurium (Te) character, e.g. chlorine, oxygen, nitrogen are all
and iodine (I), argon (Ar) and potassium (K), nickel (Ni) and electronegative elements and they are called non-metals.
cobalt (Co) which were not in accordance of their chemical (iii) In a period, metallic character of elements decreases from
behaviour. left to right, e.g. in 3rd period, Na on extreme left is most
l
Moseleys Law and Long-Form (Modern) Periodic metallic and Cl on extreme right is most electronegative.
Table From his X-ray diffraction experiments on a number (iv) In a group metallic character increases from top to
of elements, Moseley proposed that The physical and bottom, e.g. in group 14, element at the top (carbon) is
chemical properties of the elements are a periodic function of non-metal while element at the bottom (lead) is metal.
their atomic number or their electronic configuration. l
Atomic Size and Atomic Radii The distance from
l
Characteristic Features of Long Form of Periodic Table centre of the nucleus to the outermost shell is called radius of
an atom. Atomic radius, therefore, depends on two important
(i) The modern periodic table is divided into two main factors : (i) The orbit number of outermost shell and
categories known as (ii) On effective nuclear charge.
(a) vertical columnsgroups and (a) Effective nuclear charge increases from left to right
(b) horizontal rowsperiods. in a period and therefore, atomic radius decreases
(ii) There are 18 groups. These groups are further from left to right in a given period.
subdivided into A-groups and B-groups. (b) Orbit number in which the last electron enters
(iii) Members of the same group have same valence shell increases successively from top to bottom in a group,
configurations. hence atomic radius increases from top to bottom in a
(iv) Elements of group IA to VIIA (1, 2, 13, 14, 15, 16, 17 group.
groups) are known as representative elements. Members l
Covalent Radius and van der Waals Radius
of group IB to VIIIB (3, 4, 5, 6, 7, 8, 9, 10, 11 and 12) are Covalent radius is defined in covalently bonded molecules as :
known as transition elements. A A B
A
(v) Elements of group 18 (also known as zero group) are
noble gases.
(vi) There are seven horizontal rows in the periodic table,
known as periods.
(vii) In a period, number of valence shell remains the same, d d
however the number of electrons increases from left to Covalent rad =
d d = rA + rB
right. 2
36 Periodic Classification and Periodic Properties

(i) In case of homonuclear diatomic molecule, covalent Remember


radius is half of their internuclear distance. For a group of isoelectronic species involving cation,
(ii) In case of heteronuclear diatomic molecules, covalent anion and neutral atom, radius is inversely proportional
radius is defined as : to Z (atomic number).
AB : Internuclear distance = rA + rB e.g. N3 + > O2 - > F- > Na + > Mg 2 +
where, rA and rB are covalent radii of A and B and if any (Order of radius)
one of them is known, other can be known. l
Ionisation Potential Ionisation potential (or ionisation
(iii) van der Waals radius is defined as the half of the energy) is the amount of energy required to remove one or
internuclear distance between two adjacent, more electrons from the outermost shell of an isolated
non-bonded, like atoms. These types of radii are shown gaseous atom in their ground state. Ionisation energy is also
below in the diagram : known as ionisation potential because it is measured as the
A A A A
minimum potential difference required to remove the most
loosely held electrons from the rest of the atom. It is
measured in eV unit per atom or kJ per mole. Following
trends are observed for ionisation energy in periodic table :
(i) Metals usually have low ionisation energy whereas
d
non-metals have high ionisation energies. Inert gases
van der Waals radius = d have maximum ionisation energy in its period.
2
As shown above, for a given element, covalent radius is (ii) In a period ionisation energy increases from left to right.
always less than its van der Waals radius. Elements of In a group ionisation energy decreases from top to
18th group do not form covalent bond therefore, for them bottom.
the van der Waals radius is the only radius and it is (iii) In a period, ionisation energy does not vary uniformly.
always greater than the covalent radius of any elements Stability of electronic configuration alter the regular
of the same period. trends between adjacent elements in a period. Although
B comes after Be in period, Be has higher ionisation
l
Ionic Radius energy.
(a) When a cation is formed, effective nuclear charge 2s2
number of protons Removal of electron is
increases resulting a decrease in Be =1s2 easier from 2p-orbitals
2p
number of electrons than from completely
radius than the radius of normal atom. B = 1s2 2s2 filled, stable 2s-orbital.
(b) When an anion is formed, effective nuclear charge Removal of electron is
decreases, resulting an increase in radius than the radius N =1s2 2s2
difficult from completely
of normal atom. half-filled orbitals of
O =1s2 2s2
Comparison of cationic and anionic radius N than from oxygen.

Cationic radius Anionic radius


Similarly, N has higher ionisation energy than O because
N has completely half-filled, stable valence shell
Smaller than the radius of Larger than the radius of configuration :
corresponding neutral atom. corresponding neutral
However this anomaly applies only between adjacent
e.g. Mg > Mg 2 + atom.
elements in a period. Therefore, elements of G-2 has
higher ionisation energy than elements of G-13 but of the
For same atom, cationic radius For same atom, anionic
same period. Similarly, elements of G-15 has higher
is inversely proportional to the radius is directly
ionisation energy than ionisation energy of G-16
magnitude of positive charge. proportional to the elements from the same period.
e.g. Fe > Fe2 + > Fe3 + magnitude of negative
(iv) For a given element, values of successive ionisation
charge. e.g.
energy increases, i.e. removal of an electron further
O2- > O- > O require more energy than the energy required for
For isoelectronic cationic For isoelectronic species, removal of previous electron, i.e. for an element :
radius
1 1 IE1 < IE2 < IE3 .....
anionic radius
Z Z l
Electron Affinity Electron affinity is the amount of
+ 2+ 3+ 3- 2- -
e.g. Na > Mg > Al e.g. N > O >F energy released when an electron is added to an isolated,
Periodic Classification and Periodic Properties 37

neutral gaseous atom in the ground state. Electron affinity compound. The value of electronegativity of an element
of an element depends on : (a) Extent of nuclear charge, i.e. describes the ability of its atom to complete for electrons with
effective nuclear charge. (b) Size of the atom. (c) Stability the other atom to which it is bonded. Electronegativity is
of valence shell electronic configuration. however not the property of an isolated atom. In general,
If electron affinity is low, the electron is weakly bound (i) Electronegativity increases from left to right in a period,
to the nucleus and if the electron affinity is high, ending at group 17.
electron is strongly bound to the nucleus, e.g. electron (ii) In a group electronegativity decreases from top to bottom.
affinity of chlorine is 3.79 eV which is higher than that The following Scales are used to measured
of iodine, i.e. 3.28 eV. Periodic trend of electron electronegativity :
affinity is : (i) Pauling Scale The electronegativity value for other
(i) In a period, it increases from left to right. Exception element are calculated as follows for a covalent bond
occurs in case of C and N. Carbon has greater electron between A and B
affinity than nitrogen because later has stable valence c A - c B - 0.208 D
shell configuration :
where, c A and c B are electronegativities of A and B
2p3
, has less affinity for electron.
D = E A - B - A A - A EB - B
N=
2p2 (ii) Mulliken Scale According to this scale :
, has greater affinity for electron Ionisation enthalpy + Electron gain enthalpy
C=
than nitrogen. c=
2
(ii) In a group, electron affinity decreases from top to l
Inert Pair Effect It is an effect, observed mainly in groups
bottom. Exception occur between fluorine and 13, 14 and 15. According to this effect, as we descend down in
chlorine. Fluorine, due to smaller size and high a group, the two electrons of s-orbital of the valence shell
electron density, has slightly smaller electron affnity becomes inert, do not easily take part in bonding and the
than chlorine. element show its oxidation state two unit less than the group
(iii) Addition of 2nd electron is always an endothermic oxidation number. Let us take the case of group 13. Al has +3,
process. Ga has +3 as their stable oxidation state. In show both +1 and
(iv) Noble gases have positive electron gain enthalpy, i.e. +3 oxidation state but Tl has only +1 as their stable oxidation
energy is required to add an electron into their valence state, because the two s-electrons of ns2 np1 have become inert.
shell. Inert pair effect is the result of increase in effective nuclear
(v) Be, N also have positive electron gain enthalpy due to charge on moving down in a group. Similar, phenomena is
their stable electronic configuration. observed in group 14. Si, Ge show their usual oxidation state of
l
Electronegativity Electronegativity is the tendency of + 4, Sn has both + 2 and + 4 oxidation state but lead has + 2 as
an atom to attract electron towards itself in a molecule or their most stable oxidation state.

Topic 1 History and Periodic Classification


Objective Questions I (Only one correct option) Objective Questions II
1. The statement that is not correct for the periodic (One or more than one correct option)
classification of elements, is (1992, 1M) 2. The statements that is/are true for the long form of the periodic
(a) the properties of elements are the periodic functions of their table is/are (1988, 1M)
atomic numbers (a) it reflects the sequence of filling the electrons in the order of
(b) non-metallic elements are lesser in number than metallic sub-energy level s, p, d and f
elements (b) it helps to predict the stable valency states of the elements
(c) the first ionisation energies of elements along a period do not (c) it reflects tends in physical and chemical properties of the
vary in a regular manner with increase in atomic number elements
(d) for transition elements the d-subshells are filled with (d) it helps to predict the relative ionicity of the bond between any
electrons monotonically with increase in atomic number two elements
38 Periodic Classification and Periodic Properties

Topic 2 Periodic Properties


Objective Questions I (Only one correct option) 13. Amongst the following elements (whose electronic
- configurations are given below), the one having the highest
1. The ionic radii (in ) of N ,O 3- 2-
and F respectively are
(2015 Main) ionisation energy is (1990, 1M)
(a) 1.36, 1.40 and 1.71 (b) 1.36, 1.71 and 1.40 (a) [Ne] 3s2 3 p1 (b) [Ne] 3s2 3 p3
(c) 1.71, 1.40 and 1.36 (d) 1.71, 1.36 and 1.40
(c) [Ne] 3s2 3 p2 (d) [Ar] 3d 10 4s2 4 p3
2. Which one of the following alkaline earth metal sulphates
has its hydration enthalpy greater than its lattice enthalpy? 14. Which one of the following is the smallest in size?
(a) CaSO4 (b) BeSO4 (2015 Main) (a) N3- (b) O2- (1989, 1M)
- +
(c) BaSO4 (d) SrSO4 (c) F (d) Na
3. Which among the following is the most reactive? 15. The first ionisation potential of Na, Mg, Al and Si are in the
(a) Cl 2 (b) Br2 (2015 Main) order (1988, 1M)
(c) I2 (d) ICl (a) Na < Mg >Al < Si (b) Na > Mg > Al >Si
4. Which one has the highest boiling point? (c) Na < Mg <Al >Si (d) Na > Mg > Al <Si
(a) He (b) Ne (c) Kr (d) Xe 16. The electronegativity of the following elements increases in
5. The first ionisation potential of Na is 5.1 eV. The value of the order (1987, 1M)
electron gain enthalpy of Na + will be (2013 Main) (a) C, N, Si, P (b) N, Si, C, P
(a) - 2.55 eV (b) - 5.1 eV (c) Si, P, C, N (d) P, Si, N, C
(c) - 10.2 eV (d) + 2.55 eV 17. Atomic radii of fluorine and neon in Angstrom units are
6. Which of the following represents the correct order of respectively given by (1987, 1M)
increasing first ionisation enthalpy for Ca, Ba, S, Se and Ar? (a) 0.72, 1.60 (b) 1.60, 1.60
(2013 Main) (c) 0.72, 0.72 (d) None of these
(a) Ca < S < Ba < Se < Ar (b) S < Se < Ca < Ba < Ar 18. The first ionisation potential in electron volts of nitrogen and
(c) Ba < Ca < Se < S < Ar (d) Ca < Ba < S < Se < Ar oxygen atoms are respectively given by (1987, 1M)
7. Identify the least stable ion amongst the following. (a) 14.6, 13.6 (b) 13.6, 14.6
(a) Li + (b) Be- (2002, 3M) (c) 13.6, 13.6 (d) 14.6, 14.6
(c) B- (d) C - 19. The hydration energy of Mg 2+ is larger than that of
8. The set representing the correct order of first ionisation (a) Al 3+ (b) Na + (1984, 1M)
potential is (2001, 1M) (c) Be2+ (d) Mg3+
(a) K > Na > Li (b) Be > Mg > Ca 20. The element with the highest first ionisation potential is
(c) B > C > N (d) Ge > Si > C (a) boron (b) carbon (1982, 1M)
(c) nitrogen (d) oxygen
9. The correct order of radii is (2000, 1M) 21. The correct order of second ionisation potential of carbon,
(a) N < Be < B (b) F - < O2- < N3- nitrogen, oxygen and fluorine is (1981, 1M)
(c) Na < Li < K (d) Fe3+ < Fe 2+ < Fe4+ (a) C > N > O > F (b) O > N > F > C
(c) O > F > N > C (d) F > O > N > C
10. The incorrect statement among the following. (1997(C), 1M)
(a) The first ionisation potential of Al is less than the first Objective Questions II
ionisation potential of Mg
(b) The second ionisation potential of Mg is greater than the (One or more than one correct option)
second ionisation potential of Na 22. Ionic radii of (1999, 3M)
(c) The first ionisation potential of Na is less than the first (a) Ti 4+ < Mn 7 + (b) 35
Cl - < 37
Cl -
ionisation potential of Mg
(c) K + > Cl - (d) P 3+ > P 5+
(d) The third ionisation potential of Mg is greater than third
ionisation potential of Na 23. The first ionisation potential of nitrogen and oxygen atoms
11. Which of the following has the maximum number of are related as follows. (1989, 1M)
unpaired electrons ? (1996, 1M) (a) The ionisation potential of oxygen is less than the ionisation
(a) Mg2+ (b) Ti 3+ potential of nitrogen
(c) V 3+ (d) Fe2+ (b) The ionisation potential of nitrogen is greater than the
ionisation potential of oxygen
12. Which has most stable +2 oxidation state? (1995, 1M) (c) The two ionisation potential values are comparable
(a) Sn (b) Pb (c) Fe (d) Ag (d) The difference between the two ionisation potential is too large
Periodic Classification and Periodic Properties 39

24. Sodium sulphate is soluble in water whereas barium Fill in the Blanks
sulphate is sparingly soluble because (1989, 1M)
(a) the hydration energy of sodium sulphate is more than its 30. Compounds that formally contain Pb 4+ are easily reduced to Pb 2+ .
lattice energy The stability of the lower oxidation state is due to .(1997, 1M)
(b) the lattice energy of barium sulphate is more than its 31. Ca 2+ has a smaller ionic radius than K + because it has ............
hydration energy (1993, 1M
(c) the lattice energy has no role to play in solubility 32. On Mulliken scale, the average of ionisation potential and electron
(d) the hydration energy of sodium sulphate is less than its affinity is known as ................ (1985, 1M)
lattice energy
33. The energy released when an electron is added to a neutral gaseous
Assertion and Reason atom is called . (1982, 1M)

Read the following questions and answer as per the


direction given below : True/False
(a) Statement I is true; Statement II is true; Statement 34. The basic nature of the hydroxides of group 13 (III B) decreases
II is the correct explanation of Statement I. progressively down the group. (1993, 1M)
(b) Statement I is true; Statement II is true; Statement 35. The decreasing order of electron affinity of F, Cl, Br is F > Cl > Br.
II is not the correct explanation of Statement I. (1993, 1M)
(c) Statement I is true; Statement II is false.
36. In group IA of alkali metals, the ionisation potential decreases
(d) Statement I is false; Statement II is true.
down the group. Therefore, lithium is a poor reducing agent.
(1987, 1M)
25. Statement I Nitrogen and oxygen are the main
37. The softness of group IA metals increases down the group with
components in the atmosphere but these do not react to
increasing atomic number. (1986, 1M)
form oxides of nitrogen.
Statement II The reaction between nitrogen and
Subjective Questions
oxygen requires high temperature. (2015 Main)
38. Arrange the following ions in order of their increasing radii
26. Statement I Pb 4+ compounds are stronger oxidising
4+ Li + , Mg 2+ , K + , Al 3+ . (1997, 1M)
agents than Sn compounds.
Statement II The higher oxidation states for the 39. Compare qualitatively the first and second ionisation potentials of
group 14 elements are more stable for the heavier copper and zinc. Explain the observation. (1996, 2M
members of the group due to inert pair effect.
(2008, 3M)
40. Arrange the following as stated :
27. Statement I Band gap in germanium is small. Increasing order of ionic size N3 ,Na + , F- , O2 - , Mg 2+
Statement II The energy spread of each germanium (1991, 1M)
atomic energy level is infinitesimally small. 41. Explain the following :
(2007, 3M)
The first ionisation energy of carbon atom is greater than that of
28. Statement I The first ionisation energy of Be is greater boron atom whereas, the reverse is true for the second ionisation
than that of B. energy. (1989, 2M)
Statement II 2p-orbital is lower in energy than 2s. 42. Arrange the following in the order of their increasing size:
(2000, (S), 1M)
Cl - , S2 - , Ca 2+ , Ar (1986, 1M)
29. Statement I F-atom has a less negative electron
affinity than Cl-atom. 43. Arrange the following in order of their
Statement II Additional electrons are repelled more (i) decreasing ionic size Mg 2+ , O2 - , Na + ,F-
effectively by 3 p-electrons in Cl-atom than by (ii) increasing first ionisation energy Mg, Al, Si, Na
2 p-electrons in F-atom. (1998, 2M) (iii) increasing bond length F2 , N2 , Cl 2 , O2 (1985, 3M)

Topic 1 17. (a) 18. (a) 19. (b) 20. (c)


1. (d) 2. (b,c,d) 21. (c) 22. (d) 23. (a,b,c) 24. (a,b)
25. (a) 26. (c) 27. (c) 28. (c)
Topic 2
29. (c) 30. (inert pair effect)
1. (c) 2. (b) 3. (d) 4. (d)
31. (higher effective nuclear charge)
5. (b) 6. (c) 7. (b) 8. (b)
32. (electronegativity) 33. (electron affinity)
9. (b) 10. (b) 11. (d) 12. (b)
34. F 35. F 36. F 37. T
13. (b) 14. (d) 15. (a) 16. (c)
Topic 1 History and Periodic Classification On the other hand, rest of the metals, i.e Ca, Ba, Sr have
bigger size and thats why lattice energy is greater than
1. (a) Correct statement According to Moseleys law, the
properties of elements are the periodic function of their hydration energy.
atomic numbers. Time Saving Technique In the question of finding
(b) Correct statement The whole s-block, d-block, hydration energy only check the size of atom. Smaller sized
f -block and heavier p-block elements are metal. atom has more hydration energy. Thus, in this question Be is
(c) Correct statement Trend is not regular, Be has higher placed upper most in the group has lesser size and not
first ionisation energy than B, nitrogen has higher first comparable with the size of sulphates. Hence, BeSO4 is the
ionisation energy than oxygen. right response.
(d) Inccorrect statement d-subshells are not filled 3. Cl 2 , Br2 and I2 are homonuclear diatomic molecule in which
monotonically, regularity break at chromium and copper. electronegativity of the combining atoms is same, so they are
2. (a) Incorrect Electrons are not filled in sub-energy levels more stable and less reactive, whereas, I and Cl have
s, p , d and f in the same sequence. different electronegativities and bond between them are
(b) Correct Number of valence shell electrons usually polarised and reactive. Therefore, interhalogen compounds
determine the stable valency state of an element. are more reactive.
(c) Correct Physical and chemical properties of elements Time Saving Technique In this type of question of halogen,
are periodic function of atomic number which is the basis only go through the polarity of the molecules.
of modern, long form of periodic table. As we know, diatomic molecule does not have polarity but
(d) Correct Relative ionicity of the bond between any two molecules with dissimilar sizes have polarity resulting in
elements is function of electronegativity difference of the more reactivity.
bonded atoms which in turn has periodic trend in long
form of periodic table. 4. As we move down the group of noble gases, molecular mass
increases by which dipole produced for a moment and hence
London forces increases from He to Xe.
Topic 2 Periodic Properties
Therefore, more amount of energy is required to break these
1. Number of electrons in N3- , = 7 + 3 = 10 forces, thus boiling point also increases from He and Xe.
Number of electrons in O2- = 8 + 2 = 10
5. Na Na + + e- First IE
-
Number of electrons in F = 9 + 1 = 10
Na + + e- Na
Since, all the three species have each 10 electrons, hence they
are isoelectronic species. Electron gain enthalpy of Na + is reverse of (IE)
It is considered that, in case of isoelectronic species as the Because reaction is reverse so
negative charge increases, ionic radii increases and therefore DH ( eq ) = - 5.1 eV
the value of ionic radii are 6. Ionisation energy increases along a period from left to right
N3- = 1.71 (highest among the three) and decreases down a group. The position of given elements
O 2-
= 1.40 in the periodic table is as
- Group No. 2 16 18
F = 136
. (lowest among the three)
Time Saving Technique There is no need to mug up the Ca S Ar
radius values for different ions. This particular question can Ba Se
be solved through following time saving. Thus, the order of increasing DH IE1 is
Trick The charges on the ions indicate the size as Ba < Ca < Se < S < Ar
N3 - > O2 - > F- . Thus, you have to look for the option in
which the above trend is followed. Option(c) is the only one in 7. Be- is the least stable ion, Be (1s2 2s2 ) has stable electronic
which this trend is followed. Hence, it is the correct answer. configuration, addition of electron decreases stability.
2. As we move down the group, size of metal increases. Be has 8. In a group, ionisation energy decreases down the group
lower size while SO2- 4 has bigger size, thats why BeSO4 Be > Mg > Ca
breaks easily and lattice energy becomes smaller but due to
lower size of Be, water molecules are gathered around and 9. Among isoelectronic species, greater the negative charge,
hence hydration energy increases. greater the ionic size, hence F- < O2 - < N3 - .
Periodic Classification and Periodic Properties 41

10. (a) Correct statement In a period, element of 2nd group 20. Nitrogen has highest ionisation potential due to exceptional
has higher first ionisation potential than element of stability of its valence shell configuration mentioned in
group 13. question 21.
(b) Incorrect statement Mg + require less energy for
21. For second ionisation potential, electron will have to be
further ionisation than Na + because of noble gas removed from valence shell of the following ions:
configuration of Na + .
(c) Correct statement Ionisation energy increases from C+ (5e-) = 1s2 2s2
left to right in a period. 2p
(d) Correct statement Mg 2+ has noble gas configuration, N+ (6e-) = 1s2 2s2
require greater energy for further ionisation than Na 2+ . 2p
11. Mg 2+ =1s2 2 s2 2 p 6 = no unpaired electron O+ (7e-) = 1s2 2s2
3+ 2 2 6 2 6 1
Ti =1s 2 s 2 p 3s 3 p 3d = one unpaired electron 2p
V 3+ 2 2 6 2 6 2
=1s 2 s 2 p 3s 3 p 3d = two unpaired electrons F+ (8e-) = 1s2 2s2
Fe 2+ 2 2 6 2 6 6
=1s 2 s 2 p 3s 3 p 3d = four unpaired electrons 2p

12. Pb has most stable +2 oxidation state due to inert pair effect. In general, ionisation energy increases from left to right in a
2 3 period. However, exception occur between adjacent atoms in
13. [Ne] 3s 3 p has highest ionisation energy, periodic trend.
a period, greater amount energy is required for removal of
14. Among isoelectronic species, the relation in size is electron from completely half-filled or completely filled
cation < neutral < anion orbital than the same for adjacent atom with either less than
+ completely half-filled or less than completely filled orbital.
Hence, Na has smallest size.
Therefore, ionisation potential of O+ is greater than that of F+ .
15. Ionisation energy increases from left to right in a period. Also ionisation potential of N+ is greater than C+ but less than
However, exception occur between group 2 and group 13 both O+ and F+ (periodic trend). Hence, overall order is 2nd
elements on account of stability of electronic configuration IP : O > F > N > C.
of valence shell.
22. (a) Ti 4+ > Mn 7+ is the correct order of size due to lower
IE
positive charge on Ti 4+ .
Group 2 = > Group 13 = (b) 37 Cl - = 37 Cl - : Isotopes with same charge have same size
ns2 ns2 np1 because isotopes differ in compositions of nuclei which
do not affect the atomic/ionic radius.
The desired order is Na < Mg > Al < Si
(c) K + < Cl - is the correct order. Among isoelectronic
16. Electronegativity increases from left to right in a period and species, anion has greater size than cation.
decreases from top to bottom in a group. Variation is more
(d) P3+ > P5+ is the correct order. For the same elements,
rapid in group than in a period, hence the desired order is
Electronegativity : Si < P < C < N lower the positive charge, larger the ions.

17. Atomic radius of noble gases are greater than halogens of 23. (a) and (b) are infact the same statements and both are
correct. N has slightly greater ionisation energy than
same period, hence (a) is the correct answer.
oxygen which is against periodic trend. This exception is
18. First ionisation energy of oxygen is less than that of nitrogen due to completely half-filled ( 2 p 3 ) orbital in nitrogen that
on the ground of stability of valence shell configuration, makes ionisation slightly difficult than oxygen.
hence (a) is the correct answer. (c) Also correct : Although N has greater first ionisation
19. Hydration energy depends on charge of ion and ionic radius. potential than oxygen, two values of ionisation potentials
Higher the charge, greater the hydration energy. On the are comparable since they are adjacent in a period, i.e.
other hand, smaller the size, greater the hydration energy. electrons are removed from same orbit during ionisation.
Charge is considered first for comparison. Hence, Mg 2+ has (d) Incorrect opposite to (c). of the bonded atoms which in
higher hydration energy than Na + . turn has periodic trend in long form of periodic table.
42 Periodic Classification and Periodic Properties

24. (a) Correct For greater solubility, hydration energy must be 38. Li + < Al 3+ < Mg 2+ < K +
greater than lattice energy.
Size decreases from left to right in a period and it increases
(b) Correct Greater lattice energy discourage dissolution of a from top to bottom in a group. Variation is more
salt.
pronounced in group than in period.
(c) Incorrect When a salt dissolve, energy is required to
break the lattice, which comes from hydration process. 39. Zn = 3d 10 4 s2 ,
(d) Incorrect Explained in (A). Cu = 3d 10 4 s1
25. Statement I and II are true and Statement II is the correct The first ionisation energy is greater for Zn but reverse is
explanation of statement I. true for 2nd ionisation energy.
26. Statement I is true. Stronger oxidising agent is one which itself 40. Ionic size
can easily be reduced. Pb 4+ is unstable, due to inert pair effect, Mg 2+ < Na + < F- < O2 - < N3 -
can easily be reduced to stable Pb 2+ , hence a stronger oxidising
Already explained in question 1 (i).
agent than Sn 4+ .
Statement II is false. Due to inert pair effect, the higher 41. The first ionisation energy of carbon is greater than the
oxidation states of group 14 elements becomes less stable for same of boron as predicted from periodic trend. However,
heavier member. for 2nd

27. Both statements I and II are true and Statement II is the correct B+ = 1s2 ; more stable than C+ =1s2 2s2
explanation of statement I. 2s2 2p1
28. Statement I is true Be has higher first ionisation energy than B ionisation trend is reversed due to stability of completely
which is against periodic trend. filled 2s-orbital of B+ :
Statement II is false 2s-orbital is lower in energy than 2 p, 42. Size Ca 2+ < Ar < Cl - < S2 -
Aufbaus principle. Explained in (i), question 6.
29. Statement I is true; Statement II is false. 43. (i) Mg2+ , O2- , Na + and F - are all isoelectronic, has
F atom has slightly lower affinity for the electron than chlorine. 10 electrons each. Among isoelectronic species,
It is due to the reason that additional electrons are repelled more the order of size is cation < neutral < anion.
effectively by 2 p-electrons in F than by 3 p-electrons in Also, between cations, higher the charge, smaller the
Cl-atom. size and between anions, greater the negative charge,
30. Inert pair effect-favours lower oxidation state. larger the size. Therefore, the decreasing order of
ionic radii : O2 - > F- > Na + > Mg 2 +
31. Higher effective nuclear charge due to greater p/e ratio.
IP + EA (ii) First ionisation energy increases from left to right in a
32. Electronegativity = (Mulliken formula) period. However, exception occur between group 2
2
and 13 and group 15 and 16 where trend is reversed on
33. Electron affinitydefinition. the grounds of stability of completely filled and
34. Basic nature of hydroxides increases down a group. completely half-filled orbitals. Therefore,
Ionisation energy (1st) : Na < Al < Mg < Si
35. Cl has maximum electron affinity, hence the correct order is
(iii) If the atoms are from same period, bond length is
Cl > F > Br inversely proportional to bond order. In a group, bond
36. Ionisation potential decreases down the group but this is not the length is related directly to atomic radius. Therefore,
only criteria of reducing power. bond length N2 < O2 < F2 < Cl 2
37. In a group, size increases from top to bottom.
CHAPTER TEST
N0 /2 atom of X (g) are converted into X + (g) by energy and anti-clockwise (2), while on pseudo earth there is an
E1. N0 / 2 atoms of X (g) are converted into X - (g ) by additional possible value of spin quantum number called
energy E2 . Hence, ionisation potential and electron neutral spin (3) in which 1 2 3
electron is believed to be
affinity of X (g) are
fluctuating harmonically
2 E 2 2 ( E1 - E 2 ) 2 E1 2 E 2 between clockwise and
(a) , (b) ,
N0 N0 N0 N0 anti-clockwise directions,
( E1 - E 2 ) 2 E 2 about its axis. Answer the
(c) , (d) None is correct
N0 N0 following three question cw cw
based on the above
The electron affinities of N, O, S and Cl are such that information.
(a) N < O < S < Cl (b) O < N < Cl < S
The first noble gas on pseudo earth would be
(c) O Cl < N S (d) N < S < O < Cl
(a) He (b) H (c) Li (d) Ne
If Aufbau and Hunds rule are not used, then incorrect
The long form of periodic table on this pseudo earth
statement is
+ will have how many groups?
(a) K would be coloured ion
(b) Na will be in same s-block (if these rules are true) (a) 18 (b) 24 (c) 27 (d) 36
(c) Cu would be s-block element On pseudo earth, atomic number of the first transition
(d) Magnetic moment of Cr(24) would be zero metal would be
The compound vanadium has magnetic moment of (a) 21 (b) 26 (c) 29 (d) 31
1.73 BM. The vanadium chloride has the formula
Assertion-Reason Type
(a) VCl2 (b) VCl3 (c) VCl4 (d) VCl5
Following two questions have Assertion followed by the
Which of the following can be acceptable electronic Reason. Answer them according to the following options.
configuration of carbon atom in the ground state? (a) Both Assertion and Reason are correct and Reason is
the correct explanation of the Assertion.
(a) 1s2 2s2 (b) 1s2 2s2 (b) Both Assertion and Reason are correct but Reason is
2p 2p not the correct explanation of Assertion.
(c) Assertion is correct but Reason is incorrect.
(c) (d)
1s2 2s2 1s2 2s2 (d) Assertion is incorrect but Reason is correct.
2p 2p Assertion Nitrogen has higher first ionisation energy
Which of the following statement is(are) correct than oxygen.
regarding periodic table? Reason Atomic radius of nitrogen is smaller than that
(a) In a period, ionisation energy increases monotonically of oxygen.
from left to right
(b) Electronegativity increases from left to right Assertion SnCl2 is a good reducing agent while
(c) Electron affinity decreases monotonically from top to PbCl2 is stable although both Sn and Pb belongs to
bottom in a group same group of periodic table.
(d) Transition elements starts only from fourth period of
the periodic table Reason Pb show inert pair effect.

Passage for Q.Nos. 7 to 9 Match the statements of Column I with values of


Lets consider a hypothetical planet pseudo Earth which Column II.
is similar to our earth in several aspects. The similarities Column I Column II
are A. Be (Z = 4) p. Paramagnetic
On pseudo earth:
B. C (Z = 6) q. Diamagnetic
(i) There are same number of elements as on our earth
and they are known by the same name. C. Mg2+ (Z = 12) r. No valence shell orbital without
(ii) Paulis exclusion principal, Hund's Rule and Aufbau electron
principle are known to the people of pseudo earth in D. N (Z =7) s. Has higher IE than both
the same manner as we know on our earth. elements, on left and right in the
(iii) They classify elements as representative, transition period.
and inner-transition elements in the same manner as
we classify on our earth. In ground state of oxygen, in how many ways its
p-electrons can be represented in box diagram
However, there is one basic difference in understanding
the electrons spin on these two earths. On our earth the without violating either Hund's rule or Paulis exclusion
electron can have only two spin directions, 'clock wise (1) principle?

Answers
1. (b) 2. (d) 3. (a) 4. (c) 5. (a,b,d) 6. (a,c) 7. (c) 8. (c) 9. (d) 10. (c)
11. (a) 12. A q,s; B p; C q,r,s; D p,r,s 13. (6)
4
Chemical Bonding
Preliminary Concepts of Electrovalent and (ii) If there is any unpaired electrons in the molecular orbitals,
Covalent Bonding Including Bonding system will be paramagnetic otherwise diamagnetic. For
paramagnetic system,
l
Chemical bonds are formed as a result of instability of
spin-only magnetic moment (m ) = n ( n + 2 ) BM
individual isolated atoms. Through chemical bonding, atoms
or ions associate and gain stability. There are three important where n = Number of unpaired electrons
types of chemical bonds by which atoms or ions connects to (iii) A system with zero bond order means that system does not
one another: exist.
(i) Covalent bond (ii) Ionic bond l
Ionic Bond This type of bond is formed by transfer of
(iii) Metallic bond electrons from a highly electropositive element to a highly
l
Covalent Bond This type of bond is formed by sharing of electronegative element, viz.
electrons of the valence shell. Sharing of electrons may e-
occur in following ways: Na + Cl Na + Cl -
+ l
Salient Features of Ionic Bonding
s-bond (i) In ionic bonding, the oppositely charged ions are held together
Sigma bond is formed by direct overlap of orbitals as shown. by very strong electrostatic force of attraction.
p (ii) In solid state, each cation surrounds itself with anions and each
anion with cations. These very large number of ions are
s arranged in an ordinary network called ionic crystals.
+
(iii) Ionic compounds are good conductors of electricity in fused
state or in aqueous solution.
(iv) Ionic compounds have very high melting and boiling points.
Pi ( p ) covalent bond is formed by sidewise (or lateral) overlapping (v) Ionic compounds develop some extent of covalency under the
of atomic p-orbitals as shown above in the diagram. following conditions:
+
(a) If the cation has very small size or high charge, e.g. Li + or
A + B A B Al 3+ .
(b) If the anion has very large size or high negative charge, e.g.
Coordinate
covalent bond I- or S2- .
Properties of covalent system are explained with the help of (c) If the cation has partially or fully filled d or f orbitals in
various theories of which the two most common are valence penultimate shell.
shell electron pair repulsion theory and molecular orbital The above factors lowers the ionic character and makes
theory. physical properties characteristic of ionic compounds inferior.
Metallic Bond Metallic bonding is a strong cohesive
l
Highlights of MOT
interaction developed between positively charged metal ions
(i) Bond order and the sea of free electrons existing in metals. These cohesive
No. of bonding electrons - No. of antibonding electrons
= force of attraction is responsible for all the special
2 characteristics of metals.
Chemical Bonding 45

VBT, Hybridisation and VSEPR Theory


l
Valence Shell Electron Pair Repulsion Theory (VSEPR Theory) This theory is based on the concepts of hybridisation
of atomic orbitals. According to this theory, pure atomic orbitals do not take part in bonding, rather the valence shell orbitals
first hybridises and the resulting hybrid orbitals are involved in bond formation. Based on the geometries of hybrid orbitals
at the central atom, molecular structure is proposed and physiochemical properties are interpreted.

Summary of molecular geometries and hybridisation


Total Number of Number of Number of Orbital Molecular Distortion* Hybri- Bond Examples
indepen-dent independent non- geometry geometry disation angle
electron pairs bonding bonding
electron electron
pairs pairs
2 2 0 Linear Linear No sp 180 CO2 , BeCl 2

Trigonal Trigonal sp 2 BF3 , NO-3 ,


3 3 0 Possible 120
planar planar GaI3
Trigonal Bent sp 2 NO-2 , SO2 ,
3 2 1 Always 120
planar (non-linear) SnCl 2

sp 3 CH4 , SO2-
4 ,
4 4 0 Tetrahedral Tetrahedral Possible 109
CF4
Trigonal sp 3
4 3 1 Tetrahedral Always 109 SO2-
3 , NH3
pyramidal
Bent or
4 2 2 Tetrahedral Always sp 3 109 H2 O, OF2 , NH-2
V-shaped

5 5 0 Trigonal Trigonal sp 3 d 90 and PF , PCl


Possible 5 5
Bipyramidal bipyramidal 120

5 4 1 Trigonal sp 3 d 90 and SF4 , TeCl 4 ,


See-saw Always
bipyramidal 120 XeO2 F2

5 3 2 Trigonal sp 3 d IF3
T-shaped Always 90
bipyramidal
Trigonal
5 2 3 Linear Never sp 3 d 180 I-3 , IF2-
bipyramidal
Octahedral
6 6 0 (square Octahedral Possible sp 3 d 2 90 SF6 , PF6-
bipyramidal)

6 5 1 Square sp 3 d 2 IF5 , BrF5


Octahedral Always 90
pyramidal
XeF4 , ICl -4 ,
6 4 2 Octahedral Square planar Possible sp 3 d 2 90
BrF4-

NOTE
Under distortion possible means that the bond angle will be distorted when the outer atoms are not identical, e.g. CH4 is tetrahedral with all other
atoms identical, no distortion, but in CH2F2 , tetrahedral geometry but outer atoms are different, bond angles will be distorted, i.e. some will be slightly
greater than 109 and some smaller than 109.
l
Note on Molecular Geometry The following types of molecules are non-polar:
46 Chemical Bonding

AB2 -linear, AB3 -trigonal planar, AB4 -tetrahedral, AB5 -trigonal bipyramidal, AB6 -octahedral, AB4 -square planar, AB2C 2 -symmetric
and square planar. Some examples of these categories:
Type Shape Examples
AB 2 Linear BeCl 2 , CO2 , CS2
AB 3 Trigonal planar BF3 , SO3 , GaF3
AB 4 Tetrahedral CH4 , CCl 4 , SiF4
AB 5 Pentagonal bipyramidal PCl 5 , AsF5
AB 6 Octahedral (Square bipyramidal) SF6 , SeF6
AB 4 Square planar XeF4
Any other molecule having a single central atom will be polar as long as the electronegativities of elements that make it up are not
same. This includes molecules that have the above mentioned geometries but dissimilar outer atoms, e.g. CH3 F, OCS, BeFCl, etc.

Resonance, LCAO, MOT, Other Bonding Types


l
Molecular Orbital Theory (MOT) According to MOT, atomic orbitals first combines to form molecular orbitals which
finally participate in bond formation. Combination of atomic orbitals infact pertains to combination of electronic waves and
it may occur in two ways known as constructive interference and destructive interference. Constructive interference leads to
formation a resultant wave that is called bonding molecular orbitals while destructive interference gives rise to
antibonding molecular orbitals. Potential energy of a bonding molecular orbital is always less than the potential energies
of atomic orbitials combined and potential energy of an antibonding molecular orbital is always greater than the potential
energies of atomic orbitals combined. After bond formation, electrons occupy these molecular orbitals. Molecular orbitals
are filled by electrons in increasing order of energies. During filling of molecular orbitals, both Hunds rule and Pauli
exclusion principle are obeyed. The increasing order of energy of molecular orbitals in a diatomic system are
(i) For oxygen and heavier system,
p 2 py p
*
2 py *
s 1s s
*
1s s 2 s s
*
2 s s 2 px s 2 px
p 2 pz *
p2 pz
(ii) For nitrogen and lighter system,
p 2 py p
*
2 py
s 1s s
*
1s s 2 s s
*
2s s 2 px s
*
2 px
p 2 pz
p
*
2 pz
(Starred molecular orbitals denote antibonding molecular orbitals)
Electron filling in molecular orbitals of second period homonuclear diatomic molecules are shown below:

Li2 Be2 B2 N2 O2
*
s2p x
*
s2p x
*
s2p x p*2py, p* 2pz
s2px s2px p2py, p2pz
p2py, p2pz p2py, p2pz p2py, p2pz s2px
* * * * *
s2s s2s s2s s2s s2s
s2s s2s s2s s2s s2s
* * * * *
s1s s1s s1s s1s s1s
s1s s1s s1s s1s s1s
Topic 1 Preliminary Concepts of Electrovalent and Covalent
Bonding Including Bonding
Objective Questions I (Only one correct option) Objective Questions II
1. The intermolecular interaction that is dependent on the (One or more than one correct option)
inverse cube of distance between the molecules is (2015 Main)
11. Dipole moment is shown by (1986, 1M)
(a) ion-ion interaction (b) ion-dipole interaction
(c) London force (d) hydrogen bond
(a) 1, 4-dichlorobenzene (b) cis-1, 2-dichloroethene
(c) trans-1, 2-dichloroethene (d) trans-1, 2-dichloro-2- pentene
2. The nodal plane in the p-bond of ethene is located in
(a) the molecular plane (2002, 3M) Assertion and Reason
(b) a plane parallel to the molecular plane
(c) a plane perpendicular to the molecular plane which bisects the
Read the following questions and answer as per the direction
carbon-carbon s-bond at right angle given below:
(d) a plane perpendicular to the molecular plane which contains (a) Statement I is true; Statement II is true; Statement II is the
the carbon-carbon s-bond correct explanation of Statement I
3. Amongst H2 O, H2 S, H2 Se and H2 Te, the one with the (b) Statement I is true; Statement II is true; Statement II is
highest boiling point is (2000, 1M) not the correct explanation of Statement I
(a) H2O because of hydrogen bonding (c) Statement I is correct; Statement II is incorrect
(b) H2Te because of higher molecular weight (d) Statement I is incorrect; Statement II is correct
(c) H2S because of hydrogen bonding
12. Statement I LiCl is predominantly a covalent compound.
(d) H2Se because of lower molecular weight
Statement II Electronegativity difference between Li and Cl
4. Arrange the following compounds in order of increasing is too small. (1998, 2M)
dipole moment, toluene (I), m-dichlorobenzene (II),
o-dichlorobenzene (III), p-dichlorobenzene (IV) (1996, 1M) Fill in the Blanks
(a) I < IV < II < III (b) IV < I < II < III 13. There are p bonds in a nitrogen molecule. (1982, 1M)
(c) IV < I < III < II (d) IV < II < I < III
5. The number and type of bonds between two carbon atoms in True/False
CaC2 are (1996, 1M) 14. All molecules with polar bonds have dipole moment.
(a) one sigma (s) and one pi (p ) bonds (1985,
1
M)
(b) one sigma (s) and two pi (p ) bonds 2
(c) one sigma (s) and one half pi (p ) bonds 15. Linear overlapping of two atomic p-orbitals leads to a sigma
(d) one sigma (s) bond bond. (1983, 1M)
6. The molecule which has zero dipole moment is (1989, 1M)
(a) CH2Cl 2 (b) BF3 Subjective Questions
(c) NF3 (d) ClO2 16. Arrange the following ions in order of their increasing radii:
7. Element X is strongly electropositive and element Y is Li + , Mg 2+ , K + , Al 3+ . (1997, 1M)
strongly electronegative. Both are univalent. The compound 17. Between Na + and Ag + , which is stronger Lewis acid and
formed would be (1980, 1M) why? (1997, 3M)
(a) X + Y - (b) X -Y + -
18. In the reaction, I + I2 I-3 , which is the Lewis acid?
(c) X -- Y (d) X Y
(1997, 1M)
8. Which of the following compound is covalent? (1980, 1M)
(a) H2 (b) CaO
19. Explain the difference in the nature of bonding in LiF and LiI.
(1996, 2M)
(c) KCl (d) Na 2S
-29
9. The total number of electrons that take part in forming the 20. The dipole moment of KCl is 3.336 10 C-m which
bonds in N2 is (1980, 1M)
indicates that it is a highly polar molecule. The interatomic
(a) 2 (b) 4 (c) 6 (d) 10 distance between K + and Cl - in this molecule is
2.6 10-10 m. Calculate the dipole moment of KCl molecule
10. The compound which contains both ionic and covalent if there were opposite charges of one fundamental unit
bonds is (1979, 1M)
located at each nucleus. Calculate the percentage ionic
(a) CH4 (b) H2 (c) KCN (d) KCl character of KCl. (1993, 2M)
48 Chemical Bonding

21. Give reasons in two or three sentences only for the 22. State four major physical properties that can be used to
following : distinguish between covalent and ionic compounds. Mention
Hydrogen peroxide acts as an oxidising as well as a the distinguishing features in each case. (1978, 2M)

reducing agent. (1992, 1M)

Topic 2 VBT, Hybridisation and VSEPR Theory


Objective Questions I (Only one correct option) 12. The correct order of hybridisation of the central atom in the
1. The correct statement for the molecule, CsI3 is (2014 Main) following species NH3 , [PtCl 4 ]2 - , PCl 5 and BCl 3 is
(a) it is a covalent molecule (a) dsp2 , dsp3 , sp2 and sp3 (2001, 1M)
(b) it contains Cs+ and I-3 ions (b) sp3 , dsp2 , sp3d and sp2
(c) it contains Cs3+ and I- ions (c) dsp2 , sp2 , sp3 and dsp3
(d) it contains Cs+ , I- and lattice I2 molecule (d) dsp 2 , sp 3 , sp 2 and dsp 3
2. The species having pyramidal shape is (2010) 13. The common features among the species CN , CO and NO+
(a) SO3 (b) BrF3 are (2001, 1M)
(c) SiO2-
3 (d) OSF2 (a) bond order three and isoelectronic
3. Assuming that Hunds rule is violated, the bond order and (b) bond order three and weak field ligands
magnetic nature of the diatomic molecule B 2 is (2010) (c) bond order two and acceptors
(a) 1 and diamagnetic (b) 0 and diamagnetic (d) isoelectronic and weak field ligands
(c) 1 and paramagnetic (d) 0 and paramagnetic 14. The hybridisation of atomic orbitals of nitrogen in
4. The species having bond order different from that in CO is NO+2 , NO-3 and NH+4 are (2000, 1M)
(a) NO- (b) NO+ (2007, 3M)
(a) sp, sp3 and sp2 respectively
-
(c) CN (d) N2 (b) sp, sp2 and sp3 respectively
5. Among the following, the paramagnetic compound is (c) sp2 , sp and sp3 respectively
(a) Na 2O2 (b) O3 (2007, 3M) (d) sp2 , sp3 and sp respectively
(c) N2O (d) KO2 15. In the compound CH2 == CH CH2 CH2 C CH, the
6. Which of the following contains maximum number of lone C2 C3 bonds is of (1999, 2M)
pairs on the central atom? (2005, 1M) (a) sp - sp2 (b) sp3 - sp3
(a) ClO -3 (b) XeF 4 (c) sp - sp3 (d) sp2 - sp3
(c) SF 4 (d) I -3 16. The geometry of H2 S and its dipole moment are (1999, 2M)
7. Number of lone pair(s) in XeOF4 is/are (2004, 1M) (a) angular and non-zero (b) angular and zero
(a) 0 (b) 1 (c) 2 (d) 3 (c) linear and non-zero (d) linear and zero

8. Which of the following are isoelectronic and isostructural ? 17. The geometry and the type of hybrid orbital present about the
NO3 , CO2 ClO3 , central atom in BF3 is (1998, 2M)
3 , SO3 (2003, 1M)
(a) linear, sp (b) trigonal planar, sp2
(a) NO3 , CO2- (b) SO3 , NO3
3 (c) tetrahedral, sp3 (d) pyramidal, sp3
(c) ClO3 , CO2
3 (d) CO2
3 , SO 3
18. Which one of the following compounds has
9. Among the following, the molecule with the highest dipole sp 2 - hybridisation? (1997, 1M)
moment is (2003, 1M) (a) CO2 (b) SO2 (c) N2 O (d) CO
(a) CH 3Cl (b) CH 2Cl 2
(c) CHCl 3 (d) CCl 4 19. Among KO2 , AlO-2 , BaO2 and NO+2 , unpaired electron is
present in (1997 C, 1M)
10. Which of the following molecular species has unpaired
(a) NO+2 and BaO2 (b) KO2 and AlO2-
electron (s)? (2002, 3M)
(a) N2 (b) F2 (c) O2 (d) O2 (c) Only KO2 (d) Only BaO2
2
-
11. Specify the coordination geometry around and hybridisation 20. The cyanide ion CN and N2 are isoelectronic, but in contrast
of N and B atoms in a 1 : 1 complex of BF3 and NH3 . to CN- , N2 is chemically inert because of (1997 C, 1M)
3
(a) N : tetrahedral, sp ; B: tetrahedral, sp 3
(2002, 3M)
(a) low bond energy
(b) N : pyramidal, sp3; B: pyramidal, sp3 (b) absence of bond polarity
(c) N: pyramidal, sp3; B: planar, sp2 (c) unsymmetrical electron distribution
(d) N: pyramidal, sp3; B: tetrahedral, sp3 (d) presence of more number of electron in bonding orbitals
Chemical Bonding 49

21. Among the following species, identify the isostructural pairs. (d) three orbitals in a plane
NF3 , NO3- , +
BF3 , H3 O , N3 H (1996, 1M) 34. Carbon tetrachloride has no net dipole moment because of
(a) its planar structure
(a) [NF3 ,NO-3 ] and [BF3 ,H3O+ ] (1983, 1M)
(b) [NF3 , N3H] and [NO-3 ,BF3 ] (b) its regular tetrahedral structure
(c) [ NF3 , H3O+ ] and [NO3 , BF3 ] (c) similar sizes of carbon and chlorine atoms
(d) [NF3 , H3O+ ] and [N3H, BF3 ] (d) similar electron affinities of carbon and chlorine
22. Which one of the following molecules is planar? (1996, 1M) 35. The ion that is isoelectronic with CO is (1982, 1M)

(a) NF3 (b) NCl 3 (c) PH3 (d) BF3 (a) CN- (b) O+2 (c) O-2 (d) N+2

23. The maximum possible number of hydrogen bonds a water 36. Among the following, the linear molecule is (1982, 1M)

molecule can form is (1992, 1M) (a) CO2 (b) NO2 (c) SO2 (d) ClO2
(a) 2 (b) 4 (c) 3 (d) 1 37. If a molecule MX 3 has zero dipole moment, the sigma
24. The type of hybrid orbitals used by the chlorine atom in ClO-2 bonding orbitals used by M (atomic number < 21) are
is (a) pure p (b) sp-hybridised (1981, 1M)
(1992, 1M)
3 2 (c) sp2-hybridised (d) sp3-hybridised
(a) sp (b) sp
(c) sp (d) None of these
Objective Questions II
25. The molecule which has pyramidal shape is (1989, 1M)
(a) PCl 3 (b) SO3 (c) CO2 (d) NO3
(One or more than one correct option)
3
38. The molecules that will have dipole moment are (1992, 1M)
26. Which of the following is paramagnetic? (1989, 1M)
(a) 2, 2-dimethyl propane (b) trans-2-pentene
(a) O2 (b) CN (c) cis-3-hexene (d) 2, 2, 3, 3-tetramethyl butane
(c) CO (d) NO+
39. Which of the following have identical bond order?
27. The ClCCl angle in 1, 1, 2, 2-tetrachloroethene and (a) CN (b) O2 (1992, 1M)
tetrachloromethane respectively will be about (1988, 1M) (c) NO+ (d) CN+
(a) 120 and 109.5 (b) 90 and 109.5 40. The linear structure assumed by (1991, 1M)
(c) 109 and 90 (d) 109.5 and 120
(a) SnCl 2 (b) CS2 (c) NO+2 (d) NCO
28. The molecule that has linear structure is (1988, 1M) (e) SO2
(a) CO2 (b) NO2
41. CO2 is isostructural with (1986, 1M)
(c) SO2 (d) SiO2
2 (a) HgCl 2 (b) C2H2 (c) SnCl 2 (d) NO2
29. The species in which the central atom uses sp -hybrid
orbitals in its bonding is (1988, 1M)
(a) PH3 (b) NH3 (c) CH+3 (d) SbH3 Match the Columns
42. Match the orbital overlap figures shown in Column I with the
30. Of the following compounds, which will have a zero dipole
description given in Column II and select the correct answer
moment ? (1987, 1M) using the codes given below the Columns. (2014 Adv.)
(a) 1, 1-dichloroethylene
(b) cis-1, 2-dichloroethylene Column I Column II
(c) trans-1, 2-dichloroethylene
(d) None of the above A. 1. p-d p antibonding
31. The hybridisation of sulphur in sulphur dioxide is (1986, 1M)
(a) sp (b) sp3 (c) sp2 (d) dsp2
B. 2. d-d s bonding
32. The bond between two identical non-metal atoms has a pair
of electrons (1986, 1M)
(a) unequally shared between the two
(b) transferred fully from one atom to another C. 3. p-dp bonding
(c) with identical spins
(d) equally shared between them
33. On hybridisation of one s and one p-orbital we get D. 4. d-d s antibonding
(a) two mutually perpendicular orbitals (1984, 1M)
(b) two orbitals at 180 Codes
(c) four orbitals directed tetrahedrally A B C D A B C D
50 Chemical Bonding

(a) 4 3 2 1 (b) 1 2 3 4
(c) 2 3 1 4 (d) 4 1 2 3 58. SnCl 2 is a non-linear molecule. 1
(1985, M)
2
43. Match each of the diatomic molecules in Column I with its
property/properties in Column II. (2009) Integer Type Questions
Column I ColumnII 59. A list of species having the formula XZ4 is given below
A. B2 p. Paramagnetic (2014 Adv.)
XeF4 , SF4 , SiF4 , BF4- , BrF4- , [Cu(NH3 )4 ] 2+ , [FeCl 4 ] 2- ,
B. N2 q. Undergoes oxidation
[CoCl 4 ] 2- and [PtCl 4 ] 2-
C. O-2 r. Undergoes reduction Defining shape on the basis of the location of X and Z
D. O2 s. Bond order 2 atoms, the total number of species having a square planar
shape is
t. Mixing of s and p orbitals
60. The total number of lone-pair of electrons in melamine is
Codes (2013 Adv.)
A B C D 61. Based on VSEPR theory, the number of 90 FBrF
(a) q, r, s p, r, t, s q, r, t p, q, t angles in BrF5 is (2010)
(b) p, q, r, t q, r, s, t p, q, r, t p, r, s, t
(c) q, r, s, t p, q, r r, s, t p, q, r, t Subjective Questions
(d) p, q, s, t p, q, s p, t q, r, t
62. Predict whether the following molecules are isostructural
or not. Justify your answer.
Fill in the Blanks (i) NMe3 (ii) N(SiMe3 )3 (2005, 2M)
44. Among N2 O, SO2 , I3+ and I3 , the linear species are and 63. On the basis of ground state electronic configuration,
(1997 C, 1M) arrange the following molecules in increasing OO bond
length order. KO2 , O2 , O2 [AsF6 ] (2004, 2M)
45. When N2 goes to N+2 , the N N bond distance , and when
64. Draw the shape of XeF4 and OSF4 according to VSEPR
O2 goes to O+2 the O O bond distance (1996, 1M)
theory. Show the lone pair of electrons on the central atom.
46. The two types of bonds present in B2 H6 are covalent and (2004, Main, 2M)
(1994, 1M) 65. Using VSEPR theory, draw the shape of PCl 5 and BrF5 .
47. The kind of delocalisation involving sigma bond orbitals is (2003, 2M)
called................. (1994, 1M)
66. Draw the molecular structures of XeF 2 , XeF 4 and XeO2 F2 ,
48. The valence atomic orbitals on C in silver acetylide is indicating the location of lone pair(s) of electrons.
.............hybridised. (1990, 1M) (2000, 3M)

49. The shape of CH3 is . + 67. Interpret the non-linear shape of H2 S molecule and
(1990, 1M) non-planar shape of PCl 3 using valence shell electron pair
50. hybrid orbitals of nitrogen atom are involved in the repulsion (VSEPR) theory. (Atomic number : H = 1,
formation of ammonium ion. (1982, 1M)
P = 15, S = 16, Cl = 17) (1998, 4M)

51. Pair of molecules which forms strongest intermolecular 68. Using the VSEPR theory, identify the type of hybridisation
and draw the structure of OF2 . What are the oxidation states
hydrogen bonds is . (SiH4 and SiF4 , acetone and of O and F ? (1997, 3M)
CHCl 3 , formic acid and acetic acid) (1981, 1M)
69. Write the Lewis dot structural formula for each of
52. The angle between two covalent bonds is maximum in . the following. Give also, the formula of a neutral
(CH4 , H2 O, CO2 ) (1981, 1M) molecule, which has the same geometry and the same
arrangement of the bonding electrons as in each of the
True/False following. An example is given below in the case of H3 O+
53. The dipole moment of CH3 F is greater than that of CH3 Cl. and NH3 .
+
(1993, 1M) H H
54. H2 O molecule is linear.
(1993, 1M)
H O H H N H
55. The presence of polar bonds in a polyatomic molecule suggests
that the molecule has non-zero dipole moment.
(1990, 1M) . .
3
Lewis dot Neutral molecule
56. sp hybrid orbitals have equal s and p character. (1987, 1M)
structure (1983, 4M)

57. In benzene, carbon uses all the three p-orbitals for (i) O2-
2 (ii) CO2-
3

hybridisation. (1987, 1M) (iii) CN- (iv) NCS-


Topic 3 Resonance, LCAO, MOT, Other Bonding Types
Objective Questions I (Only one correct option) Objective Question II
1. Assuming 2s-2p mixing is not operative, the (One or more than one correct option)
paramagnetic species among the following is (2014 Adv.) 11. Hydrogen bonding plays a central role in which of the following
(a) Be2 (b) B2 (c) C2 (d) N2
phenomena? (2014 Adv.)
2. Stability of the species Li 2 , Li 2- and Li +2 increases in the (a) Ice floats in water
order of (2013 Main) (b) Higher Lewis basicity of primary amines than tertiary amines in
(a) Li 2 < Li +2 < Li -2 (b) Li 2 < Li +2 < Li 2 aqueous solutions
(c) Li 2 < Li -2 < Li +2 (d) Li -2 < Li 2 < Li +2 (c) Formic acid is more acidic than acetic acid
(d) Dimerisation of acetic acid in benzene
3. In which of the following pairs of molecules/ions both the
species are not likely to exist? (2013 Main) 12. Which one of the following molecules is expected to exhibit
(a) H+2 , He2-
2 (b) H -2 , He22- diamagnetic behaviour? (2013 Main)
(c) H22+ , He2 (d) H-2 , He2+
2
(a) C 2 (b) N 2
(c) O 2 (d) S 2
4. Hyperconjugation involves overlap of which of the
following orbitals? (2008, 3M)
Assertion and Reason
(a) s - s (b) s - p (c) p - p (d) p - p
Read the following questions and answer as per the direction given
5. According to MO theory, (2004, 1M)
below :
(a) O+2 is paramagnetic and bond order greater than O2
(a) Statement I is correct; Statement II is correct; Statement II is the
(b) O+2 is paramagnetic and bond order less than O2
correct explanation of Statement I.
(c) O+2 is diamagnetic and bond order is less than O2
(b) Statement I is correct; Statement II is correct; Statement II is not
(d) O+2 is diamagnetic and bond order is more than O2
the correct explanation of Statement I.
6. Molecular shape of SF4 , CF4 and XeF4 are (2000, 1M) (c) Statement I is correct; Statement II is incorrect.
(a) the same, with 2, 0 and 1 lone pair of electrons (d) Statement I is incorrect; Statement II is correct.
respectively
13. Statement I The electronic structure of O 3 is
(b) the same, with 1, 1 and 1 lone pair of electrons
+
respectively O O
(c) different, with 0, 1 and 2 lone pair of electrons -

respectively O O . Statement II O O structure is

(d) different, with 1, 0 and 2 lone pair of electrons
respectively not allowed because octet around O cannot be expanded.
(1998, 2M)
7. In compounds of type ECl 3 , where E = B, P, As or Bi, the
angles ClECl is in order (1999, 2M) Match the Columns
(a) B > P = As = Bi
14. Match the reactions in Column I with nature of the
(b) B > P > As > Bi
reactions/type of the products in Column II. (2007, 6M)
(c) B < P = As = Bi
(d) B < P < As < Bi Column I Column II
8. The correct order of increasing C O bond length of A. O-2 O2 + O22 - 1. Redox reaction
CO, CO23 - , CO2 is (1999, 2M)
One of the products has
B. CrO24 - + H+ 2.
(a) CO23-
< CO2 < CO trigonal planar structure
(b) CO2 < CO2
3 < CO MnO-4 + NO-2 Dimeric bridged
(c) CO < CO2
3 < CO 2
C. 3.
+ H+ tetrahedral metal ion
(d) CO < CO2 < CO32
9. Which contains both polar and non-polar bonds? NO-3 + H2 SO4
D. 4. Disproportionation
(a) NH4Cl (b) HCN (1997, 1M) + Fe2+
(c) H2O2 (d) CH4
10. Which one among the following does not have the Codes
hydrogen bond? (1983, 1M) A B C D A B C D
(a) Phenol (b) Liquid NH3 (a) 2 1, 4 3 4 (b) 1, 4 3 1, 2 1
(c) Water (d) HCl (c) 2 3 1 4 (d) 3 4 2, 3 1
52 Chemical Bonding

Subjective Questions
15. Write the MO electron distribution of O2 . Specify its bond order and magnetic property. (2000, 3M)

16. Arrange the following as stated.


Increasing strength of hydrogen bonding ( X H X ).
O, S, F, Cl, N (1991, 1M)
17. What effect should the following resonance of vinyl chloride have on its dipole moment? (1987, 1M)
CH2 ==CH Cl CH2 C+ HCl

Topic 1
1. (b) 2. (a) 3. (a) 4. (b)
5. (b) 6. (b) 7. (a) 8. (a) 41. (a, b) 42. A 2; B 3; C 1; D 4
9. (c) 10. (c) 11. (b, d) 12. (c) 43. A p, q, r, t; B q, r, s, t; C p, q, r, t; D p, r, s, t
13. (2) 14. F 15. T 44. N 2O, I 3- 45. increases, decreases

Topic 2 46. three centre bond-two electrons


1. (b) 2. (d) 3. (a) 4. (a) 47. hyperconjugation
5. (d) 6. (d) 7. (b) 8. (a) 48. sp 49. Triangular planar 50. sp 3
9. (a) 10. (c) 11. (a) 12. (b) 51. HCOOH and CH3COOH 52. CO2 53. F
13. (a) 14. (b) 15. (d) 16. (a) 54. F 55. F 56. F 57. F
17. (b) 18. (b) 19. (c) 20. (b) 58. T 59. (4) 60. (6) 61. (0)
21. (c) 22. (d) 23. (b) 24. (a)
Topic 3
25. (a) 26. (a) 27. (a) 28. (a)
1. (c) 2. (b) 3. (c) 4. (b)
29. (c) 30. (c) 31. (c) 32. (d)
5. (a) 6. (d) 7. (b) 8. (a)
33. (b) 34. (b) 35. (a) 36. (a)
9. (c) 10. (d) 11. (a, b, d) 12. (a, b)
37. (c) 38. (b, c) 39. (a, c) 40. (b, c, d)
13. (a) 14. A 1, 4; B 3; C 1, 2; D 1
Topic 1 Preliminary Concepts of Toluene is less polar than both ortho and para
Electrovalent and Covalent dichlorobenzene. Therefore, the increasing order of dipole
Bonding Including Bonding moment is
p-dichlorobenzene < toluene < m-dichlorobenzene
1. Ion-ion interaction is dependent on the square of distance, < o-dichlorobenzene
1
i.e. ion-ion interaction 5. The carbide (C2-
2 ) ion has the following bonding pattern:
r2

1 : C C : one sigma and two pi bonds.
Similarly, ion-dipole interaction 3
r
6. BF3 has triangular planar arrangement. F
1 1
London force and dipole-dipole interaction Three identical vectors acting in outward FB 120
r6 r3
Superficially it seems as both ion-dipole interaction and direction at equal angles in a plane cancel each F
hydrogen bonding vary with the inverse cube of distance other giving zero resultant, hence non-polar. sp2
between the molecules but when we look at the exact 7. Strongly electropositive, univalent X will form an 1 : 1 ionic
expressions of field (force) created in two situations, it comes compound with strongly electronegative, univalent Y .
as X + Y X +Y -
2| P |
| E | or | F | = (In case of ion-dipole interaction) 8. H2 is a covalent, diatomic molecule with a sigma covalent
4 p r3 bond between two hydrogen atoms.
2 2
2q r 4 q a 9. N2 has triple bond and each covalent bond is associated with
and F = (In case of dipole-dipole interaction)
4 p 0 r3 one pair of electrons, therefore, six electrons are involved in
forming bonds in N2 .
From the above, it is clear that the ion-dipole interaction is
the better answer as compared to dipole-dipole interaction, 10. In KCN, the bonding between potassium ion and cyanide ion
i.e. hydrogen bonding. is ionic while carbon and nitrogen are covalently bonded in
cyanide ion as:
Covalent bonds
H H +
2. [K] [ C N]
CC
H H Ionic bond

11. 1, 4-dichlorobenzene is non-polar, individual dipole vectors


Pi bond is formed by the p-orbitals whose lobes have minima cancel each other.
in the plane of molecule, hence molecular plane is the nodal m0
plane of pi-bond. Cl Cl H Cl ClCH2 H
3. H-bond is the strongest intermolecular force. C==C C==C C==C
All are different with 1, 0 and 2 lone pairs of electrons at H H Cl Cl C2 H 5
H
central atom. m=0
Polar Non-polar Polar
4. p-dichlorobenzene is non-polar.
12. Statement I is correct but Statement II is incorrect. The
Cl Cl covalency in LiCl is due to small size of Li + ion which brings
about large amount of polarisation in bond.
p-dichlorobenzene
The two dipole vectors cancelling each other giving zero 13. These are 2p-bonds in a nitrogen molecule.
resultant dipole moment. o-dichlorobenzene has greater 14. The resultant of individual bond dipoles may or may not be
dipole moment than meta isomer. non-zero.
Cl
15. Linear overlapping of p-orbitals form sigma bond while
Cl m1 > m2 sidewise overlapping of two p-orbitals forms a pi bond.
m1 m2 16. Li + < Al 3+ < Mg 2+ < K +
Cl Cl 17. Ag + is stronger Lewis acid because it can easily
(o-dichlorobenzene) (m-dichlorobenzene) accommodate lone pair of electrons from Lewis base. On the
dipole vectors are at 60 angle dipole vectors are at 120 angle other hand, Na + has noble gas configuration, cannot accept
lone pair of electron, not at all a Lewis acid.
54 Chemical Bonding

18. I2 is Lewis acid because I - coordinate its one lone pair to I2 . No unpaired electron-diamagnetic.
bonding electrons - antibonding electrons
19. Both LiF and LiI are expected to be ionic compounds. Bond order =
2
However , LiI is predominantly covalent because of small
6-4
size of Li + and large size of iodide ion. A smaller cation and a = =1
larger anion introduces covalency in ionic compound. 2
20. Dipole moment is calculated theoretically as 4. The bond order of CO = 3. NO+ , CN- and N2 are
isoelectronic with CO, have the same bond order as CO. NO-
m = qd
(16e- ) has bond order of 2.
Here, q = 1.6 10-19 C and d = 2.6 10-10 m 5. O-2 in KO2 has 17 electrons, species with odd electrons are
-19 -10 -29
m Theo = 1.6 10 2.6 10 = 4.16 10 cm always paramagnetic.
m obs 3.336 10-29
% ionic character = 100 = 100 6. ClO-3 : - O Cl == O one lone pair at Cl.
m Theo 4.16 10-29
= 80.2% O
21. In hydrogen peroxide (H2 O2 ), oxygen is in 1 oxidation
XeF4 : F Xe two lone pairs at Xe.
state, can be oxidised to O2 (zero oxidation state) or can be
reduced to H2 O (2 oxidation state of oxygen). Hence, H2 O2 F F
F
can act as both oxidising agent and reducing agent. With
strong oxidising agent like KMnO4 , H2 O2 acts as a reducing SF4 : F one lone pair at S.
agent while with strong reducing agent like H2 C2 O4 , it acts F
S
as an oxidising agent.
F
22. (i) Melting points Ionic compounds have higher melting F
points than covalent compounds. -
-
(ii) Boiling points Ionic compounds have higher boiling I3 : I I I three lone pairs at central iodine.

points than covalent compounds.
O
(iii) Solubility Ionic compounds have greater solubility in F F
water than a covalent compound.
7. Xe At central atom (Xe), there is one lone pair.
(iv) Conductivity in aqueous solution Ionic compounds
F F
have greater electrical conductivity in aqueous solution
while covalent compounds are usually non-conducting. 8. NO-3 and CO2-
3 both have 32 electrons, central atom
sp 2 hybridised, triangular planar.
Topic 2 VBT, Hybridisation and
VSEPR Theory 9. CH3 Cl has the highest dipole moment.
1. I -3 -
is an ion made up of I 2 and I which has linear shape. 10. O-2 has odd number(17) of electrons, therefore it must
+
While Cs is an alkali metal cation. contain at least one unpaired electron.
2. F H

FS==O S
11. F B- N+ H Both B and N sp 3 tetrahedral.

F F O
F
F H
S is sp3 hybridised Pyramidal
SO3 is planar (S is sp 2 hybridised), BrF3 is T-shaped and 12. NH3 = sp 3 ,[ PtCl 4 ]2 - = dsp 2 , PCl 5 = sp 3 d , BCl 3 = sp 2
SiO2- 2
3 is planar (Si is sp hybridised).
3. For molecules lighter than O2 , the increasing order of 13. All three have 14 electrons (iso electronic) with bond order
energies of molecular orbitals is of three.
* O H
p2 p y p 2 p y .....
s1s s
*
1s s 2s s
*
2s s 2 p s
*
2 p
p2 p z *
x x + +
H
p 2 p z O==N==O, ON==O, N
14.
H
where, p2 p y and p2 p z are degenerate molecular orbitals, sp sp2 sp3 H
first singly occupied and then pairing starts if Hunds rule is 1 2 3 4 5 6
obeyed. If Hunds rule is violated in B2 , electronic 15. CH2 ==CH CH2 CH2 C CH
arrangement would be
p2 p 2y Hybridisation at C2 = sp 2 and at C3 = sp 3 .
s1 s2 s
*
1s2 s2s2 s
*
2s2 ...
p2 p z
Chemical Bonding 55

16. H2 S has sp 3 hybridised sulphur, therefore, angular in shape 22. BF3 has triangular planar arrangement.
with non-zero dipole moment. F
H FB 120
F
S sp2-hybridised
H There identical vectors acting in outward direction, at equal
angles in a plane, cancel each other giving zero resultant,
(Non-linear, polar molecule) hence non-polar.
F
23. A water molecule can form at the most four H-bonds.
17. FB
F H
sp2 O
(Trigonal planar)

18. Sulphur in SO2 is sp 2 -hybridised. H


Four sites of H-bonding
S
-
O O 24. O Cl == O

Electron pair = 2 (s-bonds) + 1 (lone pair) = 3 electron pairs at Cl = 2 (s-bonds) + 2 (lone-pairs) = 4
Hybridisation = sp 2 Hybridisation at Cl = sp 3
Carbon in CO2 is sp-hybridised, N in N2 O is sp-hybridised,
carbon in CO is sp-hybridised.
25. PCl 3 has sp 3 -hybridised phosphorus, with one lone pair.
Therefore, molecule has pyramidal shape like ammonia.
19. Molecular orbital electronic configuration are
26. O-2 has odd number of electrons, hence it is paramagnetic.
p2 p 2y p*
2 p 2y * 0
KO2 (O-2 ) : s1s2 s 1s s2s2 s
* 2 *
2s2 s 2 px2 s 2 px Cl
p 2 p 2z * 1 Cl Cl 109
p2 pz
27. C==C 120 C Cl
Has one unpaired electron in p
*
2 p orbital. Cl Cl Cl
AlO-2 has both oxygen in O2- state, therefore, no unpaired sp2-hybridised
Cl
sp3-hybridised
electron is present.
BaO2 (O2- 28. CO2 is linear because carbon is sp-hybridised.
2 )
p 2 p 2y p 2 p 2y *
*
29. In CH+3 , there are only three electron pairs around carbon
s1s2 s 1s s2s2 s
* 2 *
2s2 s2 px2 s 2 px0
p 2 p 2z atom giving sp 2 -hybridisation state.
p
*
2 p 2z
Has no unpaired electron. +
H
+
NO+2 has [O== N==O] bonding, hence no unpaired electron. HC
H
20. N2 is a neutral, non-polar, inert molecule while CN - is a sp2-hybridised
highly polar, highly active ion.
30. Dipole vectors in trans-1, 2-dichloroethylene are at 180 and
F
directed in opposite direction, cancelling each other.
21. NF3 : N F BF3 : FB
H Cl
F F
F Triangular planar C==C dipole moment = 0
Pyramidal (B-sp2)
(N-sp3) Cl H

O
+ 31. In SO2 , the Lewis-dot structure is
+ O
NO3 : H 3O : H
O==N H O == S == O
O H
Triangular planar Pyramidal Electron pairs at S = 2 (s-bonds) + 1 (lone-pair) = 3
(O-sp3)
(N-sp2) sp 2 hybridised.
+ NOTE
N3 H N == N == N H
p-bonded electrons are not present in hybrid orbitals, therefore not
Central nitrogen is sp - hybridised counted in electron pairs. Rather p bonds are formed by lateral
overlapping of pure p-orbitals.
Therefore, NF3 , H3 O+ and BF3 , NO-3 pairs have same shape.
56 Chemical Bonding

32. Bonds between identical non-metal is purely covalent due to Any orbital has two phase +ve and ve. In the following
same electronegativities of the bonded atoms. Hence, the diagram, +ve phase is shown by darkening the lobes and ve by
bonded atoms have equal holds on the shared pair of without darkening the lobes.
electrons.
33. Hybridisation of one s and one p orbitals gives two
sp hybrid orbitals oriented linearly at 180.
s + p 2 sp hybrid orbitals Bonding MO Antibonding MO
34. CCl 4 has a regular tetrahedral shape. When two same phase overlap with each other, it forms
Cl bonding molecular orbital otherwise antibonding.

m s-bond
C m Cl Net dipole = 0
Cl
Cl
35. CO has a total of 14 electrons and CN- also has 14 electrons. p-bond
- - - - -
C (6e ) + N (7e ) + e CN (14 e )
36. CO2 is a linear molecule because of sp-hybridisation On the basis of above two concepts, correct matching can be
around carbon atom. done as shown below:
37. For non-polar MX 3 , it must have triangular planar
arrangement, i.e. there should be sp 2 -hybridisation
around M. A. d-d s bonding
CH3
H3 C H
38. H3 C C CH3 C== C
H CH2 CH3
CH3 Polar B. p-d p bonding
Symmetric, non-polar

CH3 CH3
CH3 H2 C CH2 CH3
C== C H3 C C C CH3 C. p-d p antibonding
H H
CH3 CH3
Polar Symmetric, non-polar
- +
39. CN and NO are isoelectronic, have the same bond order
of 3.
+ D. d-d s antibonding
-
40. S== C==S O== N==O O C N
Linear Linear Linear

Sn S \ A 2, B 3, C 1, D 4
Cl Cl O O Hence, (c) is the correct option.
Bent Bent
p2 p1y
41. CO2 , HgCl 2 , C2 H2 are all linear. 43. (A) B2 : s1s2 s* 1s2 s2 s2 s* 2s2 paramagnetic.
p 2 p1z
42. PLAN This problem includes basic concept of bonding. It
can be solved by using the concept of molecular orbital 6-4
Bond order = =1
theory. 2
+ ve phase Bond is formed by mixing of s and p orbitals.
B2 undergoes both oxidation and reduction as
Heat
B2 + O2 B2 O3 (Oxidation)
ve phase B2 + H2 B2 H6 (Reduction)
Chemical Bonding 57

p2 p 2y 56. In sp 3 -hybrid orbital, there is 25 % s-character and 75 %


(B) N2 : s1s2 s
*
1s2 s2 s2 s
*
2 s2 s2 px2 diamagnetic. p-character.
p 2 p 2z
57. Carbon in benzene is sp 2 -hybridised, i.e. uses only two of its
10 - 4 p-orbitals in hybridisation.
Bond order = =3>2
2 58. Sn in SnCl 2 has sp 2 -hybridisation.
N2 undergoes both oxidation and reduction as
59. PLAN This problem includes concept of hybridisation using VBT,
D
N2 + O2 NO VSEPR theory, etc.,
XeF4 , BrF4- , [Cu(NH3 )4 ]2+ , [PtCl 4 ]2- are square planar as
Catalyst
N2 + 3H2 NH3 shown below:
In N2 , bonds are formed by mixing of s and p orbitals. s
F F F
- p2 p 2y p
*
2 p 2y
F
(C) O2 : s1s2 s
*
1s2 s2 s2 s
*
2 s2 s2 px2 s
*
2 px0 Xe Br
p 2 p 2z p
*
2 p1z F F F F
-
Paramagnetic with bond order = 1.5. undergoes both
O2 2+ 2
oxidation and reduction and bond involves mixing of s and H3N NH3 Cl Cl
p-orbitals. Cu Pt
H3N NH3 Cl Cl
p2 p 2y p
*
2 p1y
(D) O2 : s1s s1s s2 s s 2 s
2 * 2 2 * 2
s2 px2 s 2 px0
*
SF4 (See-saw) as shown below:
p 2 p 2z
p
*
2 p1z F
Paramagnetic with bond order = 2. F
O2 undergoes reduction and the bond involves mixing of S
F
s and p-orbitals. F
44. N2 O and I-3 are linear species. SiF4 , BF4- , [FeCl 4 ]2 - , [CoCl 4 ]2 - are tetrahedral as shown
45. Bond order in N2 is 3 while same in N+2
is 2.5, hence bond below:
distance increases as N2 goes to N+2 .
Bond order in O2 is 2 while same in O+2 is 2.5, hence bond
distance decreases as O2 goes to O+2 .
46. Three centred-2 electrons.
47. Hyperconjugation involves delocalisation of s-electrons.
48. sp-hybridised.
49. Triangular planar. Carbon in CH+3 is sp 2 hybridised.
50. sp 3 -hybrid orbital holding the lone pair is involved in
formation of ammonium ion.
O O Hence, correct integer is 4.

51. H C OH and CH3 C OH . Both are capable of 60. PLAN Melamine is a heterocyclic compound.

forming H-bonds. H 2N N NH2


52. CO2 , it is 180. N N
53. Dipole moment (m ) = q.d
NH2
Since electronegativity of F and Cl are very close, it is the
internuclear distance (d) that decides dipole moment here. Each nitrogen atom has one pair of lone pair
Hence, C Cl bond has greater dipole moment the C-F Thus, in all six lone pairs.
bond. F
F F
54. H2 O is V-shaped molecule.
61. Br
O
H H F F
V-shaped
Lone pair would push the BrF bond pairs in upward
55. Explained in 2. direction and all BrF bond angles will contract.
58 Chemical Bonding

62. No, (i) NMe3 is pyramidal while (ii) N(SiMe3 )3 is planar. In Topic 3 Resonance, LCAO, MOT,
the latter case, pp - dp back bonding between N and Si makes
N sp 2 -hybridised.
Other Bonding Types
63. Bond order : O2 - = 1.5, O2 = 2, O+2 = 2.5 1. PLAN This problem can be solved by using the concept involved in
molecular orbital theory. Write the molecular orbital electronic
Bond length : O+2 < O2 < O2- configuration keeping in mind that there is no 2s-2p mixing,
then if highest occupied molecular orbital contain unpaired
F F electron then molecule is paramagnetic otherwise diamagnetic.
F F Assuming that no 2s-2p mixing takes place the molecular
64. orbital electronic configuration can be written in the
Xe O S
following sequence of energy levels of molecular orbitals
F * *
F
F s1s, s1s, s2s, s 2s, s2 p z , p2 px p2 p y ,
F
Square planar Trigonal bipyramidal
* * *
F p 2 px p 2 p y , s 2 p z
Cl Cl
F F
* *
(a) Be2 s1s2 , s1s2 , s2s2 , s 2s2 (diamagnetic)
65. Cl P Br
* * p2 px0
(b) B2 s1s2 , s1s2 , s2s2 , s 2s2 , s2 p 2z ,
Cl
F F p2 p 0y
Cl
Trigonal bipyramidal Square pyramidal (diamagnetic)
(P is sp3d-hybridised) (Br is sp3d 2 -hybridised) * * p 2 p 1

(c) C2 s1s2 , s1s2 , s2s2 , s 2s2 , s2 p 2z , x,


F F p2 p1y
F F O
* 0
p 2 px * 0
, s 2 p z (paramagnetic)
Xe Xe Xe * 0
* * p2 p p2 px2
66. (d) N2 s1s2 , s1s2 , s2s2 , s 2s2 , sy 2 p 2z , ,
F F
O p2 p 2y
F F
Linear Square planar See-saw shaped * 0
p 2 px
, s * 2 p 0z (diamagnetic)
67. In H2 S, S is sp 3 -hybridised with two lone pairs of electrons * 0
p2 py
on it giving V-shaped (water like) shape. In PCl 3 , P is
sp 3 -hybridised with one lone pair of electrons on it. Hence, (c) is the correct choice.

Therefore, PCl 3 is pyramidal in shape. 2. Li 2 ( 3 + 3 = 6 ) = s1s2 , s* 1s2 , s2s2


+2 Nb - Na 4 - 2
O Bond order = = =1
-1 2 2
68. F O F F F-1
V-shaped Li +2 ( 3 + 3 - 1 = 5 ) = s1s2 , s* 1s2 , s2s1
3-2 1
Electron pair = P = 2 + 2 = 4 Bond order = = = 0.5
2 2
Hybridisation = sp 3 -
Li 2 (3 + 3 + 1 = 7 ) = s1s2 , s* 1s2 , s2s2 s* 2s1
2-

4-3 1
2 : O O
69. (i) O2- and Cl Cl (Cl 2 ) Bond order = = = 0.5

2 2
2-
Stability order is Li -2 < Li +2 < Li 2 (because Li -2 has more
O
number of electrons in antibonding orbitals which

(ii) CO3 :
2-
C and F B F (BF3 ) destabilises the species).


O O 3. Species having zero or negative bond order do not exist.
F
2 (1 + 1 - 2 = 0 ) = s1s
H2+ 0

-

Bond order = 0
(iii) CN - : C N and C O (CO) He2 ( 2 + 2 = 4 ) = s1s2 , s* 1s2


- N - Na 2 - 2

Bond order = b = =0
(iv) NCS- : S C N and Cl C N (ClCN) 2 2


So, both H2+
2 and He 2 do not exist.
Chemical Bonding 59

4. C2 ( 6 + 6 = 12 ) = s1s2 , s* 1s2 , s2s2 , s* 2s2 , p2 px2 p2 p 2y 10. H2 O2 H


Since, all the electrons are paired, it is a diamagnetic OO
species. polar bond
H
N2 ( 7 + 7 = 14 ) = s1s2 , s
* 1s2, s2s2 , Non-polar bond
* 2s2 , p2 px2 p2 p 2y , s2 p 2z
s 11. HCl does not form hydrogen bond. For formation of
It is also a diamagnetic species because of the absence of hydrogen bond, atleast one hydrogen atom must be bonded to
unpaired electrons. one of the three most electronegative atom O , N and F.
O2 (8 + 8 = 16) 12. PLAN This problem can be solved by using concept of H-bonding and
or S2 = s1s2 , s* 1s2 , s2s2 , s* 2s2 , applications of H-bonding.
* 2 p1 *p 2 p1
s 2 p 2 , p2 p 2 p2 p 2 p 13. Statement I is correct, given structure is one of the resonance
z x y x y

Due to the presence of two unpaired electrons, O2 and S2 structure of ozone.


both are paramagnetic molecules. + +
O O
H H
O O- -O
-
O
5. H C CH == CH2 H C == CH CH2 Statement II is also correct because oxygen cannot expand its
+
H octet. It is also the explanation for the given structure of ozone.
H
(I) (II) 14. (A) In the reaction : O-2 O2 + O22 -
I and II are hyperconjugation structures of propene and 1
Oxygen on reactant side is in - oxidation state. In
involves s-electrons of CH bond and p-orbitals of pi bond 2
product side, one of the oxygen is in zero oxidation state,
in delocalisation. i.e. oxidised while the other oxygen is in 1 oxidation
p2 p 2y p
*
2 p1y state, i.e. reduced. Hence, in the above reaction, oxygen
6. O+2 (15e- ) : s1s2 s* 1s2 s2s2 s* 2s2 s2 px2 s
*
2 px0 ( O-1 / 2 ) is simultaneously oxidised and reduced
p 2 p 2z disproportionated.
p
*
2 p 0z
(B) In acidic medium, CrO2- 2-
4 is converted into Cr2 O7 which
10 - 5
Bond order = = 2.5; paramagnetic. is a dimeric, bridged tetrahedral.
2 O O
p2 p 2y p
*
2 p1y
O2 (16e- ) : s1s2 s 1s s2s2 s
* 2 *
2s2 s2 px2 s
*
2 px Cr Cr
O
p 2 p 2z O O
p 2 p1z
*
O O
10 - 6 (C) MnO-4 + NO-2 + H+ Mn 2+ + NO-3
Bond order = =2
2
Hence, (a) is the correct answer. The above is a redox reaction and a product NO-3 has
trigonal planar structure.
F F F F F (D) NO-3 + H2 SO4 + Fe2+ Fe+ + NO

S Xe The above is a redox reaction.


7. C
F
F F F F p2 p 2y p
*
2 p1y
F F 15. (a) O2 : s1s2 s* 1s2 s2s2 s* 2s2 s2 px2
p 2 p 2z
See-saw shape Tetrahedral Square planar p
*
2 p1z
molecule
10 - 6
8. When E = B in BCl 3 , bond angle is 120. When E = P, As or Bond order = = 2, paramagnetic.
3 2
Bi in ECl 3 , hybridisation at E will be sp . Also, if central
atoms are from same group, bond angle decreases down the 16. Strength of hydrogen bonding in XHX depends on
group provided all other things are similar. Hence, the order electronegativity as well as size of X . X with higher
of bond angles is BCl 3 > PCl 3 > AsCl 3 > BiCl 3 electronegativity and smaller size forms stronger H-bond.
1 Hence, increasing order of strength of H-bond is
9. Bond length S < Cl < N < O < F
Bond order
1 4
Bond order : CO2 = 2, CO = 3 , CO23 - = 1 + = 17. Resonance in vinyl chloride increases polar character of the
3 3 molecule.
Therefore, order of bond length is CO3 < CO2 < CO
2-
CHAPTER TEST
1. Which of the following molecules would be Comprehension for Q. Nos. 9 to 11
expected to be planar?
VSEPR-theory can be used to describe the shapes of
1. NH3 2. XeF4
SO 2- 2-
3 , SO 3 and SO 4 . From their shapes, it appears that
3. SF4 4. ICl -4
some or all of them are capable of exhibiting the
(a) 1, 2 and 4 (b) 2 and 3
phenomenon of resonance.
(c) 3 and 4 (d) 2 and 4
2. Which set of species is arranged in order of 9. Which of the following will have the strongest S O bond?
increasing ONO bond angle? (a) SO 2-
3 (b) SO 3
(a) NO -2 , NO+2 , NO -3 (b) NO -3 , NO -2 , NO +2 (c) SO 2-
4 (d) All have equal strength

(c) NO +2 , NO 2, NO -2 (d) NO -2 , NO -3 , NO +2 10. Which of the following will have the atoms lying in the
same plane?
3. Which of the following could be the atom X in the (a) SO 2- (b) SO 3
3
following neutral (uncharged) molecule?
(c) SO 2-
4 (d) None of these

X
11. Which of the following will have the smallest value of
OSO bond angle?
N
(a) SO 2-
3 (b) SO 3

X F

(c) SO 2-
4 (d) SO 2-
3 and SO 3

(a) F (b) C Assertion-Reason Type


(c) N (d) O
Following two questions have assertion followed by the
4. In the molecule C3Hm, the X-ray diffraction (which reason. Answer them according to the following options.
does not see Hs) finds CCC bond angle of (a) Both Assertion and Reason are correct and Reason is the
about 120. The value of m must be correct explanation of Assertion
(a) 3 (b) 4 (b) Both Assertion and Reason are correct but Reason is not
(c) 5 (d) 6 the correct explanation of Assertion
(c) Assertion is correct but Reason is incorrect
5. Which of the following is thermally less stable (d) Assertion is incorrect but Reason is correct
than CaCO3?
(a) Na 2CO 3 (b) SrCO 3
12. Assertion The COC bond angle in CH3OCH3 is
closer to 109 while the SiOSi bond angle is
(c) BaCO 3 (d) CuCO 3
H3SiOSiH3 is closer to 120.
6. Which have fractional bond order? Reason Carbon has greater electronegativity than
(a) O +2 (b) O -2 (c) NO (d) H+2 silicon.

7. In the structure of H2CSF4 (S is the central atom), 13. Assertion H2 molecule is more stable than a He-H
molecule.
which of the following statements is/are true?
(a) The two CH bonds are in the same plane of Reason The antibonding electron in He-H molecule
axial SF bonds decreases the bond order and therefore the stability.
(b) The two CH bonds are in the same plane of 14. Match Column I with Column II.
equatorial SF bonds
(c) Total six atoms are in the same plane Column I Column II
(d) Equatorial SF bonds are perpendicular to nodal A. XeO2F2 p. Polar
plane of p-bond
B. XeF4 q. Non-polar
8. The statement which is/are true regarding
C. AsF5 r. Planar
molecular structure of SOF4 (ignore bond angle
distortion) D. SOCl2 s. Non-planar
(a) It has three equivalent SF bonds which are
15. In the molecule SOF4, how many two fold axis of symmetry
longer than the fourth SF bond
is(are) present?
(b) The FSF bond angles are both 90 and 120
(c) The OSF bond angles are both 90 and 120 16. In the molecule COCl2, the maximum number of atomic
(d) It possesses a two fold axis of symmetry orbitals involved in hybridisation at any one atom is/are

Answers
1. (a) 2. (d) 3. (d) 4. (d) 5. (d) 6. (a,b,c,d) 7. (a,c,d) 8. (b,c,d) 9. (b) 10. (b)
11. (a) 12. (b) 13. (a) 14. A p,s; B q,r; C q,s; D p,s 15. (1) 16. (4)

You might also like